Assessment CC Flashcards

1
Q

Convert 1 L to mL

A. 100
B. 1,000
C. 10,000
D. 100,000

A

B. 1,000

1 L (1 × 10^0) = ? mL (milli = 10^–3); move the decimal place three places to the right and it becomes 1,000 mL; reverse the process to determine the expression in L (move the decimal three places to the left of 1,000 mL to get 1 L).

How well did you know this?
1
Not at all
2
3
4
5
Perfectly
2
Q

Convert 50 mL to L

A. 0.5
B. 0.05
C. 5
D. 500

A

B. 0.05

50 mL (milli = 10^-3 and is smaller) = ? L (larger by 10^3); move the decimal by three places to the left and it becomes 0.050 L.

How well did you know this?
1
Not at all
2
3
4
5
Perfectly
3
Q

Convert 5 dL to mL

A. 0.5
B. 50
C. 500
D. 5,000

A

C. 500

5 dL (deci = 10^-1 and is larger) = ? mL (milli = 10^-3 and is smaller by 10^–2); move the decimal place two places to the right and it becomes 500 mL.

How well did you know this?
1
Not at all
2
3
4
5
Perfectly
4
Q

Which of the following containers is calibrated to hold only one exact volume of liquid?

A. Volumetric flask
B. Erlenmeyer flask
C. Griffin beaker
D. Graduated cylinder

A

A. Volumetric flask

A Class A volumetric flask is calibrated to hold one exact volume of liquid (TC)

How well did you know this?
1
Not at all
2
3
4
5
Perfectly
5
Q

Which of the following standards requires that SDSs are accessible to all employees who come in contact with a hazardous compound?

A. Hazard Communication Standard
B. Bloodborne Pathogen Standard
C. CDC Regulation
D. Personal Protection Equipment Standard

A

A. Hazard Communication Standard

In the August 1987 issue of the Federal Register, OSHA published the new Hazard Communication Standard (Right to Know Law, 29 CFR 1910.1200).
The Right to Know Law was developed for employees who may be exposed to hazardous chemicals in the workplace.
Employees must be informed of the health risks associated with those chemicals.

How well did you know this?
1
Not at all
2
3
4
5
Perfectly
6
Q

Chemical should be stored:

A. According to their chemical properties and classification
B. Alphabetically, for easy accessibility
C. Inside a safety cabinet with proper ventilation
D. Inside a fume hood, if toxic vapors can be released when opened

A

A. According to their chemical properties and classification

Arrangements for the storage of chemicals will depend on the quantities of chemicals needed and the nature or type of chemicals.
Proper storage is essential to prevent and control laboratory fires and accidents.
Ideally, the storeroom should be organized so that each class of chemicals is isolated in an area that is not used for routine work.

How well did you know this?
1
Not at all
2
3
4
5
Perfectly
7
Q

Proper personal protection equipment (PPE) in the chemistry laboratory for ROUTINE testing includes:

A.Respirators with HEPA filter
B. Gloves with rubberized sleeves
C. Safety glasses for individuals not wearing contact lenses
D. Impermeable lab coat with gloves

A

D. Impermeable lab coat with gloves

How well did you know this?
1
Not at all
2
3
4
5
Perfectly
8
Q

A fire caused by a flammable liquid should be extinguished by using which type of extinguisher?

A. Halogen
B. Class B
C. Pressurized water
D. Class C

A

B. Class B

How well did you know this?
1
Not at all
2
3
4
5
Perfectly
9
Q

Which of the following is the proper means of disposal for the type of waste?

A. Xylene into the sewer system
B. Microbiologic waste by steam sterilization
C. Mercury by burial
D. Radioactive waste by incineration

A

B. Microbiologic waste by steam sterilization

How well did you know this?
1
Not at all
2
3
4
5
Perfectly
10
Q

Which of the following are examples of NONIONIZING RADIATION?

A. Ultraviolet light and microwaves
B. Gamma rays and x-rays
C. Alpha and beta radiation
D. Neutron radiation

A

A. Ultraviolet light and microwaves

How well did you know this?
1
Not at all
2
3
4
5
Perfectly
11
Q

10 6th

A. Kilo
B. Mega
C. Milli
D. Micro

A

B. Mega

How well did you know this?
1
Not at all
2
3
4
5
Perfectly
12
Q

The prefix which means 10 -9 is:

A. Micro
B. Milli
C. Nano
D. Pico

A

C. Nano

How well did you know this?
1
Not at all
2
3
4
5
Perfectly
13
Q

Concentration expressed as the amount of solute per 100 parts of solution:

A. Molarity
B. Normality
C. Percent solution
D. Ratio

A

C. Percent solution

How well did you know this?
1
Not at all
2
3
4
5
Perfectly
14
Q

Indication of relative concentration:

A. Dilution
B. Molarity
C. Normality
D. Ratio

A

A. Dilution

How well did you know this?
1
Not at all
2
3
4
5
Perfectly
15
Q

What is the molarity of a solution that contains 18.7 grams of KCl in 500 mL (MW 74.5)?

A. 0.1
B. 0.5
C. 1.0
D. 5.0

A

B. 0.5

M = 18.7 g / (74.5)(0.5L) = 0.5 M

How well did you know this?
1
Not at all
2
3
4
5
Perfectly
16
Q

How much 95% v/v alcohol is required to prepare 5L of 70% v/v alcohol?

A. 2.4 L
B. 3.5 L
C. 3.7 L
D. 4.4 L

A

C. 3.7 L

C1V1 = C2V2
V1 = (70)(5L) / 95 = 3.7 L

How well did you know this?
1
Not at all
2
3
4
5
Perfectly
17
Q

A colorimetric method calls for the use of 0.1 mL serum, 5 mL of the reagent and 4.9 mL of water. What is the dilution of the serum in the final solution?

A. 1 to 5
B. 1 to 10
C. 1 to 50
D. 1 to 100

A

D. 1 to 100

Amount of serum: 0.1 mL
Total volume: 10 mL
Dilution: 0.1:10 or 1:100

How well did you know this?
1
Not at all
2
3
4
5
Perfectly
18
Q

Convert 72 Fahrenheit to its Celsius equivalent:

A. 12.2C
B. 22.2C
C. 40.2C
D. 44.4C

A

B. 22.2C

C = 5/9 (F - 32)
= 5/9 (72 - 32)
= 22.2C

How well did you know this?
1
Not at all
2
3
4
5
Perfectly
19
Q

Convert 100 Celsius to its Kelvin equivalent:

A. 73.15K
B. 173.15K
C. 273.15K
D. 373.15K

A

D. 373.15K

K = C + 273.15
= 100 + 273.15
= 373.15K

How well did you know this?
1
Not at all
2
3
4
5
Perfectly
20
Q

Most basic pipette:

A. Automatic pipette
B. Glass pipette

A

B. Glass pipette

ROUTINELY USED: automatic pipette
MOST BASIC: glass pipette

How well did you know this?
1
Not at all
2
3
4
5
Perfectly
21
Q

Does not have graduations to the tip:

A. Mohr pipet
B. Serologic pipet
C. Micropipet
D. None of these

A

A. Mohr pipet

A Mohr pipet does not have graduations to the tip.
It is a self-draining pipet, but the tip should not be allowed to touch the vessel while the pipet is draining.

How well did you know this?
1
Not at all
2
3
4
5
Perfectly
22
Q

Pipets are used with biologic fluids having a viscosity greater than that of water:

A. Mohr pipets
B. Ostwald-Folin pipets
C. Pasteur pipets
D. Volumeteric pipets

A

B. Ostwald-Folin pipets

Ostwald-Folin pipets are used with biologic fluids having a viscosity greater than that of water. They are blowout pipets, indicated by two etched continuous rings at the top.

How well did you know this?
1
Not at all
2
3
4
5
Perfectly
23
Q

Pipette with BULB CLOSER TO THE DELIVERY TIP and are used for accurate measurement of VISCOUS FLUIDS, such as blood or serum:

A. Ostwald-Folin pipette
B. Volumetric pipette

A

A. Ostwald-Folin pipette

How well did you know this?
1
Not at all
2
3
4
5
Perfectly
24
Q

Pipette with cylindrical glass bulb near the CENTER of the pipette that helps to distinguish them from other types of transfer pipettes.

A. Ostwald-Folin pipette
B. Volumetric pipette

A

B. Volumetric pipette

How well did you know this?
1
Not at all
2
3
4
5
Perfectly
25
Q

Extremely inert, excellent temperature tolerance and chemical resistance; used for stir bars, stopcocks and tubing:

A. Polyethylene
B. Polycarbonate
C. Polystyrene
D. Teflon

A

D. Teflon

POLYETHYLENE
- Widely used in plastic ware, too, including test tubes, bottles, graduated tubes, stoppers, disposable transfer pipets, volumetric pipets, and test tube racks.
- May bind or absorb proteins, dyes, stains, and picric acid

POLYCARBONATE
- Used in tubes for centrifugation, graduated cylinders, and flasks
- Usable temperature range is broad: –100° C to +160° C
- Very strong plastic but is not suitable for use with strong acids, bases, and oxidizing agents
- May be autoclaved but with limitations

POLYSTYRENE
- Rigid, clear type of plastic that should not be autoclaved
- Used in an assortment of tubes, including capped graduated tubes and test tubes
- Not resistant to most hydrocarbons, ketones, and alcohols

TEFLON
- Widely used for manufacturing stirring bars, tubing, cryogenic vials, and bottle cap liners
- Almost chemically inert and is suitable for use at temperatures ranging from –270° C to +255° C
- Resistant to a wide range of chemical classes, including acids, bases, alcohol, and hydrocarbons

How well did you know this?
1
Not at all
2
3
4
5
Perfectly
26
Q

It is used when rapid centrifugation of solutions containing small particles is needed; an example is the microhematocrit centrifuge:

A. Horizontal-head centrifuge
B. Fixed-angle head centrifuge
C. Ultracentrifuge
D. Cytocentrifuge

A

B. Fixed-angle head centrifuge

How well did you know this?
1
Not at all
2
3
4
5
Perfectly
26
Q

Horizontal-head centrifuge:

A. Cytocentrifuge
B. Fixed-angle head centrifuge
C. Swinging bucket centrifuge
D. Ultracentifuge

A

C. Swinging bucket centrifuge

HORIZONTAL-HEAD OR SWINGING BUCKET CENTRIFUGE
* HORIZONTAL WHEN MOVING OR SPINNING
* VERITICAL WHEN NOT MOVING
Cups holding the tubes of material to be centrifuged occupy a vertical position when the
centrifuge is at rest but assume a horizontal position when the centrifuge revolves

How well did you know this?
1
Not at all
2
3
4
5
Perfectly
27
Q

High-speed centrifuges used to separate layers of different specific gravities, commonly used to separate lipoproteins:

A. Horizontal-head centrifuge
B. Fixed-angle head centrifuge
C. Ultracentrifuge
D. Cytocentrifuge

A

C. Ultracentrifuge

ULTRACENTRIFUGE
- High-speed centrifuges used to separate layers of different specific gravities
- Commonly used to separate lipoproteins
- Usually refrigerated to counter heat produced through friction

How well did you know this?
1
Not at all
2
3
4
5
Perfectly
28
Q

Uses a very high-torque and low-inertia motor to spread MONOLAYER OF CELLS rapidly across a special slide for critical morphologic studies:

A. Horizontal-head centrifuge
B. Fixed-angle head centrifuge
C. ltracentrifuge
D. Cytocentrifuge

A

D. Cytocentrifuge

CYTOCENTRIFUGE
- Uses a very high-torque and low-inertia motor to spread monolayers of cells rapidly across a special slide for critical morphologic studies
- Used for blood, urine, body fluid, or any other liquid specimen that can be spread on a slide

How well did you know this?
1
Not at all
2
3
4
5
Perfectly
29
Q

The speed of the centrifuge should be checked every 3 months with:

A. Tachometer
B. Wiper
C. Potentiometer
D. Ergometer

A

A. Tachometer

How well did you know this?
1
Not at all
2
3
4
5
Perfectly
30
Q

Calibration of centrifuges is customarily performed every ______.

A. Daily
B. Weekly
C. Every 3 months (quarterly)
D. Yearly

A

C. Every 3 months (quarterly)

Photoelectric tachometer or strobe tachometer
CAP recommends that the number of revolutions per minute for a centrifuge used in chemistry laboratories be checked every 3 months

How well did you know this?
1
Not at all
2
3
4
5
Perfectly
31
Q

Centrifuges are routinely disinfected on a _ basis.

A. Daily
B. Weekly
C. Monthly
D. Quarterly

A

B. Weekly

Calibration of centrifuges is customarily performed every 3 months, and the appropriate relative centrifugal force for each setting is recorded.
Centrifuges are routinely disinfected on a weekly basis.

How well did you know this?
1
Not at all
2
3
4
5
Perfectly
32
Q

HIGHLY PURIFIED SUBSTANCES of a known composition:

A. Control
B. Standard

A

B. Standard

A standard may differ from a control in its overall composition and in the way it is handled in the test.
Standards are the best way to measure ACCURACY. Standards are used to establish reference points in the construction of graphs (e.g., manual hemoglobin curve) or to calculate a test result.

How well did you know this?
1
Not at all
2
3
4
5
Perfectly
33
Q

It represents a specimen that is SIMILAR IN COMPOSITION TO THE PATIENT’S WHOLE BLOOD or plasma:

A. Control
B. Standard

A

A. Control

The value of a control specimen is known. Control specimens are tested in exactly the same way as the patient specimen and are tested daily or in conjunction with the unknown (patient) specimen.
Controls are the best measurements of PRECISION and may represent normal or abnormal test values.

How well did you know this?
1
Not at all
2
3
4
5
Perfectly
34
Q

Water produced using either an anion or a cation EXCHANGE RESIN, followed by replacement of the removed ions with hydroxyl or hydrogen ions.

A. Deionized water
B. Distilled water
C. RO water

A

A. Deionized water

How well did you know this?
1
Not at all
2
3
4
5
Perfectly
35
Q

The PUREST TYPE OF REAGENT WATER is:

A. Type I
B. Type II
C. Type III
D. All are equal

A

A. Type I

How well did you know this?
1
Not at all
2
3
4
5
Perfectly
36
Q

Chemicals that are used to manufacture drugs:

Technical or commercial grade
Analytical grade
Ultrapure grade
USP and NF chemical grade

A

USP and NF chemical grade

How well did you know this?
1
Not at all
2
3
4
5
Perfectly
37
Q

Basic unit for mass:

A. Gram
B. Kilogram
C. Mole
D. Pound

A

B. Kilogram

BASE QUANTITY
1. Length (meter)
2. Mass (kilogram)
3. Time (second)
4. Electric current (ampere)
5. Thermodynamic temperature (Kelvin)
6. Amount of substance (mole)
7. Luminous intensity (Candela)

How well did you know this?
1
Not at all
2
3
4
5
Perfectly
38
Q

Which of the following is NOT A COLLIGATIVE PROPERTY of solutions?

A. pH
B. Freezing point
C. Osmotic pressure
D. Vapor pressure

A

A. pH

The properties of osmotic pressure, vapor pressure, freezing point, and boiling point are
called COLLIGATIVE PROPERTIES.
When a solute is dissolved in a solvent, these colligative properties change in a predictable manner for each osmole of substance present:
- FREEZING POINT IS LOWERED by −1.86°C
- VAPOR PRESSURE IS LOWERED by 0.3 mm Hg or torr
- OSMOTIC PRESSURE IS INCREASED by a factor of 1.7 × 104 mm Hg or torr
- BOILING POINT IS RAISED by 0.52°C

How well did you know this?
1
Not at all
2
3
4
5
Perfectly
39
Q

Most clinical microbiology laboratories are categorized at what biosafety
level?

A. 1
B. 2
C. 3
D. 4

A

B. 2

How well did you know this?
1
Not at all
2
3
4
5
Perfectly
40
Q

Degree of hazard #2:

A. Slight
B. Moderate
C. Serious
D. Extreme

A

B. Moderate

DEGREE OF HAZARD
0: NO OR MINIMAL
1: SLIGHT
2: MODERATE
3: SERIOUS
4: EXTREME

How well did you know this?
1
Not at all
2
3
4
5
Perfectly
41
Q

Electrical equipment fire:

A. Class A
B. Class B
C. Class C
D. Class D

A

C. Class C

Fires have been divided into four classes based on the nature of the combustible material
and requirements for extinguishment:
Class A: ordinary combustible solid materials, such as paper, wood, plastic, and fabric
Class B: flammable liquids/gases and combustible petroleum products
Class C: energized electrical equipment
Class D: combustible/reactive metals, such as magnesium, sodium, and potassium

How well did you know this?
1
Not at all
2
3
4
5
Perfectly
42
Q

Type of extinguisher for CLASS A FIRES:
1. Pressurized water
2. Dry chemical
3. Carbon dioxide
4. Halon

A. 1 and 2
B. 1 and 3
C. 1, 2 and 3
D. Only 1

A

A. 1 and 2

TYPE OF EXTINGUISHER (Bishop page 47)
Class A: Pressurized water and dry chemical
Class B: Dry chemical and carbon dioxide
Class C: Dry chemical, carbon dioxide and halon
Class D: Metal X

How well did you know this?
1
Not at all
2
3
4
5
Perfectly
43
Q

All of the following are CRYOGENIC MATERIALS HAZARDS, EXCEPT:

A. Asphyxiation
B. Fire or explosion
C. Shock
D. Tissue damage similar to thermal burns

A

C. Shock

Liquid nitrogen is probably one of the most widely used cryogenic fluids (liquefi ed gases) in the laboratory.
There are, however, several hazards associated with the use of any cryogenic material: fire or explosion, asphyxiation, pressure buildup, embrittlement of materials, and tissue damage similar to that of thermal burns.

How well did you know this?
1
Not at all
2
3
4
5
Perfectly
44
Q

Repetitive strain disorders such as tenosynovitis, bursitis, and ganglion cysts:

A. Cryogenic materials hazards
B. Electrical hazards
C. Ergonomic hazards
D. Mechanical hazards

A

C. Ergonomic hazards

ERGONOMIC HAZARDS
The primary contributing factors associated with repetitive strain disorders are position/posture, applied force, and frequency of repetition.
Remember to consider the design of hand tools (e.g., ergonomic pipets), adherence to ergonomically correct technique, and equipment positioning when engaging in any repetitive task. Chronic symptoms of pain, numbness, or tingling in extremities may indicate the onset of repetitive strain disorders. Other hazards include acute musculoskeletal injury. Remember to lift heavy objects properly, keeping the load close to the body and using the muscles of the legs rather than the back. Gradually increase force when pushing or pulling, and avoid pounding actions with the extremities.

How well did you know this?
1
Not at all
2
3
4
5
Perfectly
45
Q

The first step to take when attempting to repair electronic equipment is to:

A. Check all electronic connections
B. Turn instrument off and unplug it
C. Reset all the printed circuit boards
D. Review instrument manual

A

B. Turn instrument off and unplug it

Before REPAIR OR ADJUSTMENT of electrical equipment
The following should be done
(1) unplug the equipment
(2) make sure the hands are dry
(3) remove jewelry.

How well did you know this?
1
Not at all
2
3
4
5
Perfectly
46
Q

When a person is receiving an electrical shock, all of the following should be done EXCEPT:

A. Pull the person away from the electrical source
B. Turn off the circuit breaker
C. Move the electrical source using a glass object
D. Move the electrical source using a wood object

A

A. Pull the person away from the electrical source

When an accident involving electrical shock occurs:
The electrical source must be removed immediately. TURNING OFF THE CIRCUIT
BREAKER, UNPLUGGING THE EQUIPMENT, or moving the equipment using a nonconductive glass or wood object are safe procedures to follow

How well did you know this?
1
Not at all
2
3
4
5
Perfectly
47
Q

Most common source of light for work in the visible and near-infrared regions:

A. Deuterium discharge lamp and mercury arc lamp
B. Incandescent tungsten or tungsten-iodide lamp

A

B. Incandescent tungsten or tungsten-iodide lamp

The most common source of light for work in the visible and near-infrared regions is the incandescent tungsten or tungsten-iodide lamp.

How well did you know this?
1
Not at all
2
3
4
5
Perfectly
48
Q

The lamps most commonly used for ultraviolet (UV) work are:

A. Deuterium discharge lamp and mercury arc lamp
B. Incandescent tungsten or tungsten-iodide lamp

A

A. Deuterium discharge lamp and mercury arc lamp

The lamps most commonly used for ultraviolet (UV) work are the deuterium discharge lamp and the mercury arc lamp.

How well did you know this?
1
Not at all
2
3
4
5
Perfectly
49
Q

Which is the most sensitive detector for spectrophotometry?

A. Photomultiplier
B. Phototube
C. Electron multiplier
D. Photodiode array

A

A. Photomultiplier

Because of this amplification, the PM tube is 200 times more sensitive than the phototube.
PM tubes are used in instruments designed to be extremely sensitive to very low light levels and light flashes of very short duration.

How well did you know this?
1
Not at all
2
3
4
5
Perfectly
50
Q

Reflectance spectrometry uses which of the following?

A. Luminometer
B. Tungsten–halogen lamp
C. Photomultiplier tube
D. UV lamp
E. hermometer to monitor temperature in reaction

A

B. Tungsten–halogen lamp

Slide technology depends on refl ectance spectrophotometry.
For colorimetric determinations, the light source is a tungsten–halogen lamp.

How well did you know this?
1
Not at all
2
3
4
5
Perfectly
51
Q

Which of the following light sources is used in atomic absorption spectrophotometry?

A. Hollow-cathode lamp
B. Xenon arc lamp
C. Tungsten light
D. Deuterium lamp
E. Laser

A

A. Hollow-cathode lamp

The usual light source, known as a hollow-cathode lamp, consists of an evacuated gastight chamber containing an anode, a cylindrical cathode, and an inert gas, such as helium or argon.

How well did you know this?
1
Not at all
2
3
4
5
Perfectly
52
Q

Used to measure concentration of LARGE PARTICLES:
1. Nephelometry
2. Turbidimetry
3. Absorption spectroscopy

A. 1 only
B. 2 only
C. 1 and 2
D. 1, 2 and 3

A

C. 1 and 2

Nephelometry and turbidimetry are used to measure the concentrations of large particles (such as antigen–antibody complexes, prealbumin, and other serum proteins) that because of their size cannot be measured by absorption spectroscopy.

How well did you know this?
1
Not at all
2
3
4
5
Perfectly
53
Q

Temperature is _ proportional to fluorescence.

A. Directly proportional
B. Inversely proportional
C. No effect

A

B. Inversely proportional

How well did you know this?
1
Not at all
2
3
4
5
Perfectly
54
Q

Low temperature:

A. Increase in fluorescence
B. Decrease in fluorescence

A

A. Increase in fluorescence

How well did you know this?
1
Not at all
2
3
4
5
Perfectly
55
Q

Which of the following techniques measures light scattered and has a light source placed at 90 degrees from the incident light?

A. Chemiluminescence
B. Atomic absorption spectrophotometry
C. Nephelometry
D. Turbidimetry

A

C. Nephelometry

How well did you know this?
1
Not at all
2
3
4
5
Perfectly
56
Q

Which of the following instruments is used in the clinical laboratories to detect beta and gamma emissions?

A. Fluorometer
B. Nephelometer
C. Scintillation counter
D. Spectrophotometer

A

C. Scintillation counter

How well did you know this?
1
Not at all
2
3
4
5
Perfectly
57
Q

Liquids (reagents, diluents, and samples) are pumped through a system of [continuous] tubing:

A. Continuous flow analysis
B. Centrifugal analysis
C. Discrete analysis
D. None of these

A

A. Continuous flow analysis

How well did you know this?
1
Not at all
2
3
4
5
Perfectly
58
Q

Which of the following types of analyzers offers RANDOM-ACCESS CAPABILITIES?

A. Discrete analyzers
B. Continuous-flow analyzers
C. Centrifugal analyzers
D. None of these

A

A. Discrete analyzers

All three can use batch analysis (i.e., large number of specimens in one run), but only discrete analyzers offer random-access, or stat, capabilities.

How well did you know this?
1
Not at all
2
3
4
5
Perfectly
59
Q

Checking instrument calibration, temperature accuracy, and electronic parameters are part of:

A. Preventive maintenace
B. Quality control
C. Function verification
D. Precision verification

A

C. Function verification

Function verification includes monitoring temperature, setting electronic parameters, calibrating instruments and analyzing control data.

How well did you know this?
1
Not at all
2
3
4
5
Perfectly
60
Q

A pre-analytical error can be introduced by:

A. Drawing a coagulation tube before an EDTA tube
B. Mixing an EDTA tube 8 to 10 times
C. Transporting the specimen in a biohazard bag
D. Vigorously shaking the blood tube to prevent clotting

A

D. Vigorously shaking the blood tube to prevent clotting

Vigorously mixing can cause hemolysis.

How well did you know this?
1
Not at all
2
3
4
5
Perfectly
61
Q

In quality control,± 2 standard deviations from the mean includes what percentage of the sample population?

A. 50%
B. 75%
C. 95%
D. 98%

A

C. 95%

The probability of an observation having a value within ± 2 standard deviations of the mean in a normal distribution is 95%.

How well did you know this?
1
Not at all
2
3
4
5
Perfectly
62
Q

The acceptable limit of error in the chemistry laboratory is 2 standard deviations. If you run the normal control of 100 times, how many of the values would be out of the control range due to random error?

A. 1
B. 5
C. 10
D. 20

A

B. 5

The probability of an observation having a value of 2 SD from the mean in a normal distribution is 95.5%. Therefore, 5 control values out of 100 would be out of control due to random error.

How well did you know this?
1
Not at all
2
3
4
5
Perfectly
63
Q

A delta check is a method that:

A. Determines the mean and variance of an instrument
B. Monitors the testing system for precision
C. Monitors patient sample day by day
D. Is determined by each laboratory facility

A

C. Monitors patient sample day by day

The delta check method compares current results from automated analyzers with the result from the most recent, previous values for the same patient.

How well did you know this?
1
Not at all
2
3
4
5
Perfectly
63
Q

The following data were calculated on a series of 30 determinations of serum uric acid control: mean = 5.8 mg/dL, 1 standard deviation = 0.15 mg/dL. If confidence limits are set at± 2 SD, which o f the following represents allowable limits for the control?

A. 5.65 to 5.95 mg/dL
B. 5.35 to 6.25 mg/dL
C. 5.50 to 6.10 mg/dL
D. 5.70 to 5.90 mg/dL

A

C. 5.50 to 6.10 mg/dL

Standard deviation is a measure of the dispersion of data around the mean.

How well did you know this?
1
Not at all
2
3
4
5
Perfectly
64
Q

PLEASE CHECK THREE (3) BOXES: Measures of center.

  • Coefficient of variation
  • Mean
  • Median
  • Mode
  • Range
  • Standard deviation
A

Mean
Median
Mode

The three most commonly used descriptions of the center of a dataset are the mean, the median, and the mode.

How well did you know this?
1
Not at all
2
3
4
5
Perfectly
65
Q

PLEASE CHECK THREE (3) BOXES: Measures of spread.

  • Coefficient of variation
  • Mean
  • Median
  • Mode
  • Range
  • Standard deviation
A

Coefficient of variation
Range
Standard deviation

The spread represents the relationship of all the data points to the mean. There are three commonly used descriptions of spread: (1) range (2) standard deviation (SD), and (3) coefficient of variation (CV).

How well did you know this?
1
Not at all
2
3
4
5
Perfectly
66
Q

Systematic errors include: PLEASE CHECK 3 BOXES.

  • Calibrator reconstitution
  • Electro-optical mechanism
  • Environmental conditions
  • Fluctuations in line voltage
  • Instability of instrument
  • Reagent dispensing
  • Reagent lot variability
  • Sample evaporation
  • Temperature of analyzer
  • Variation in handling techniques: pipetting, mixing, timing
  • Variation in operators
  • Wear and tear of instrument
A

Fluctuations in line voltage
Reagent lot variability
Wear and tear of instrument

Reference: Clinical Chemistry: A Laboratory Perspective [Arneson]

A SYSTEMATIC ERROR, on the other hand, will be seen as a trend in the data. Control values gradually rise (or fall) from the previously established limits. This type of error includes improper calibration, deterioration of reagents, sample instability, instrument drift, or changes in standard materials. All the Westgard rules that indicate trends identify systematic errors. 2(2S), 4(1S) and 10(x) rule.

SYSTEMATIC ERRORS MAY BE DUE TO:
* Aging reagents
* Aging calibrators
* Instrument components
* Optical changes
* Fluctuations in line voltage
* Wear and tear of instrument
* Reagent lot variability
* Calibration differences
* Technologist interactions

How well did you know this?
1
Not at all
2
3
4
5
Perfectly
67
Q

Random errors include: PLEASE CHECK 3 BOXES.

  • Aging reagents
  • Aging calibrators
  • Calibration differences
  • Instrument components
  • Fluctuations in line voltage
  • Optical changes
  • Reagent lot variability
  • Reagent dispensing
  • Technologist interactions
  • Variation in handling techniques: pipetting, mixing, timing
  • Variation in operator
  • Wear and tear of instrument
A

Reagent dispensing
Variation in handling techniques: pipetting, mixing, timing
Variation in operator

Reference: Clinical Chemistry: A Laboratory Perspective [Arneson]

RANDOM ERROR is one with no trend or means of predicting it. Random errors include such situations as mislabeling a sample, pipetting errors, improper mixing of sample and reagent, voltage fluctuations not compensated for by instrument circuitry, and temperature fluctuations. Violations of the 1(2S), 1(3S) and R(4S) Westgard rules are usually associated with random error. To assess the situation, the sample is assayed using the same reagents. If a random error occurred, the same mistake may not be made again, and the result will be within appropriate control limits.

RANDOM ERRORS MAY BE DUE TO:
* Reagent dispensing
* Sample evaporation
* Temperature of analyzer
* Electro-optical mechanism
* Calibrator reconstitution
* Environmental conditions
* Instability of instrument
* Variation in handling techniques: pipetting, mixing, timing
* Variation in operators

How well did you know this?
1
Not at all
2
3
4
5
Perfectly
68
Q

Most frequently occurring value in a dataset:

A. Mean
B. Median
C. Mode
D. Range

A

C. Mode

The mode is the most frequently occurring value in a dataset. Although it is seldom used to describe data, it is referred to when in reference to the shape of data, a bimodal distribution, for example.

How well did you know this?
1
Not at all
2
3
4
5
Perfectly
69
Q

Type of systemic error in the sample direction and magnitude; the magnitude of change is constant and not dependent on the amount of analyte.

A. Constant systematic error
B. Proportional systematic error

A

A. Constant systematic error

Constant error: Type of systemic error in the sample direction and magnitude; the magnitude of change is constant and not dependent on the amount of analyte.

Proportional error: Type of systemic error where the magnitude changes as a percent of the analyte present; error dependent on analyte concentration.

How well did you know this?
1
Not at all
2
3
4
5
Perfectly
70
Q

Type of systemic error where the magnitude changes as a percent of the analyte present; error dependent on analyte concentration.

A. Constant systematic error
B. Proportional systematic error

A

B. Proportional systematic error

Constant error: Type of systemic error in the sample direction and magnitude; the magnitude of change is constant and not dependent on the amount of analyte.

Proportional error: Type of systemic error where the magnitude changes as a percent of the analyte present; error dependent on analyte concentration.

How well did you know this?
1
Not at all
2
3
4
5
Perfectly
71
Q

Difference between the observed mean and the reference mean:

A. Bias
B. Confidence interval
C. Parametric method
D. Nonparametric method

A

A. Bias

Bias: Difference between the observed mean and the reference mean.

Negative bias indicates that the test values tend to be lower than the reference value, whereas positive bias indicates test values are generally higher.

Bias is a type of constant systematic error.

How well did you know this?
1
Not at all
2
3
4
5
Perfectly
72
Q

Ability of a test to detect a given disease or condition.

A. Analytic sensitivity
B. Analytic specificity
C. Diagnostic sensitivity
D. Diagnostic specificity

A

C. Diagnostic sensitivity

Analytic sensitivity: Ability of a method to detect small quantities of an analyte.
Analytic specificity: Ability of a method to detect only the analyte it is designed to determine.
Diagnostic sensitivity: Ability of a test to detect a given disease or condition.
Diagnostic specificity: Ability of a test to correctly identify the absence of a given disease or condition.

How well did you know this?
1
Not at all
2
3
4
5
Perfectly
73
Q

Ability of a test to correctly identify the absence of a given disease or condition.

A. Analytic sensitivity
B. Analytic specificity
C. Diagnostic sensitivity
D. Diagnostic specificity

A

D. Diagnostic specificity

Analytic sensitivity: Ability of a method to detect small quantities of an analyte.
Analytic specificity: Ability of a method to detect only the analyte it is designed to determine.
Diagnostic sensitivity: Ability of a test to detect a given disease or condition.
Diagnostic specificity: Ability of a test to correctly identify the absence of a given disease or condition.

How well did you know this?
1
Not at all
2
3
4
5
Perfectly
74
Q

Ability of a method to detect small quantities of an analyte

A. Analytic sensitivity
B. Analytic specificity
C. Diagnostic sensitivity
D. Diagnostic specificity

A

A. Analytic sensitivity

Analytic sensitivity: Ability of a method to detect small quantities of an analyte.
Analytic specificity: Ability of a method to detect only the analyte it is designed to determine.
Diagnostic sensitivity: Ability of a test to detect a given disease or condition.
Diagnostic specificity: Ability of a test to correctly identify the absence of a given disease or condition.

How well did you know this?
1
Not at all
2
3
4
5
Perfectly
75
Q

Ability of a method to detect only the analyte it is designed to determine

A. Analytic sensitivity
B. Analytic specificity
C. Diagnostic sensitivity
D. Diagnostic specificity

A

B. Analytic specificity

Analytic sensitivity: Ability of a method to detect small quantities of an analyte.
Analytic specificity: Ability of a method to detect only the analyte it is designed to determine.
Diagnostic sensitivity: Ability of a test to detect a given disease or condition.
Diagnostic specificity: Ability of a test to correctly identify the absence of a given disease or condition.

How well did you know this?
1
Not at all
2
3
4
5
Perfectly
76
Q

Positive predictive value:

A. Ability of a test to detect a given disease or condition.
B. Ability of a test to correctly identify the absence of a given disease or condition.
C. Chance of an individual having a given disease or condition if the test is abnormal.
D. Chance an individual does not have a given disease or condition if the test is within the reference interval.

A

C. Chance of an individual having a given disease or condition if the test is abnormal.

Positive predictive value: Chance of an individual having a given disease or condition if the test is abnormal.
Negative predictive value: Chance an individual does not have a given disease or condition if the test is within the reference interval.

How well did you know this?
1
Not at all
2
3
4
5
Perfectly
76
Q

Negative predictive value:

A. Ability of a test to detect a given disease or condition.
B. Ability of a test to correctly identify the absence of a given disease or condition.
C. Chance of an individual having a given disease or condition if the test is abnormal.
D. Chance an individual does not have a given disease or condition if the test is within the reference interval.

A

D. Chance an individual does not have a given disease or condition if the test is within the reference interval.

Positive predictive value: Chance of an individual having a given disease or condition if the test is abnormal.
Negative predictive value: Chance an individual does not have a given disease or condition if the test is within the reference interval.

How well did you know this?
1
Not at all
2
3
4
5
Perfectly
77
Q

What percentage of values will fall between ±2 s in a Gaussian (normal) distribution?

A. 34.13%
B. 68.26%
C. 95.45%
D. 99.74%

A

C. 95.45%

68.26% will lie within ±1 s
95.45% will lie within ±2 s
99.74% will lie within ±3 s

How well did you know this?
1
Not at all
2
3
4
5
Perfectly
78
Q

Two (2) consecutive control values exceed the same 2 standard deviation limit:

A. 1:2S
B. 2:2S
C. R:4S
D. 4:1S

A

B. 2:2S

Westgard multirule is a control procedure that utilizes control rules to assess numerical quality control data; the control rules establish the limits for data rejection in a system with two controls. Other rules apply when three controls are used.
1:2s = 1 control value exceeds the mean ±2 standard deviations; warning rule that triggers inspection of control values using the other rejection rules that follow; only rule that is not used to reject a run; results are reportable 1:3s = 1 control value exceeds the mean ±3 standard deviations; detects random error 2:2s = 2 consecutive control values exceed the same 2 standard deviation limit (same mean +2 s or same mean -2 s); detects systematic error R:4s = 1 control value in a group exceeds the mean +2 s and a second control value exceeds the mean -2 s, creating a 4 standard deviation spread; detects random error 4:ls = 4 consecutive control values are recorded on one side of the mean and exceed either the same mean +1 s or the same mean -1 s; detects systematic error 10:x =10 consecutive control values are recorded on one side of the mean (either above or below the mean); detects systematic error

How well did you know this?
1
Not at all
2
3
4
5
Perfectly
79
Q

The term R:4S means that:

A. Four consecutive controls are greater than ±1 standard deviation from the mean
B. Two controls in the same run are greater than 4s units apart
C. Two consecutive controls in the same run are each greater than ±4s from the mean
D. There is a shift above the mean for four consecutive controls

A

B. Two controls in the same run are greater than 4s units apart

The R:4s rule is applied to two control levels within the same run. The rule is violated when the algebraic difference between them (level 1 – level 2) exceeds 4s.
The R:4s rule detects random error (error due to poor precision).

How well did you know this?
1
Not at all
2
3
4
5
Perfectly
80
Q

Error always in one direction:

A. Random error
B. Systematic error

A

B. Systematic error

Systemic error: Error always in one direction.
Random error: Error varies from sample to sample.

How well did you know this?
1
Not at all
2
3
4
5
Perfectly
81
Q

Which of the following terms refers to deviation from the true value caused by indeterminate errors inherent in every laboratory measurement?

A. Random error
B. Standard error of the mean
C. Parametric analysis
D. Nonparametric analysis

A

A. Random error

Random errors are deviations from the true value caused by unavoidable errors inherent in laboratory measurements.
The standard error of the mean is a statistical concept refl ecting sampling variation. It is the standard deviation of the entire population.
Parametric statistics refer to a Gaussian (normal) distribution of data.
Nonparametric statistics are more general and require no assumptions.

How well did you know this?
1
Not at all
2
3
4
5
Perfectly
82
Q

A trend in QC results is most likely caused by:

A. Deterioration of the reagent
B. Miscalibration of the instrument
C. Improper dilution of standards
D. Electronic noise

A

A. Deterioration of the reagent

How well did you know this?
1
Not at all
2
3
4
5
Perfectly
83
Q

Which of the following plots is best for detecting all types of QC errors?

A. Levy–Jennings
B. Tonks–Youden
C. Cusum
D. Linear regression

A

A. Levy–Jennings

The Levy–Jennings plot is a graph of all QC results with concentration plotted on the y axis and run number on the x axis.

How well did you know this?
1
Not at all
2
3
4
5
Perfectly
84
Q

Which of the following plots is best for comparison of precision and accuracy among laboratories?

A. Levy–Jennings
B. Tonks–Youden
C. Cusum
D. Linear regression

A

B. Tonks–Youden

The Tonks–Youden plot is used for interlaboratory comparison of monthly means. The method mean for level 1 is at the center of the y axis and mean for level 2 at the center of the x axis. Lines are drawn from the means of both levels across the graph, dividing it into four equal quadrants. If a laboratory’s monthly means both plot in the lower left or upper right, then systematic error (SE) exists in its method.

How well did you know this?
1
Not at all
2
3
4
5
Perfectly
85
Q

Which plot will give the earliest indication of a shift or trend?

A. Levy–Jennings
B. Tonks–Youden
C. Cusum
D. Histogram

A

C. Cusum

Cusum points are the algebraic sum of the difference between each QC result and the mean. The y axis is the sum of differences and the x axis is the run number. The center of the y axis is 0. Because QC results follow a random distribution, the points should distribute about the zero line. Results are out of control when the slope exceeds 45° or a decision limit (e.g., ±2.7s) is exceeded.

How well did you know this?
1
Not at all
2
3
4
5
Perfectly
86
Q

Which of the following terms refers to the closeness with which the measured value agrees with the true value?

A. Random error
B. Precision
C. Accuracy
D. Variance

A

C. Accuracy

The accuracy of an analytical result is the closeness with which the measured value agrees with the true value.
Precision is reproducibility.
Accuracy and precision are independent, but it is the goal of the clinical laboratory to design methods that are both precise and accurate.

How well did you know this?
1
Not at all
2
3
4
5
Perfectly
87
Q

Relatively easy to measure and maintain:

A. Accuracy
B. Precision
C. Sensitivity
D. Specificity

A

B. Precision

Accuracy is easy to define but difficult to establish and maintain.
Precision is relatively easy to measure and maintain.

How well did you know this?
1
Not at all
2
3
4
5
Perfectly
88
Q

Which of the following describes the ability of an analytical method to maintain both accuracy and precision over an extended period of time?

A. Reliability
B. Validity
C. Probability
D. Sensitivity

A

A. Reliability

The reliability of an analytical procedure is its ability to maintain accuracy and precision over an extended period of time during which supplies, equipment, and personnel in the laboratory may change. It is often used interchangeably with the term “consistency.” It is the goal of every clinical laboratory to produce reliable results.

How well did you know this?
1
Not at all
2
3
4
5
Perfectly
89
Q

Which of the following statistical tests is used to compare the means of two methods?

A. Student’s t test
B. F distribution
C. Correlation coefficient (r)
D. Linear regression analysis

A

A. Student’s t test

T TEST: COMPARES MEANS, ACCURACY
F TEST: COMPARES S.D., PRECISION
Student’s t test is the ratio of mean difference to the standard error of the mean difference
(bias/random error) and tests for a signifi cant difference in means.
The F test is the ratio of variances and determines if one method is significantly less precise.

How well did you know this?
1
Not at all
2
3
4
5
Perfectly
90
Q

Only carbohydrate directly used for energy:

A. Glucose
B. Maltose
C. Fructose
D. Lactose

A

A. Glucose

How well did you know this?
1
Not at all
2
3
4
5
Perfectly
91
Q

In a person with normal glucose metabolism, the blood glucose level usually increases rapidly after carbohydrates are ingested but returns to a normal level after:

A. 30 minutes
B. 45 minutes
C. 60 minutes (1 hour)
D. 120 minutes (2 hours)

A

D. 120 minutes (2 hours)

How well did you know this?
1
Not at all
2
3
4
5
Perfectly
92
Q

Glucose measurements can be ____ mg/dL erroneously higher by reducing methods than by more accurate enzymatic methods.

A. 1 to 5 mg/dL
B. 5 to 15 mg/dL
C. 20 to 25 mg/dL
D. 30 to 35 mg/dL

A

B. 5 to 15 mg/dL

Glucose measurements can be 5 to 15 mg/dL erroneously higher by reducing methods than by more accurate enzymatic methods.

How well did you know this?
1
Not at all
2
3
4
5
Perfectly
93
Q

Select the enzyme that is most specific for beta D-glucose:

A. Glucose oxidase
B. Glucose-6-phosphate dehydrogenase
C. Hexokinase
D. Phosphohexose isomerase

A

A. Glucose oxidase

Glucose oxidase is the most specifi c enzyme reacting with only beta D-glucose. Glucose oxidase converts beta D-glucose to gluconic acid.
Mutarotase may be added to the reaction to facilitate the conversion of alpha-D-glucose to beta-D-glucose.

How well did you know this?
1
Not at all
2
3
4
5
Perfectly
94
Q

Select the coupling enzyme used in the hexokinase method for glucose:

A. Glucose dehydrogenase
B. Glucose-6-phosphatase
C. Glucose-6-phosphate dehydrogenase
D. Peroxidase

A

C. Glucose-6-phosphate dehydrogenase

The hexokinase method is considered more accurate than the glucose oxidase methods because the coupling reaction using glucose-6-phosphate dehydrogenase is highly specific; therefore, it has less interference than the coupled glucose oxidase procedure.

How well did you know this?
1
Not at all
2
3
4
5
Perfectly
95
Q

Which of the following is a potential source of error in the HEXOKINASE METHOD?

A. Galactosemia
B. Hemolysis
C. Sample collected in fluoride
D. Ascorbic acid

A

B. Hemolysis

Hemolyzed samples require a serum blank correction (subtraction of the reaction rate with hexokinase omitted from the reagent).

How well did you know this?
1
Not at all
2
3
4
5
Perfectly
96
Q

Gross hemolysis and extremely elevated bilirubin may cause in HEXOKINASE RESULTS.

A. False increase
B. False decrease
C. No effect
D. Variable

A

B. False decrease

Generally accepted as the reference method, hexokinase method is not affected by ascorbic acid or uric acid. Gross hemolysis and extremely elevated bilirubin may cause a false decrease in results.

How well did you know this?
1
Not at all
2
3
4
5
Perfectly
97
Q

VERY LOW OR UNDETECTABLE C-peptide:

A. Type 1 diabetes mellitus
B. Type 2 diabetes mellitus

A

A. Type 1 diabetes mellitus

How well did you know this?
1
Not at all
2
3
4
5
Perfectly
98
Q

DETECTABLE C-peptide:

A. Type 1 diabetes mellitus
B. Type 2 diabetes mellitus

A

B. Type 2 diabetes mellitus

How well did you know this?
1
Not at all
2
3
4
5
Perfectly
99
Q

A 62-year-old patient presents to the physician with report of increased thirst and increased urination, particularly at night. The physician requests a series of tests over the next few days. The following data are received: RANDOM GLUCOSE: 186 mg/dL; FASTING GLUCOSE: 114 mg/dL; 2-HOUR OGTT: 153 mg/dL HbA1c: 5.9%. Which of the following conclusions may be made regarding these data?

A. Data represents normal glucose status
B. Data represents an impaired glucose status
C. Data represents the presence of insulinoma
D. Data represents diagnosis of diabetes

A

B. Data represents an impaired glucose status

Resting plasma glucose
NORMAL <140
IMPAIRED 140 – 199
DIAGNOSTIC ≥ 200 mg/dL

Fasting plasma glucose
NORMAL <100
IMPAIRED 100 – 125
DIAGNOSTIC ≥ 126 mg/dL

2-hour OGTT
NORMAL <140
IMPAIRED 140 – 199
DIAGNOSTIC ≥ 200 mg/dL

HbA1c
NORMAL <5.7 %
IMPAIRED 5.7 – 6.4%
DIAGNOSTIC ≥ 6.5%

How well did you know this?
1
Not at all
2
3
4
5
Perfectly
100
Q

What is the recommended cutoff value for adequate control of blood glucose in diabetics as measured by glycated hemoglobin?

A. 5%
B. 6.5%
C. 9.5%
D. 11%

A

B. 6.5%

If the result is 6.5% or more, the treatment plan should be adjusted to achieve a lower
level, and the test performed every 3 months until control is improved.

How well did you know this?
1
Not at all
2
3
4
5
Perfectly
101
Q

A factor, other than average plasma glucose values, that can affect the HbA1c level is:

A. Serum ketone bodies
B. Red blood cell life span
C. Ascorbic acid intake
D. Increased triglyceride levels

A

B. Red blood cell life span

Two factors determine the glycosylated hemoglobin levels: the average glucose concentration and the red blood cell life span.
If the red blood cell life span is decreased because of another disease state such as hemoglobinopathies, the hemoglobin will have less time to become glycosylated and the glycosylated hemoglobin level will be lower.

How well did you know this?
1
Not at all
2
3
4
5
Perfectly
102
Q

LONG-TERM estimation of glucose concentration can be followed by measuring:

A. Glycosylated hemoglobin (HbA1c)
B. Fructosamine

A

A. Glycosylated hemoglobin (HbA1c)

Long-term estimation of glucose concentration can be followed by measuring glycosylated hemoglobin (Hb A1c). A level of 8% or less is considered “good” glycemic control.
Because the average red blood cell lives approximately 120 days, the glycosylated hemoglobin level at any one time reflects the average blood glucose level over the previous 2 to 3 months. Therefore, measuring the glycosylated hemoglobin provides the clinician with a time-averaged picture of the patient’s blood glucose concentration over the
past 3 months.

How well did you know this?
1
Not at all
2
3
4
5
Perfectly
103
Q

Most widely used to assess SHORT-TERM (3 to 6 weeks) glycemic control:

A. Glycosylated hemoglobin (HbA1c)
B. Fructosamine

A

B. Fructosamine

FRUCTOSAMINE: GLYCOSYLATED ALBUMIN
Fructosamine assays are the most widely used to assess short-term (3 to 6 weeks)
glycemic control because the average half-life of the proteins is 2–3 weeks.

How well did you know this?
1
Not at all
2
3
4
5
Perfectly
104
Q

Formation of glucose-6-phosphate from noncarbohydrate sources:

A. Glycolysis
B. Gluconeogenesis
C. Glycogenolysis
D. Glycogenesis

A

B. Gluconeogenesis

Glycolysis
Metabolism of glucose molecule to pyruvate or lactate for production of energy

Gluconeogenesis
Formation of glucose-6-phosphate from noncarbohydrate sources

Glycogenolysis
Breakdown of glycogen to glucose for use as energy

Glycogenesis
Conversion of glucose to glycogen for storage

Lipogenesis
Conversion of carbohydrates to fatty acids

Lipolysis
Decomposition of fat

How well did you know this?
1
Not at all
2
3
4
5
Perfectly
105
Q

HYPERGLYCEMIC FACTOR produced by the pancreas is:

A. Epinephrine
B. Glucagon
C. Growth hormone
D. Insulin

A

B. Glucagon

Glucagon is the primary hormone responsible for increasing glucose levels. It is synthesized by the ALPHA-cells of islets of Langerhans in the pancreas and released during stress and fasting states.

ACTION OF GLUCAGON
Increases glycogenolysis: glycogen → glucose
Increases gluconeogenesis: fatty acids → acetyl-CoA → ketone, proteins → amino acids

How well did you know this?
1
Not at all
2
3
4
5
Perfectly
106
Q

HYPOGLYCEMIC FACTOR produced by the pancreas is:

A. Epinephrine
B. Glucagon
C. Growth hormone
D. Insulin

A

D. Insulin

Insulin is the only hormone that decreases glucose levels and can be referred to as a hypoglycemic agent.
Insulin is the primary hormone responsible for the entry of glucose into the cell. It is synthesized by the beta cells of islets of Langerhans in the pancreas.
It decreases plasma glucose levels by increasing the transport entry of glucose in muscle and adipose tissue by way of nonspecific receptors. It also regulates glucose by increasing glycogenesis, lipogenesis, and glycolysis and inhibiting glycogenolysis.

ACTION OF INSULIN
* Increases glycogenesis and glycolysis: glucose → glycogen → pyruvate → acetyl-CoA
* Increases lipogenesis
* Decreases glycogenolysis

How well did you know this?
1
Not at all
2
3
4
5
Perfectly
107
Q

What would an individual with CUSHING SYNDROME tend to exhibit?

A. Hyperglycemia
B. Hypoglycemia
C. Normal blood glucose level
D. Decreased 2-hour postprandial glucose

A

A. Hyperglycemia

CORTISOL INCREASES BLOOD GLUCOSE.

How well did you know this?
1
Not at all
2
3
4
5
Perfectly
108
Q

Symptoms of HYPOGLYCEMIA usually occur when blood glucose has fallen below _ mg/dL.

A. Below 50 mg/dL
B. Below 60 mg/dL
C. Below 70 mg/dL
D. Below 80 mg/dL

A

A. Below 50 mg/dL

How well did you know this?
1
Not at all
2
3
4
5
Perfectly
109
Q

Beta cell destruction, usually leading to absolute insulin deficiency:

A. Type 1 DM
B. Type 2 DM

A

A. Type 1 DM

How well did you know this?
1
Not at all
2
3
4
5
Perfectly
110
Q

May range from predominantly insulin resistance with relative insulin deficiency to a predominantly secretory defect with insulin resistance:

A. Type 1 DM
B. Type 2 DM

A

B. Type 2 DM

How well did you know this?
1
Not at all
2
3
4
5
Perfectly
111
Q

Usual dose of LACTOSE in the oral lactose tolerance test is:

A. 25 grams
B. 50 grams
C. 75 grams
D. 100 grams

A

B. 50 grams

ORAL LACTOSE TOLERANCE TESTS, measuring the increase in plasma glucose or galactose following ingestion of lactose, have been used to diagnose lactase defi ciency.

How well did you know this?
1
Not at all
2
3
4
5
Perfectly
112
Q

Which of the following apoproteins is inversely related to risk for coronary heart disease and is a surrogate marker for HDL?

A. Apo-A1
B. Apo-B
C. Apo-B100
D. Apo-E

A

A. Apo-A1

Apo-A1 is the predominant apoprotein associated with the high-density lipoprotein (HDL) molecule, activates (lecithin cholesterol acyltransferase [LCAT]), and is associated with reverse cholesterol transport. As a result, it is protective against coronary artery disease.

How well did you know this?
1
Not at all
2
3
4
5
Perfectly
113
Q

A patient sample is assayed for fasting triglycerides and a triglyceride value of 1036 mg/dL. This value is of immediate concern because of its association with which of the following conditions?

A. Coronary heart disease
B. Diabetes
C. Pancreatitis
D. Gout

A

C. Pancreatitis

Marked increases in triglyceride levels, between 1000 and 2000 mg/dL have been associated with increased risk for the development of pancreatitis.

How well did you know this?
1
Not at all
2
3
4
5
Perfectly
114
Q

Patients with Waldenström’s macroglobulinemia exhibit abnormally large amounts of:

A. IgM
B. IgG
C. IgE
D. IgA

A

A. IgM

Waldenström’s primary macroglobulinemia (WM), or simply macroglobulinemia, is a B cell disorder characterized by the infiltration of lymphoplasmacytic cells into bone marrow and the presence of an IgM monoclonal gammopathy.

How well did you know this?
1
Not at all
2
3
4
5
Perfectly
115
Q

Which test is the most sensitive in detecting early monoclonal gammopathies?

A. High-resolution serum protein electrophoresis
B. Urinary electrophoresis for monoclonal light chains
C. Capillary electrophoresis of serum and urine
D. Serum-free light chain immunoassay

A

D. Serum-free light chain immunoassay

Measurement of free light chains is recommended along with protein electrophoresis when testing for myeloma.
Free light chains are normally present in serum because L chains are made at a faster rate than H chains.

How well did you know this?
1
Not at all
2
3
4
5
Perfectly
116
Q

Which test is the most useful way to evaluate the response to treatment for multiple myeloma?

A. Measure of total immunoglobulin
B. Measurement of 24-hour urinary light chain concentration (Bence-Jones protein)
C. Capillary electrophoresis of M-protein recurrence
D. Measurement of serum-free light chains

A

D. Measurement of serum-free light chains

Unlike electrophoresis methods, serum free light chain assays are quantitative and an increase in free light chain production with an abnormal kappa:lambda ratio occurs earliest in recurrence of myeloma.

How well did you know this?
1
Not at all
2
3
4
5
Perfectly
117
Q

Which of the following is the most common application of IMMUNOELECTROPHORESIS (IEP)?

A. Identification of the absence of a normal serum protein
B. Structural abnormalities of proteins
C. Screening for circulating immune complexes
D. Diagnosis of monoclonal gammopathies

A

D. Diagnosis of monoclonal gammopathies

How well did you know this?
1
Not at all
2
3
4
5
Perfectly
118
Q

PRE-ECLAMPSIA, also referred to as TOXEMIA OF PREGNANCY is marked by specific symptoms including:

A. Water retention (with swelling particularly in the feet, legs, and hands)
B. High blood pressure
C. Protein in the urine
D. All of these

A

D. All of these

How well did you know this?
1
Not at all
2
3
4
5
Perfectly
119
Q

A sensitive, although not specific indicator of damage to the kidneys:

A. Urea
B. Creatinine
C. Proteinuria
D. Cystatin C

A

C. Proteinuria

How well did you know this?
1
Not at all
2
3
4
5
Perfectly
120
Q

At pH 8.6, proteins are ________ charged and migrate toward the _________.

A. Negatively, anode
B. Positively, cathode
C. Positively, anode
D. Negatively, cathode

A

A. Negatively, anode

How well did you know this?
1
Not at all
2
3
4
5
Perfectly
121
Q

For albumin assay, absorbance at 630 nm is less likely to be affected by bilirubin or hemoglobin in the sample. Which dye gives a much greater absorbance change at 630 nm than it would at 500 nm?

A. HABA (Hydroxyazobenzene-benzoic acid)
B. BCG (Bromcresol green)

A

B. BCG (Bromcresol green)

How well did you know this?
1
Not at all
2
3
4
5
Perfectly
122
Q

Which of the following dyes is the MOST SPECIFIC for measurement of albumin?

A. Bromcresol green (BCG)
B. Bromcresol purple (BCP)
C. Tetrabromosulfophthalein
D. Tetrabromphenol blue

A

B. Bromcresol purple (BCP)

BCP is more specific for albumin than BCG.

How well did you know this?
1
Not at all
2
3
4
5
Perfectly
123
Q

In what condition would an increased level of serum albumin be expected?

A. Malnutrition
B. Acute inflammation
C. Dehydration
D. Renal disease

A

C. Dehydration

How well did you know this?
1
Not at all
2
3
4
5
Perfectly
124
Q

ARTIFACTUAL INCREASE in albumin concentration:

A. Prolonged tourniquet application
B. Dehydration
C. Nephrotic syndrome
D. Inflammation

A

A. Prolonged tourniquet application

How well did you know this?
1
Not at all
2
3
4
5
Perfectly
125
Q

Identification of which of the following is useful in early stages of glomerular dysfunction?

A. Microalbuminuria
B. Ketonuria
C. Hematuria
D. Urinary light chains

A

A. Microalbuminuria

How well did you know this?
1
Not at all
2
3
4
5
Perfectly
126
Q

Most abundant amino acid in the body:

A. Glutamine
B. Lysine
C. Phenylalanine
D. Tyrosine

A

A. Glutamine

Glutamine is the most abundant amino acid in the body, being involved in more metabolic processes than any other amino acid.

How well did you know this?
1
Not at all
2
3
4
5
Perfectly
127
Q

Precursor of the adrenal hormones epinephrine, norepinephrine, and dopamine and the thyroid hormones, including thyroxine:

A. Glutamine
B. Lysine
C. Phenylalanine
D. Tyrosine

A

D. Tyrosine

Tyrosine is a precursor of the adrenal hormones epinephrine, norepinephrine, and dopamine and the thyroid hormones, including thyroxine.
It is important in overall metabolism, aiding in the functions of the adrenal, thyroid, and pituitary glands.

How well did you know this?
1
Not at all
2
3
4
5
Perfectly
128
Q

The plasma protein mainly responsible for maintaining colloidal osmotic pressure in vivo is:

A. Albumin
B. Hemoglobin
C. Fibrinogen
D. Alpha2-macroglobulin

A

A. Albumin

Albumin is responsible for nearly 80% of the colloid osmotic pressure (COP) of the intravascular fluid, which maintains the appropriate fluid balance in the tissue.

How well did you know this?
1
Not at all
2
3
4
5
Perfectly
129
Q

Sensitive marker of poor nutritional status:

A. Prealbumin
B. Fibrinogen
C. Gc-globulin
D. Orosomucoid

A

A. Prealbumin

A low prealbumin level is a sensitive marker of poor nutritional status.

When a diet is deficient in protein, hepatic synthesis of proteins is reduced, with the resulting decrease in the level of the proteins originating in the liver, including prealbumin, albumin, and β-globulins. Because prealbumin has a short half-life of approximately 2 days, it decreases more rapidly than do other proteins.

How well did you know this?
1
Not at all
2
3
4
5
Perfectly
130
Q

Nutritional assessment with poor protein-caloric status is associated with:

A. A decreased level of prealbumin
B. A low level of γ-globulins
C. An elevated ceruloplasmin concentration
D. An increased level of α1-fetoprotein

A

A. A decreased level of prealbumin

How well did you know this?
1
Not at all
2
3
4
5
Perfectly
131
Q

Retinol (vitamin A) binding protein:

A. Albumin
B. Alpha1-antitrypsin
C. Fibronectin
D. Prealbumin

A

D. Prealbumin

Prealbumin is the transport protein for thyroxine and triiodothyronine (thyroid hormones); it also binds with retinol-binding protein to form a complex that transports retinol (vitamin A) and is rich in tryptophan.

How well did you know this?
1
Not at all
2
3
4
5
Perfectly
132
Q

Which of the following conditions is the result of a LOW ALPHA1- ANTITRYPSIN LEVEL?

A. Asthma
B. Emphysema
C. Pulmonary hypertension
D. Sarcoidosis

A

B. Emphysema

How well did you know this?
1
Not at all
2
3
4
5
Perfectly
133
Q

All are conditions associated with an elevated AFP, EXCEPT:

A. Neural tube defects
B. Spina bifida
C. Anencephaly
D. Down syndrome

A

D. Down syndrome

Conditions associated with an elevated AFP level include spina bifi da, neural tube defects, abdominal wall defects, anencephaly (absence of the major portion of the brain), and general fetal distress.
Low levels of maternal AFP indicate an increased risk for Down syndrome and trisomy 18, while it is increased in the presence of twins and neural tube defects.

How well did you know this?
1
Not at all
2
3
4
5
Perfectly
134
Q

In nephrotic syndrome, the levels of serum ____ may increase as much as 10 times because its large size aids in its retention.

A. Alpha2-macroglobulin
B. Ceruloplasmin
C. Orosomucoid
D. Transferrin

A

A. Alpha2-macroglobulin

In nephrosis, the levels of serum α2-macroglobulin may increase as much as 10 times because its large size aids in its retention. The protein is also increased in diabetes and liver disease.
Use of contraceptive medications and pregnancy increase the serum levels by 20%.

How well did you know this?
1
Not at all
2
3
4
5
Perfectly
135
Q

Orosomucoid:

A. Alpha1-antitrypsin
B. Alpha1-chymotrypsin
C. Alpha1-fetoprotein
D. Alpha1-acid glycoprotein

A

D. Alpha1-acid glycoprotein

α1-Acid Glycoprotein (Orosomucoid)
α1-Acid glycoprotein (AAG), a major plasma glycoprotein, is negatively charged even in acid solutions, a fact that gave it its name. This protein is produced by the liver and is an acute-phase reactant.

How well did you know this?
1
Not at all
2
3
4
5
Perfectly
136
Q

Which of the following is a low-weight protein that is found on the cell surfaces of nucleated cells?

A. Alpha2-macroglobulin
B. Beta2-microglobulin
C. C-reactive protein
D. Ceruloplasmin

A

B. Beta2-microglobulin

β2-microglobulin is a single polypeptide chain that is the light chain component of human leukocyte antigens (HLAs). It is found on the surface of nucleated cells and is notably present on lymphocytes. Increased plasma levels of β2-microglobulin are associated with renal failure, lymphocytosis, rheumatoid arthritis, and systemic lupus erythematosus.

How well did you know this?
1
Not at all
2
3
4
5
Perfectly
137
Q

Variants demonstrate a wide variety of cellular interactions, including roles in cell adhesion, tissue differentiation, growth, and wound healing:

A. Beta-trace protein
B. Cystatin C
C. Fibronectin
D. Troponin

A

C. Fibronectin

How well did you know this?
1
Not at all
2
3
4
5
Perfectly
138
Q

A glycoprotein used to help predict the short-term risk of PREMATURE DELIVERY:

A. Adiponectin
B. Alpha-fetoprotein
C. Amyloid
D. Fetal fibronectin

A

D. Fetal fibronectin

Fetal fibronectin (fFN) is a glycoprotein used to help predict the short-term risk of premature delivery.

How well did you know this?
1
Not at all
2
3
4
5
Perfectly
139
Q

BIOCHEMICAL MARKER OF BONE RESORPTION that can be detected in serum and urine:

A. Beta-trace protein
B. Crosslinked C-telopeptides (CTX)
C. Fibronectin
D. Troponin

A

B. Crosslinked C-telopeptides (CTX)

Cross-linked C-telopeptides (CTXs) are proteolytic fragments of collagen I formed during bone resorption (turnover).
CTX is a biochemical marker of bone resorption that can be detected in serum and urine.

How well did you know this?
1
Not at all
2
3
4
5
Perfectly
140
Q

An accurate marker of CSF leakage:

A. Beta-trace protein
B. Crosslinked C-telopeptides (CTX)
C. Fibronectin
D. Troponin

A

A. Beta-trace protein

β-Trace protein (BTP; synonym prostaglandin D synthase) is a 168–amino acid, low-molecular-mass protein in the lipocalin protein family. Recently, it was verifi ed that BTP was established as an accurate marker of CSF leakage.
It has also been reported recently as a potential marker in detecting impaired renal function, although no more sensitive than cystatin C.

How well did you know this?
1
Not at all
2
3
4
5
Perfectly
141
Q

Recently proposed new marker for the early assessment of changes to the glomerular filtration rate:

A. Adiponectin
B. Beta-trace protein
C. Cross-linked C-telopeptides (CTX)
D. Cystatin C

A

D. Cystatin C

How well did you know this?
1
Not at all
2
3
4
5
Perfectly
142
Q

Supplemental tests to help differentiate a diagnosis of ALZHEIMER DISEASE from other forms of dementia:

A. Amyloid β42 (Aβ42) and Tau protein
B. Crosslinked C-telopeptides (CTX)
C. Fibronectin
D. Troponin

A

A. Amyloid β42 (Aβ42) and Tau protein

In a symptomatic patient, low Aβ42 along with high Tau refl ects an increased likelihood of Alzheimer disease, but it does not mean that the person defi nitely has Alzheimer disease.
If a patient does not have abnormal levels of these proteins, then the dementia is more likely due to a cause other than Alzheimer disease.

How well did you know this?
1
Not at all
2
3
4
5
Perfectly
143
Q

As a cardiac biomarker, this protein has been used in conjunction with troponin to help diagnose or rule out a heart attack:

A. Brain natriuretic peptide (BNP)
B. Cross-linked C-telopeptides (CTX)
C. Cystatin C
D. Myoglobin

A

D. Myoglobin

As a cardiac biomarker, myoglobin has been used in conjunction with troponin to help diagnose or rule out a heart attack. When striated muscle is damaged, myoglobin is released, elevating the blood levels.

In an AMI, this increase is seen within 2 to 3 hours of onset and reaches peak concentration in 8 to 12 hours. Myoglobin is a small molecule freely filtered by the kidneys, allowing levels to return to normal in 18 to 30 hours after the AMI.

Because of the speed of appearance and clearance of myoglobin, it is also a useful marker for monitoring the success or failure of reperfusion.

How well did you know this?
1
Not at all
2
3
4
5
Perfectly
144
Q

“Gold standard” in the diagnosis of acute coronary syndrome (ACS):

A. Brain natriuretic peptide (BNP)
B. Cross-linked c-telopeptides
C. Myoglobin
D. Troponin

A

D. Troponin

ACUTE CORONARY SYNDROME/MYOCARDIAL INFARCTION
Cardiac troponin (cTn) represents a complex of regulatory proteins that include troponin I (cTnI) and troponin T (cTnT) that are specific to heart muscle.
cTnI and cTnT are the “gold standard” in the diagnosis of acute coronary syndrome (ACS).
cTn should be measured in all patients presenting with symptoms suggestive of ACS, in conjunction with physical examination and ECG.

How well did you know this?
1
Not at all
2
3
4
5
Perfectly
145
Q

Which test, if elevated, would PROVIDE INFORMATION about risk for developing coronary artery disease?

A. CK-MB
B. hs-CRP
C. Myoglobin
D. Troponin

A

B. hs-CRP

HIGH-SENSITIVITY CRP
Considered a GOOD PREDICTOR TEST for assessing cardiovascular risk

CK-MB, TROPONIN AND MYOGLOBIN
Used to assess if a myocardial infarction HAS OCCURRED

How well did you know this?
1
Not at all
2
3
4
5
Perfectly
146
Q

If elevated, which laboratory test would support a diagnosis of CONGESTIVE HEART FAILURE?

A. Albumin cobalt binding
B. B-type natriuretic peptide
C. Homocysteine
D. Troponin

A

B. B-type natriuretic peptide

B-type (brain) natriuretic peptide (BNP) is used to determine if physical symptoms are related to congestive heart failure.

How well did you know this?
1
Not at all
2
3
4
5
Perfectly
147
Q

Which two tests detect swelling of the ventricles that occurs in congestive heart failure?

A. BNP and electrocardiogram
B. BNP and echocardiogram
C. Troponin T and electrocardiogram
D. Troponin I and echocardiogram

A

B. BNP and echocardiogram

DIAGNOSIS OF CONGESTIVE HEART FAILURE:
Until recently, this condition was diagnosed strictly on the basis of symptomatology and/or as a result of procedures such as echocardiography, but more recently a biomarker for this condition is the brain form or B-type natriuretic peptide (BNP), which has been approved as a definitive test for this condition and appears to be an excellent marker for early heart failure.

How well did you know this?
1
Not at all
2
3
4
5
Perfectly
148
Q

Which of the following laboratory tests is a marker for ISCHEMIC HEART DISEASE?

A. Albumin cobalt binding
B. CK-MB isoforms
C. Free fatty acid binding protein
D. Myosin light chain

A

A. Albumin cobalt binding

Albumin cobalt binding is a test that measures ischemia-modified albumin, which is a marker for ischemic heart disease.

How well did you know this?
1
Not at all
2
3
4
5
Perfectly
149
Q

The turbid, or milky, appearance of serum after fat ingestion is termed postprandial lipemia, which is caused by the presence of what substance?

A. Bilirubin
B. Cholesterol
C. Chylomicron
D. Phospholipid

A

C. Chylomicron

These chylomicrons enter the blood through the lymphatic system, where they impart a turbid appearance to serum.

How well did you know this?
1
Not at all
2
3
4
5
Perfectly
150
Q

When the plasma appears OPAQUE AND MILKY, the triglyceride level is probably:

A. Less than 100 mg/dL
B. Less than 200 mg/dL
C. Greater than 300 mg/dL
D. Greater than 600 mg/dL

A

D. Greater than 600 mg/dL

The appearance of the plasma or serum can be observed and noted after a minimum 12- hour fast.

If the plasma is clear, the triglyceride level is probably less than 200 mg/dL. When the plasma appears hazy or turbid, the triglyceride level has increased to greater than 300 mg/dL, and if the specimen appears opaque and milky (lipemic, from chylomicrons), the triglyceride level is probably greater than 600 mg/dL.

Note:
1. Clear plasma: TAG < 200 mg/dL
2. Hazy or turbid plasma: TAG > 300 mg/dL
3. Opaque or milky plasma: TAG > 600 mg/dL

How well did you know this?
1
Not at all
2
3
4
5
Perfectly
151
Q

Which of the following tests would most likely be included in a routine lipid profile?

A. Total cholesterol, triglyceride, fatty acid, chylomicron
B. Total cholesterol, triglyceride, HDL cholesterol, phospholipid
C. Triglyceride, HDL cholesterol, LDL cholesterol, chylomicron
D. Total cholesterol, triglyceride, HDL cholesterol, LDL cholesterol

A

D. Total cholesterol, triglyceride, HDL cholesterol, LDL cholesterol

A “routine” lipid profile would most likely consist of the measurement of total cholesterol, triglyceride, HDL cholesterol, and LDL cholesterol.

How well did you know this?
1
Not at all
2
3
4
5
Perfectly
152
Q

To produce reliable results, when should blood specimens for lipid studies be drawn?

A. Immediately after eating
B. Anytime during the day
C. In the fasting state, approximately 2 to 4 hours after eating
D. In the fasting state, approximately 12 hours after eating

A

D. In the fasting state, approximately 12 hours after eating

Blood specimens for lipid studies should be drawn in the fasting state at least 9 to 12 hours after eating. Although fat ingestion only slightly affects cholesterol levels, the triglyceride results are greatly affected. Triglycerides peak at about 4 to 6 hours after a meal, and these exogenous lipids should be cleared from the plasma before analysis.

How well did you know this?
1
Not at all
2
3
4
5
Perfectly
153
Q

Which of the following lipid tests is LEAST affected by the fasting status of the patient?

A. Cholesterol
B. Triglyceride
C. Fatty acid
D. Lipoprotein

A

A. Cholesterol

Total cholesterol screenings are commonly performed on nonfasting individuals.
Total cholesterol is only slightly affected by the fasting status of the individual, whereas triglycerides, fatty acids, and lipoproteins are greatly affected.

How well did you know this?
1
Not at all
2
3
4
5
Perfectly
154
Q

The kinetic methods for quantifying serum triglyceride employ enzymatic hydrolysis. The hydrolysis of triglyceride may be accomplished by what enzyme?

A. Amylase
B. Leucine aminopeptidase
C. Lactate dehydrogenase
D. Lipase

A

D. Lipase

It is first necessary to hydrolyze the triglycerides to free fatty acids and glycerol. This hydrolysis step is catalyzed by the enzyme lipase.

The glycerol is then free to react in the enzyme-coupled reaction system that includes glycerokinase, pyruvate kinase, and lactate dehydrogenase or in the enzyme-coupled system that includes glycerokinase, glycerophosphate oxidase, and peroxidase.

How well did you know this?
1
Not at all
2
3
4
5
Perfectly
155
Q

The largest and the least dense of the lipoprotein particles:

A. LDL
B. HDL
C. VLDL
D. Chylomicrons

A

D. Chylomicrons

Chylomicrons, which contain apo B-48, are the largest and the least dense of the lipoprotein particles.
HDL, the smallest and most dense lipoprotein particle, is synthesized by both the liver and the intestine.

How well did you know this?
1
Not at all
2
3
4
5
Perfectly
156
Q

The smallest and most dense lipoprotein particle:

A. LDL
B. HDL
C. VLDL
D. Chylomicrons

A

B. HDL

Chylomicrons, which contain apo B-48, are the largest and the least dense of the lipoprotein particles.
HDL, the smallest and most dense lipoprotein particle, is synthesized by both the liver and the intestine.

How well did you know this?
1
Not at all
2
3
4
5
Perfectly
157
Q

An abnormal lipoprotein present in patients with biliary cirrhosis or cholestasis:

A. LDL
B. B-VLDL
C. Lp(a)
D. LpX

A

D. LpX

Lipoprotein X is an abnormal lipoprotein present in patients with biliary cirrhosis or cholestasis and in patients with mutations in lecithin:cholesterol acyltransferase (LCAT), the enzyme that esterifi es cholesterol.

How well did you know this?
1
Not at all
2
3
4
5
Perfectly
158
Q

Exogenous triglycerides are transported in the plasma in what form?

A. Phospholipids
B. Cholestryl esters
C. Chylomicrons
D. Free fatty acids

A

C. Chylomicrons

From the epithelial cells, the chylomicrons are released into the lymphatic system, which transports chylomicrons to the blood. The chylomicrons may then carry the triglycerides to adipose tissue for storage, to organs for catabolism, or to the liver for incorporation of the triglycerides into very-low-density lipoproteins (VLDLs). Chylomicrons are normally cleared
from plasma within 6 hours after a meal.

How well did you know this?
1
Not at all
2
3
4
5
Perfectly
159
Q

Select the lipoprotein fraction that carries most of the endogenous triglycerides.

A. VLDL
B. HDL
C. LDL
D. Chylomicrons

A

A. VLDL

VLDL transports the majority of endogenous triglycerides, while the triglycerides of chylomicrons are derived entirely from dietary absorption.

How well did you know this?
1
Not at all
2
3
4
5
Perfectly
160
Q

Each lipoprotein fraction is composed of varying amounts of lipid and protein components. The beta-lipoprotein fraction consists primarily of which lipid?

A. Fatty acid
B. Cholesterol
C. Phospholipid
D. Triglyceride

A

B. Cholesterol

The beta-lipoprotein fraction is composed of approximately 50% cholesterol, 6% triglycerides, 22% phospholipids, and 22% protein.

The beta-lipoproteins, which are also known as the low-density lipoproteins (LDLs), are the principal transport vehicle for cholesterol in the plasma.

How well did you know this?
1
Not at all
2
3
4
5
Perfectly
161
Q

The protein composition of HDL is what percentage by weight?

A. Less than 2%
B. 25%
C. 50%
D. 90%

A

C. 50%

About 50% of the weight of HDL is protein, largely apo A-I and apo A-II. The HDL is about 30% phospholipid and 20% cholesterol by weight.

How well did you know this?
1
Not at all
2
3
4
5
Perfectly
162
Q

High levels of cholesterol leading to increased risk of coronary artery disease would be associated with which lipoprotein fraction?

A. LDL
B. VLDL
C. HDL
D. Chylomicrons

A

A. LDL

How well did you know this?
1
Not at all
2
3
4
5
Perfectly
163
Q

What is the sedimentation nomenclature associated with alpha-lipoprotein?

A. Very-low-density lipoproteins (VLDLs)
B. High-density lipoproteins (HDLs)
C. Low-density lipoproteins (LDLs)
D. Chylomicrons

A

B. High-density lipoproteins (HDLs)

The HDLs, also known as the alpha-lipoproteins, have the greatest density of 1.063-1.210 g/mL and move the fastest electrophoretically toward the anode.

How well did you know this?
1
Not at all
2
3
4
5
Perfectly
164
Q

The quantification of the high-density lipoprotein cholesterol level is thought to be significant in the risk assessment of what disease?

A. Pancreatitis
B. Cirrhosis
C. Coronary artery disease
D. Hyperlipidemia

A

C. Coronary artery disease

The quantifi cation of the HDL cholesterol level is thought to contribute in assessing the risk that an individual may develop coronary artery disease (CAD).

There appears to be an inverse relationship between HDL cholesterol and CAD. With low levels of HDL cholesterol, the risk of CAD increases.

How well did you know this?
1
Not at all
2
3
4
5
Perfectly
165
Q

Coronary heart disease POSITIVE risk factor:

A. LDL-C concentration < 100 mg/dL
B. HDL-C concentration ≥ 60 mg/dL
C. HDL-C concentration < 40 mg/dL
D. None of these

A

C. HDL-C concentration < 40 mg/dL

POSITIVE (INCREASED) RISK FACTORS
* Age: ≥ 45 y for men; ≥ 55 y or premature menopause for women
* Family history of premature CHD
* Current cigarette smoking
* Hypertension (blood pressure ≥ 140/90 mm Hg or taking antihypertensive medication)
* LDL-C concentration ≥ 160 mg/dL (≥ 4.1 mmol/L), with ≤ 1 risk factor
* LDL-C concentration ≥ 130 mg/dL (3.4 mmol/L), with ≥ 2 risk factors
* LDL-C concentration ≥ 100 mg/dL (2.6 mmol/L), with CH D or risk equivalent
* HDL-C concentration < 40 mg/dL (< 1.0 mmol/L)
* Diabetes mellitus = CH D risk equivalent
* Metabolic syndrome (multiple metabolic risk factors)

NEGATIVE (DECREASED) RISK FACTORS
* HDL-C concentration ≥ 60 mg/dL (≥ 1.6 mmol/L)
* LDL-C concentration < 100 mg/dL (< 2.6 mmol/L)

How well did you know this?
1
Not at all
2
3
4
5
Perfectly
166
Q

Which apoprotein is inversely related to risk of coronary heart disease?

A. Apoprotein A-I
B. Apoprotein B100
C. Apoprotein C-II
D. Apoprotein E4

A

A. Apoprotein A-I

Apoprotein A-I and apo A-II are the principal apoproteins of HDL, and low apo A-I has a high correlation with atherosclerosis. Conversely, apo-B100 is the principal apoprotein of LDL, and an elevated level is a major risk factor in developing coronary heart disease.

How well did you know this?
1
Not at all
2
3
4
5
Perfectly
167
Q

LDL primarily contains:

A. Apo AI
B. Apo-AII
C. Apo-B100
D. Apo-B48

A

C. Apo-B100

LDL primarily contains apo B-100 and is more cholesterol rich than other apo B–containing lipoproteins.

Note:
Apo-AI, Apo-AII = HDL
Apo-B100 = LDL, VLDL
Apo-B48 = chylomicrons

How well did you know this?
1
Not at all
2
3
4
5
Perfectly
168
Q

The VLDL fraction primarily transports what substance?

A. Cholesterol
B. Chylomicron
C. Triglyceride
D. Phospholipid

A

C. Triglyceride

The VLDL fraction is primarily composed of triglycerides and lesser amounts of cholesterol and phospholipids.

How well did you know this?
1
Not at all
2
3
4
5
Perfectly
169
Q

A commonly used precipitating reagent to separate HDL cholesterol from other lipoprotein cholesterol fractions:

A. Zinc sulfate
B. Trichloroacetic acid
C. Heparin-manganese
D. Isopropanol

A

C. Heparin-manganese

Either a dextran sulfate-magnesium chloride mixture or a heparin sulfate-manganese chloride mixture may be used to precipitate the LDL and VLDL cholesterol fractions. This allows the HDL cholesterol fraction to remain in the supernatant.

How well did you know this?
1
Not at all
2
3
4
5
Perfectly
170
Q

Which of the following is associated with Tangier disease?

A. Apoprotein C-II deficiency
B. Homozygous apo-B100 deficiency
C. Apoprotein C-II activated lipase
D. Apoprotein A-I deficiency

A

D. Apoprotein A-I deficiency

Deficiency of apo A-I is seen in Tangier disease, a familial hypocholesterolemia.

How well did you know this?
1
Not at all
2
3
4
5
Perfectly
171
Q

A patient’s total cholesterol is 300 mg/dL, his HDL cholesterol is 50 mg/dL, and his triglyceride is 200 mg/dL. What is this patient’s calculated LDL cholesterol?

A. 200
B. 210
C. 290
D. 350

A

B. 210

Once the total cholesterol, triglyceride, and HDL cholesterol are known, LDL cholesterol can be quantified by using the Friedewald equation

LDL cholesterol = Total cholesterol — (HDL cholesterol + Triglyceride/5)

In this example, all results are in mg/dL:
LDL cholesterol
= 300 - (50 + 200/5)
= 300 - (90)
= 210 mg/dL

How well did you know this?
1
Not at all
2
3
4
5
Perfectly
172
Q

The Friedewald formula is not valid for triglycerides over_____.

A. Triglycerides over 100 mg/dL
B. Triglycerides over 200 mg/dL
C. Triglycerides over 300 mg/dL
D. Triglycerides over 400 mg/dL

A

D. Triglycerides over 400 mg/dL

LDL cholesterol may be calculated or measured directly:
1. Friedewald formula
Indirect, not valid for triglycerides over 400mg/dL
LDL cholesterol = total cholesterol — [HDL cholesterol + triglyceride/5]
2. Homogeneous assay uses detergents to block HDL and VLDL from reacting with the dye to form a colored chromogen product. An enzymatic cholesterol analysis is performed with only LDL cholesterol able to react.

How well did you know this?
1
Not at all
2
3
4
5
Perfectly
173
Q

Select the order of mobility of lipoproteins electrophoresed on cellulose acetate or agarose at pH 8.6.

A. – Chylomicrons→pre-β →β→α+
B. – β→pre-β→α→chylomicrons +
C. – Chylomicrons →β→pre-β→α +
D. – α→β→pre-β→chylomicrons +

A

C. – Chylomicrons →β→pre-β→α +

Although pre-β lipoprotein is lower in density than β lipoprotein, it migrates faster on agarose or cellulose acetate owing to its more negative apoprotein composition.

LIPOPROTEINS
1. By electrophoresis
From the origin: chylomicrons > beta (LDL) > prebeta (VLDL) > alpha (HDL) Anode

  1. By ultracentrifugation
    From the least dense and largest: chylomicrons > VLDL > LDL > HDL most dense and smallest
How well did you know this?
1
Not at all
2
3
4
5
Perfectly
174
Q

Floating beta lipoprotein:

A. Lp(a)
B. B-VLDL

A

B. B-VLDL

β-VLDL (‘fl oating β’ lipoprotein) is an abnormal lipoprotein that accumulates in type 3 hyperlipoproteinemia. It is richer in cholesterol than VLDL and apparently results from the defective catabolism of VLDL. The particle is found in the VLDL density range but migrates electrophoretically with or near LDL.

How well did you know this?
1
Not at all
2
3
4
5
Perfectly
175
Q

Sinking pre-β-lipoprotein:

A. Lp(a)
B. B-VLDL

A

A. Lp(a)

Lp(a) has a density similar to LDL, but migrates similarly to VLDL on electrophoresis. Thus it can be detected when the d > 1.006 g/mL protein is examined electrophoretically. When Lp(a) is present in concentrations exceeding 20-30 mg/dL (i.e., when it contributes more than about 10 mg/dL to the LDL-C measurement) an additional band with pre-β mobility is also observed in the d > 1.006 kg/L fraction (hence the name sinking pre-β-lipoprotein).

How well did you know this?
1
Not at all
2
3
4
5
Perfectly
176
Q

Which of the following may be described as a variant form of LDL, associated with increased risk of atherosclerotic cardiovascular disease?

A. Lp(a)
B. HDL
C. Apo-AI
D. Apo-AII

A

A. Lp(a)

Lipoprotein (a) is an apolipoprotein that is more commonly referred to as Lp(a). Although it is related structurally to LDL, Lp(a) is considered to be a distinct lipoprotein class with an electrophoretic mobility in the prebeta region.

Lp(a) is believed to interfere with the lysis of clots by competing with plasminogen in the coagulation cascade, thus increasing the likelihood of atherosclerotic cardiovascular disease.

How well did you know this?
1
Not at all
2
3
4
5
Perfectly
177
Q

Type V hyperlipoproteinemia:

A. Extremely elevated TG due to the presence of chylomicrons
B. Elevated LDL and VLDL
C. Elevated VLDL
D. Elevated VLDL and presence of chylomicrons

A

D. Elevated VLDL and presence of chylomicrons

BLOOD LIPOPROTEIN PATTERNS IN PATIENTS WITH HYPERLIPOPROTEINEMIA
Type I: Extremely elevated TG due to the presence of chylomicrons
Type IIa: Elevated LDL
Type IIb: Elevated LDL and VLDL
Type III: Elevated cholesterol, TG; presence of B-VLDL
Type IV: Elevated VLDL
Type V: Elevated VLDL and presence of chylomicrons

How well did you know this?
1
Not at all
2
3
4
5
Perfectly
178
Q

It is the result of POOR PERFUSION of the kidneys and therefore diminished glomerular filtration. The kidneys are otherwise normal in their functioning capabilities. Poor perfusion can result from dehydration, shock, diminished blood volume, or congestive heart failure.

A. Pre-renal azotemia
B. Renal azotemia
C. Post-renal azotemia

A

A. Pre-renal azotemia

How well did you know this?
1
Not at all
2
3
4
5
Perfectly
179
Q

It is caused primarily by DIMINISHED GLOMERULAR FILTRATION as a consequence of acute or chronic renal disease. Such diseases include acute glomerulonephritis, chronic glomerulonephritis, polycystic kidney disease, and nephrosclerosis.

A. Pre-renal azotemia
B. Renal azotemia
C. Post-renal azotemia

A

B. Renal azotemia

How well did you know this?
1
Not at all
2
3
4
5
Perfectly
180
Q

It is usually the result of any type of OBSTRUCTION in which urea is reabsorbed into the circulation. Obstruction can be caused by stones, an enlarged prostate gland, or tumors.

A. Pre-renal azotemia
B. Renal azotemia
C. Post-renal azotemia

A

C. Post-renal azotemia

How well did you know this?
1
Not at all
2
3
4
5
Perfectly
181
Q

Urea is produced from:

A. The catabolism of proteins and amino acids
B. Oxidation of pyrimidines
C. The breakdown of complex carbohydrates
D. Oxidation of purines

A

A. The catabolism of proteins and amino acids

Urea is generated by deamination of amino acids. Most is derived from the hepatic catabolism of proteins. Uric acid is produced by the catabolism of purines. Oxidation of pyrimidines produces orotic acid.

How well did you know this?
1
Not at all
2
3
4
5
Perfectly
182
Q

Creatinine is formed from the:

A. Oxidation of creatine
B. Oxidation of protein
C. Deamination of dibasic amino acids
D. Metabolism of purines

A

A. Oxidation of creatine

How well did you know this?
1
Not at all
2
3
4
5
Perfectly
183
Q

The red complex developed in the Jaffe method todetermine creatinine measurements is a result of the complexing of creatinine with which of the following?

A. Alkaline picrate
B. Diacetyl monoxide
C. Sulfuric acid
D. Sodium hydroxide

A

A. Alkaline picrate

The classic Jaffe reaction involves complexing of creatinine with an alkaline picrate solution to produce a red complex (Janovski complex).

How well did you know this?
1
Not at all
2
3
4
5
Perfectly
184
Q

The most widely used test of overall renal function is:

A. Urea
B. Creatinine
C. Proteinuria
D. Cystatin C

A

B. Creatinine

How well did you know this?
1
Not at all
2
3
4
5
Perfectly
185
Q

What substance may be measured as an alternative to creatinine for evaluating GFR?

A. Plasma urea
B. Cystatin C
C. Uric acid
D. Potassium

A

B. Cystatin C

How well did you know this?
1
Not at all
2
3
4
5
Perfectly
186
Q

Uric acid is derived from the:

A. Oxidation of proteins
B. Catabolism of purines
C. Oxidation of pyrimidines
D. Reduction of catecholamines

A

B. Catabolism of purines

Uric acid is the principal product of purine (adenosine and guanosine) metabolism.

How well did you know this?
1
Not at all
2
3
4
5
Perfectly
187
Q

Which of the following is measured using glutamate dehydrogenase and is a measure of advanced stages, poor prognosis, and coma in liver disease?

A. Total bilirubin
B. Ammonia
C. Unconjugated bilirubin
D. Urea

A

B. Ammonia

Severe liver disease is the most common cause of altered ammonia metabolism.
Therefore the monitoring of ammonia levels may be used to determine prognosis.

How well did you know this?
1
Not at all
2
3
4
5
Perfectly
188
Q

Blood ammonia levels are usually measured in order to evaluate:

A. Renal failure
B. Acid–base status
C. Hepatic coma
D. Gastrointestinal malabsorption

A

C. Hepatic coma

Hepatic coma is caused by accumulation of ammonia in the brain as a result of liver failure. The ammonia increases central nervous system pH and is coupled to glutamate, a central nervous system neurotransmitter, forming glutamine. Blood and cerebrospinal fluid ammonia levels are used to distinguish encephalopathy caused by cirrhosis or other liver
disease from nonhepatic causes and to monitor patients with hepatic coma.

How well did you know this?
1
Not at all
2
3
4
5
Perfectly
189
Q

What is the compound that comprises the majority of the nonprotein-nitrogen fractions in serum?

A. Uric acid
B. Creatinine
C. Ammonia
D. Urea

A

D. Urea

UREA > AMINO ACIDS > URIC ACID > CREATININE > CREATINE > AMMONIA

Constituents in the plasma that contain the element nitrogen are categorized as being protein- or nonprotein-nitrogen compounds. The principal substances included among the nonprotein-nitrogen compounds are urea, amino acids, uric acid, creatinine, creatine, and ammonia.
Of these compounds, urea is present in the plasma in the greatest concentration, comprising approximately 45% of the nonprotein-nitrogen fraction.

How well did you know this?
1
Not at all
2
3
4
5
Perfectly
190
Q

Urea concentration is calculated from the BUN by multiplying by a factor
of:

A. 0.5
B. 2.14
C. 6.45
D. 14

A

B. 2.14

BUN is multiplied by 2.14 to give the urea concentration in mg/dL.

How well did you know this?
1
Not at all
2
3
4
5
Perfectly
191
Q

Express 30 mg/dL of urea nitrogen as urea.

A. 14 mg/dL
B. 20 mg/dL
C. 50 mg/dL
D. 64 mg/dL

A

D. 64 mg/dL

30 x 2.14 = 64.2 mg/dL
When it is necessary to convert urea nitrogen values to urea, the concentration may be calculated easily by multiplying the urea nitrogen value by 2.14.
This factor is derived from the molecular mass of urea (60 daltons) and the molecular weight of its two nitrogen atoms (28):

How well did you know this?
1
Not at all
2
3
4
5
Perfectly
192
Q

In the urea method, the enzymatic action of urease is inhibited when blood for analysis is drawn in a tube containing what anticoagulant?

A. Sodium heparin
B. Sodium fluoride
C. Sodium oxalate
D. EDTA

A

B. Sodium fluoride

With the urease reagent systems for the quantifi cation of urea, the use of sodium fluoride must be avoided because of its inhibitory effect on this system.

How well did you know this?
1
Not at all
2
3
4
5
Perfectly
193
Q

In the diacetyl method, what does diacetyl react with to form a yellow product?

A. Ammonia
B. Urea
C. Uric acid
D. Nitrogen

A

B. Urea

In the diacetyl method, acidic diacetyl reacts directly with urea to form a yellow-diazine derivative.

How well did you know this?
1
Not at all
2
3
4
5
Perfectly
194
Q

Which of the following disorders is NOT associated with an elevated blood ammonia level?

A. Reye syndrome
B. Renal failure
C. Chronic liver failure
D. Diabetes mellitus

A

D. Diabetes mellitus

Diseases associated with elevated blood ammonia levels include Reye syndrome, renal failure, chronic liver failure, cirrhosis, and hepatic encephalopathy.

How well did you know this?
1
Not at all
2
3
4
5
Perfectly
195
Q

When measuring ammonia blood levels, which of the following might cause a false increase in this analyte?

A. The patient had two cigarettes 15 minutes prior to blood draw.
B. The patient was fasting for hours prior to blood collection.
C. Immediately after phlebotomy, the blood sample was maintained on ice.
D. The patient had a steak dinner the night before the blood draw.
E. None of the above will falsely increase the blood ammonia levels.

A

A. The patient had two cigarettes 15 minutes prior to blood draw.

Cigarette smoking by the patient is a significant source of ammonia contamination. It is recommended that patients do not smoke for several hours before a specimen is collected.

How well did you know this?
1
Not at all
2
3
4
5
Perfectly
196
Q

The assay for urea is only a rough estimate of renal function and will not show any significant level of increased concentration until the glomerular filtration rate is decreased by at least _____.

A. Glomerular filtration rate is decreased by at least 50%
B. Glomerular filtration rate is decreased by at least 60%
C. Glomerular filtration rate is decreased by at least 70%
D. Glomerular filtration rate is decreased by at least 80%

A

A. Glomerular filtration rate is decreased by at least 50%

The assay for urea is only a rough estimate of renal function and will not show any significant level of increased concentration until the glomerular filtration rate is decreased by at least 50%.
A more reliable single index of renal function is the test for serum creatinine. Contrary to urea concentration, creatinine concentration is relatively independent of protein intake (from the diet), degree of hydration, and protein metabolism.

How well did you know this?
1
Not at all
2
3
4
5
Perfectly
197
Q

What compound normally found in urine may be used to assess the completeness of a 24-hour urine collection?

A. Urea
B. Uric acid
C. Creatine
D. Creatinine

A

D. Creatinine

The quantity of creatinine formed daily is a relatively constant amount because it is related to muscle mass.
Therefore, it has been customary to quantify the creatinine present in a 24-hour urine specimen as an index of the completeness of the collection.

How well did you know this?
1
Not at all
2
3
4
5
Perfectly
198
Q

When mixed with phosphotungstic acid, what compound causes the reduction of the former to a tungsten blue complex?

A. Urea
B. Ammonia
C. Creatinine
D. Uric acid

A

D. Uric acid

Uric acid may be quantifi ed by reacting it with phosphotungstic acid reagent in alkaline solution.

How well did you know this?
1
Not at all
2
3
4
5
Perfectly
199
Q

Which of the following disorders is best characterized by laboratory findings that include increased serum levels of inorganic phosphorus, magnesium, potassium, uric acid, urea, and creatinine and decreased serum calcium and erythropoietin levels?

A. Chronic renal failure
B. Renal tubular disease
C. Nephrotic syndrome
D. Acute glomerulonephritis

A

A. Chronic renal failure

As renal function continues to be lost over time, chronic renal failure develops. Chronic renal failure is manifested by loss of excretory function, inability to regulate water and electrolyte balance, and increased production of parathyroid hormone, all of which contribute to the abnormal laboratory findings. The decreased production of erythropoietin
causes anemia to develop.

How well did you know this?
1
Not at all
2
3
4
5
Perfectly
200
Q

In gout, what analyte deposits in joints and other body tissues?

A. Calcium
B. Creatinine
C. Urea
D. Uric acid

A

D. Uric acid

Gout is a pathological condition that may be caused by a malfunction of purine metabolism or a depression in the renal excretion of uric acid. Two of the major characteristics of gout are hyperuricemia and a deposition of uric acid as monosodium urate crystals in joints, periarticular cartilage, bone, bursae, and subcutaneous tissue.

How well did you know this?
1
Not at all
2
3
4
5
Perfectly
201
Q

A complete deficiency of hypoxanthine guanine phosphoribosyl transferase results in which disease?

A. Lesch-Nyhan syndrome
B. Maple syrup urine disease
C. Reye’s syndrome
D. Megaloblastic anemia

A

A. Lesch-Nyhan syndrome

Lesch-Nyhan syndrome is an X-linked genetic disorder (seen only in males) caused by the complete deficiency of hypoxanthine-guanine phosphoribosyltransferase, an importantenzyme in the biosynthesis of purines.

How well did you know this?
1
Not at all
2
3
4
5
Perfectly
202
Q

During chemotherapy for leukemia, which of the following analytes would most likely be elevated in the blood?

A. Uric acid
B. Urea
C. Creatinine
D. Ammonia

A

A. Uric acid

An increase in serum uric acid levels may be seen during chemotherapy for leukemia. The cause of this is the accelerated breakdown of cell nuclei in response to the chemotherapy. Other proliferative disorders that may respond similarly are lymphoma, multiple myeloma, and polycythemia. It is important that serum uric acid be monitored during chemotherapy
to avoid nephrotoxicity.

How well did you know this?
1
Not at all
2
3
4
5
Perfectly
203
Q

What is the IMMEDIATE PRECURSOR of bilirubin formation?

A. Mesobilirubinogen
B. Verdohemoglobin
C. Urobilinogen
D. Biliverdin

A

D. Biliverdin

It is biliverdin that is the immediate precursor of bilirubin formation. Mesobilirubinogen and urobilinogen represent intestinal breakdown products of bilirubin catabolism.

How well did you know this?
1
Not at all
2
3
4
5
Perfectly
204
Q

To quantify serum bilirubin levels, it is necessary that bilirubin couples with diazotized sulfanilic acid to form what complex?

A. Verdobilirubin
B. Azobilirubin
C. Azobilirubinogen
D. Bilirubin glucuronide

A

B. Azobilirubin

Diazo reagent is a mixture of sulfanilic acid, sodium nitrite, and hydrochloric acid. The mixing of sodium nitrite with hydrochloric acid forms nitrous acid, which in turn reacts with sulfanilic acid to form a diazonium salt. This diazotized sulfanilic acid mixture, when mixed with solubilized bilirubin, forms a red azobilirubin complex.

How well did you know this?
1
Not at all
2
3
4
5
Perfectly
205
Q

What enzyme catalyzes the conjugation of bilirubin?

A. Leucine aminopeptidase
B. Glucose-6-phosphate dehydrogenase
C. Uridine diphosphate glucuronyltransferase
D. Carbamoyl phosphate synthetase

A

C. Uridine diphosphate glucuronyltransferase

In order for the bilirubin-albumin complex to reach the parenchymal cells of the liver, the complex must be transported from the sinusoids to the sinusoidal microvilli and into the parenchymal cell. The microsomal fraction of the parenchymal cell is responsible for the conjugation of bilirubin. It is here that bilirubin reacts with uridine diphosphate glucuronate
in the presence of the enzyme uridine diphosphate glucuronyltransferase to form bilirubin diglucuronide.

How well did you know this?
1
Not at all
2
3
4
5
Perfectly
206
Q

What breakdown product of bilirubin metabolism is produced in the colon from the oxidation of urobilinogen by microorganisms?

A. Porphobilinogen
B. Urobilin
C. Stercobilinogen
D. Protoporphyrin

A

B. Urobilin

In the colon, a portion of the urobilinogen is oxidized by the action of microorganisms to urobilin, which is excreted in the feces as an orange-brown pigment

How well did you know this?
1
Not at all
2
3
4
5
Perfectly
207
Q

Which of the following functions as a transport protein for bilirubin in the blood?

A. Albumin
B. Alpha-globulin
C. Beta-globulin
D. Gamma-globulin

A

A. Albumin

Albumin acts as the transport vehicle for unconjugated bilirubin in the blood, with each mole of albumin capable of binding two moles of bilirubin.

How well did you know this?
1
Not at all
2
3
4
5
Perfectly
208
Q

What term is used to describe the accumulation of bilirubin in the skin?

A. Jaundice
B. Hemolysis
C. Cholestasis
D. Kernicterus

A

A. Jaundice

Jaundice may be caused by an increase in either the unconjugated or conjugated form of bilirubin. Such increases in bilirubin levels may be caused by prehepatic, hepatic, or posthepatic disorders.

209
Q

In the condition kernicterus, the abnormal accumulation of bilirubin occurs in what tissue?

A. Brain
B. Liver
C. Kidney
D. Blood

A

A. Brain

Kernicterus refers to the accumulation of bilirubin in brain tissue that occurs with elevated levels of unconjugated bilirubin. This condition is most commonly seen in newborns with hemolytic disease resulting from maternal-fetal Rh incompatibility. Newborns afflicted with kernicterus will exhibit severe neural symptoms.

210
Q

As a reduction product of bilirubin catabolism, this compound is partially reabsorbed from the intestine through the portal circulation for reexcretion by the liver. What is this compound?

A. Verdohemoglobin
B. Urobilinogen
C. Urobilin
D. Biliverdin

A

B. Urobilinogen

In the small intestine, urobilinogen is formed through the enzymatic reduction process of anaerobic bacteria on bilirubin.

211
Q

Which of the following factors will NOT adversely affect the accurate quantification of bilirubin in serum?

A. Lipemia
B. Hemolysis
C. Exposure to light
D. Specimen refrigeration

A

D. Specimen refrigeration

Bilirubin will deteriorate when exposed to either white or UV light. This deterioration is also temperature sensitive. Thus, specimens for bilirubin analysis should be stored in the dark at refrigerator temperature until the assay can be performed.

Lipemia should be avoided, due to its interference with spectrophotometric analyses. Because hemoglobin reacts with diazo reagent, use of hemolyzed specimens should be avoided. Hemolysis will cause bilirubin results to be falsely low.

212
Q

Which bilirubin fraction is conjugated and covalently bound to albumin? - Bishop

A. Alpha
B. Beta
C. Delta
D. Gamma

A

C. Delta

Four bilirubin fractions represented by Greek letters have been identifi ed: unconjugated (alpha), monoconjugated (beta), diconjugated (gamma), and unconjugated bilirubin covalently bound to albumin (delta).

Delta-bilirubin is normally present in low concentration in the blood, and it is known to react directly with diazotized sulfanilic acid. Increased serum levels of delta-bilirubin are associated with liver-biliary disease.

According to Bishop: DELTA bilirubin is CONJUGATED bilirubin that is covalently bound to albumin.

213
Q

As the red blood cells disintegrate, hemoglobin is released and converted to the pigment bilirubin. Which organ is primarily responsible for this function?

A. Spleen
B. Kidneys
C. Intestines
D. Liver

A

A. Spleen

The cells of the reticuloendothelial system are responsible for the removal of old red blood cells from the peripheral circulation. As the red blood cells reach the end of their 120-day life span, the specialized cells mainly of the spleen phagocytize the aged cells and convert the released hemoglobin into the excretory pigment bilirubin.

214
Q

Which of the following does NOT accurately describe direct bilirubin?

A. Insoluble in water
B. Conjugated in the liver
C. Conjugated with glucuronic acid
D. Excreted in the urine of jaundiced patients

A

A. Insoluble in water

Direct bilirubin was so named because of its ability in the van den Bergh method to react directly with diazotized sulfanilic acid without the addition of alcohol. Such a direct reaction is possible because direct bilirubin is conjugated in the liver with glucuronic acid, thereby making it a polar, water-soluble compound. Because conjugated bilirubin is both water soluble and not protein bound, it may be filtered through the glomerulus and excreted in the urine of jaundiced patients.

215
Q

Excreted in the urine of jaundiced patients:

A. Conjugated bilirubin
B. Unconjugated bilirubin
C. Both of these
D. None of these

A

A. Conjugated bilirubin

Because conjugated bilirubin is both water soluble and not protein bound, it may be filtered through the glomerulus and excreted in the urine of jaundiced patients.

216
Q

Which of the following reagent systems contains the components sulfanilic acid, hydrochloric acid, and sodium nitrite?

A. Jaffe
B. Zimmerman
C. Diazo
D. Lowry

A

C. Diazo

Ehrlich’s DIAZO REAGENT consists of sulfanilic acid, hydrochloric acid, and sodium nitrite.

217
Q

Indirect-reacting bilirubin may be quantified by reacting it initially in which reagent?

A. Dilute hydrochloric acid
B. Dilute sulfuric acid
C. Caffeine-sodium benzoate
D. Sodium hydroxide

A

C. Caffeine-sodium benzoate

Unlike direct bilirubin, indirect-reacting bilirubin is insoluble in deionized water and dilute hydrochloric acid. Indirect-reacting bilirubin must first be mixed with methanol or caffeinesodium benzoate to solubilize it before proceeding with the diazo reaction.

Note:
EVELYN-MALLOY: methanol
JENDRASSIK-GROF: caffeine-sodium benzoate

218
Q

What condition is characterized by an elevation of total bilirubin primarily due to an increase in the CONJUGATED BILIRUBIN fraction?

A. Hemolytic jaundice
B. Neonatal jaundice
C. Crigler-Najjar syndrome
D. Obstructive jaundice

A

D. Obstructive jaundice

“Obstructive jaundice” is a term applied to conditions in which the common bile duct is obstructed because of gallstone formation, spasm, or neoplasm. Such an obstruction blocks the flow of bile from the gallbladder into the small intestine. This impedance of bile flow will result in a backfl ow of bile from the gallbladder into the sinusoids of the liver and
ultimately into the peripheral circulation. Because the liver is not initially involved and the disorder is of posthepatic origin, the increased levels of bilirubin in the blood are caused by the backfl ow of conjugated bilirubin.

219
Q

Which of the following is characteristic of hemolytic jaundice?

A. Unconjugated serum bilirubin level increased
B. Urinary bilirubin level increased
C. Urinary urobilinogen level decreased
D. Fecal urobilin level decreased

A

A. Unconjugated serum bilirubin level increased

Hemolytic jaundice is also referred to as prehepatic jaundice. It is caused by excessive destruction of erythrocytes at a rate that exceeds the conjugating ability of the liver. As a result, increased levels of unconjugated bilirubin appear in the blood.

220
Q

What may be the cause of NEONATAL PHYSIOLOGICAL JAUNDICE?

A. Hemolytic episode caused by an ABO incompatibility
B. Stricture of the common bile duct
C. Hemolytic episode caused by an Rh incompatibility
D. Deficiency in the bilirubin conjugation enzyme system

A

D. Deficiency in the bilirubin conjugation enzyme system

The enzyme uridine diphosphate glucuronyltransferase catalyzes the conjugation of bilirubin with glucuronic acid. In newborns, especially premature infants, this liver enzyme system is not fully developed or functional. Because of this deficiency in the enzyme system, the concentration of unconjugated bilirubin rises in the blood, because only the
conjugated form may be excreted through the bile and urine. The increased levels of unconjugated bilirubin will cause the infant to appear jaundiced. Generally, this condition persists for only a short period because the enzyme system usually becomes functional within several days after birth.

221
Q

Which of the following disorders is characterized by an INABILITY TO TRANSPORT BILIRUBIN from the sinusoidal membrane into the hepatocyte?

A. Carcinoma of the common bile duct
B. Crigler-Najjar syndrome
C. Dubin-Johnson syndrome
D. Gilbert syndrome

A

D. Gilbert syndrome

Gilbert syndrome is a preconjugation transport disturbance. In this disorder the hepatic uptake of bilirubin is defective because the transportation of bilirubin from the sinusoidal membrane to the microsomal region is impaired.

222
Q

Which of the following characterizes Crigler-Najjar syndrome?

A. Inability to transport bilirubin from the sinusoidal membrane to the microsomal region
B. Deficiency of the enzyme system required for conjugation of bilirubin
C. Inability to transport bilirubin glucuronides to the bile canaliculi
D. Severe liver cell damage accompanied by necrosis

A

B. Deficiency of the enzyme system required for conjugation of bilirubin

Both Crigler-Najjar syndrome and neonatal jaundice, a physiological disorder, are due to a deficiency in the enzyme-conjugating system. With a defi ciency in uridine diphosphate glucuronyltransferase, the liver is unable to conjugate bilirubin, and both of these conditions are characterized by increased levels of unconjugated bilirubin.

Unlike Crigler-Najjar syndrome, which is a hereditary disorder, neonatal physiological jaundice is a temporary situation that usually corrects itself within a few days after birth.

223
Q

Which of the following is NOT characteristic of Dubin-Johnson syndrome?

A. Impaired excretion of bilirubin into the bile
B. Hepatic uptake of bilirubin is normal
C. Inability to conjugate bilirubin
D. Increased level of bilirubin in urine

A

C. Inability to conjugate bilirubin

In Dubin-Johnson syndrome, the transport of conjugated (direct) bilirubin from the microsomal region to the bile canaliculi is impaired. In this rare familial disorder, plasma conjugated bilirubin levels are increased because of defective excretion of bilirubin in the bile. Because conjugated bilirubin is water soluble, increased amounts of bilirubin are found in the urine.

224
Q

Beta-gamma bridging effect:

A. Multiple myeloma
B. Hepatic cirrhosis
C. Nephrotic syndrome
D. Inflammation

A

B. Hepatic cirrhosis

Gamma spike: multiple myeloma
Beta-gamma bridging: hepatic cirrhosis
α1 globulin flat curve: juvenile cirrhosis (AATdefi ciency)
Α2 globulin band spike: nephrotic syndrome
Spikes in α1, α2 and β: inflammation

225
Q

Less than 80% liver damage:

A. Hepatitis
B. Cirrhosis

A

A. Hepatitis

HEPATITIS: less than 80% liver damage
High: AST, ALT, LD, ALP, bilirubin
Normal: Total protein, albumin, ammonia

226
Q

80% liver tissue damage:

A. Hepatitis
B. Cirrhosis

A

B. Cirrhosis

CIRRHOSIS: 80% liver tissue damage
Death of liver cells with regeneration leads to fibrosis, scarring and destruction of the normal liver architecture
Low: Total protein, albumin
High: Bilirubin, ammonia
Normal to slightly high: ALP
Normal: AST, ALT and LD

227
Q

Destruction of liver architecture:

A. Cirrhosis
B. Hepatitis
C. Bile duct obstruction
D. None of these

A

A. Cirrhosis

Cirrhosis is defi ned as destruction of the liver’s architecture.
The leading cause of this condition is alcohol abuse.

228
Q

Increased in hepatic jaundice:

A. B1 only
B. B2 only
C. B1 and B2
D. None of these

A

C. B1 and B2

Hemolytic jaundice: unconjugated bilirubin (B1)
Hepatic jaundice: unconjugated and conjugated bilirubin (B1 and B2)
Obstructive jaundice: conjugated bilirubin (B2)

229
Q

What does an increase in the serum enzyme levels indicate?

A. Decreased enzyme catabolism
B. Accelerated enzyme production
C. Tissue damage and necrosis
D. Increased glomerular filtration rate

A

C. Tissue damage and necrosis

The majority of serum enzymes that are of interest clinically are of intracellular origin.
These enzymes function intracellularly, with only small amounts found in serum as a result of normal cellular turnover. Increased serum levels are due to tissue damage and necrosis, where the cells disintegrate and leak their contents into the blood. Thus, elevated serum levels of intracellular enzymes are used diagnostically to assess tissue damage.

230
Q

When measuring enzyme activity, if the instrument is operating 5°C lower than the temperature prescribed for the method, how will the results be affected?

A. Lower than expected
B. Higher than expected
C. Varied, showing no particular pattern
D. All will be clinically abnormal.

A

A. Lower than expected

Factors that affect enzyme assays include temperature, pH, substrate concentration, and time of incubation. For each clinically important enzyme, the optimum temperature and pH for its specifi c reaction are known. When lower than optimum temperature or pH is employed, the measured enzyme activity will be lower than the expected activity value. As
temperature increases, the rate of the reaction increases.

231
Q

The properties of enzymes are correctly described by which of the following statements?

A. Enzymes are stable proteins.
B. Enzymes are protein catalysts of biological origin.
C. Enzymes affect the rate of a chemical reaction by raising the activation energy
needed for the reaction to take place.
D. Enzyme activity is not altered by heat denaturation.

A

B. Enzymes are protein catalysts of biological origin.

Enzymes are protein in nature. Like all proteins, they may be denatured with a loss of activity as a result of several factors (e.g., heat, extreme pH, mechanical agitation, strong acids, and organic solvents). Enzymes act as catalysts for the many chemical reactions of the body. Enzymes increase the rate of a specifi c chemical reaction by lowering the activation energy needed for the reaction to proceed.

232
Q

The shape of the key (substrate) must fit into the lock (enzyme):

A. Induced-fit theory by Emil Fischer
B. Induced-fit theory by Daniel Koshland
C. Lock-and-key theory by Emil Fischer
D. Lock-and-key theory by Daniel Koshland

A

C. Lock-and-key theory by Emil Fischer

LOCK-AND-KEY (EMIL FISCHER)
The shape of the key (substrate) must fit into the lock (enzyme)

INDUCED-FIT (DANIEL KOSHLAND)
Substrate binding to the active site of the enzyme

233
Q

The reaction rate is directly proportional to substrate concentration:

A. First-order kinetics
B. Zero-order kinetics

A

A. First-order kinetics

In 1913, Michaelis and Menten hypothesized the role of substrate concentration in formation of the enzyme–substrate (ES) complex. According to their hypothesis, the substrate readily binds to free enzyme at a low-substrate concentration. With the amount of enzyme exceeding the amount of substrate, the reaction rate steadily increases as more substrate is added. The reaction is following first-order kinetics because the reaction rate is directly proportional to substrate concentration.

Eventually, however, the substrate concentration is high enough to saturate all available enzyme, and the reaction velocity reaches its maximum. When product is formed, the resultant free enzyme immediately combines with excess free substrate. The reaction is in zero-order kinetics, and the reaction rate depends only on enzyme concentration.

234
Q

The reaction rate depends only on enzyme concentration:

A. First-order kinetics
B. Zero-order kinetics

A

B. Zero-order kinetics

In 1913, Michaelis and Menten hypothesized the role of substrate concentration in formation of the enzyme–substrate (ES) complex. According to their hypothesis, the substrate readily binds to free enzyme at a low-substrate concentration. With the amount of enzyme exceeding the amount of substrate, the reaction rate steadily increases as more substrate is added. The reaction is following first-order kinetics because the reaction rate is directly proportional to substrate concentration.

Eventually, however, the substrate concentration is high enough to saturate all available enzyme, and the reaction velocity reaches its maximum. When product is formed, the resultant free enzyme immediately combines with excess free substrate. The reaction is in zero-order kinetics, and the reaction rate depends only on enzyme concentration.

235
Q

Multiple measurements, usually of absorbance change, are made during the reaction, either at specific time intervals (usually every 30 or 60 seconds) or continuously by a continuous- recording spectrophotometer:

A. Fixed-time assay
B. Kinetic assay

A

B. Kinetic assay

One of two general methods may be used to measure the extent of an enzymatic reaction:
(1) fixed-time and (2) continuous-monitoring or kinetic assay.
1. In the fixed time method, the reactants are combined, the reaction proceeds for a designated time, the reaction is stopped (usually by inactivating the enzyme with a weak acid), and a measurement is made of the amount of reaction that has occurred. The reaction is assumed to be linear over the reaction time; the larger the reaction, the more
enzyme is present.
2. In continuous-monitoring or kinetic assays, multiple measurements, usually of absorbance change, are made during the reaction, either at specifi c time intervals (usually every 30 or 60 seconds) or continuously by a continuous- recording spectrophotometer

236
Q

An organic cofactor, such as nicotinamide adenine dinucleotide (NAD):

A. Activator
B. Coenzyme
C. Proenzyme
D. Zymogen

A

B. Coenzyme

A nonprotein molecule, called a cofactor, may be necessary for enzyme activity. Inorganic cofactors, such as chloride or magnesium ions, are called activators. A coenzyme is an organic cofactor, such as nicotinamide adenine dinucleotide (NAD).

When bound tightly to the enzyme, the coenzyme is called a prosthetic group. The enzyme portion (apoenzyme), with its respective coenzyme, forms a complete and active system, a holoenzyme.

237
Q

Inorganic cofactors, such as chloride or magnesium ions:

A. Activator
B. Coenzyme
C. Proenzyme
D. Zymogen

A

A. Activator

A nonprotein molecule, called a cofactor, may be necessary for enzyme activity. Inorganic cofactors, such as chloride or magnesium ions, are called activators. A coenzyme is an organic cofactor, such as nicotinamide adenine dinucleotide (NAD).

When bound tightly to the enzyme, the coenzyme is called a prosthetic group. The enzyme portion (apoenzyme), with its respective coenzyme, forms a complete and active system, a holoenzyme.

238
Q

Enzymes that catalyze the transfer of groups between compounds are classified as belonging to which enzyme class?

A. Hydrolases
B. Lyases
C. Oxidoreductases
D. Transferases

A

D. Transferases

There are six major classes of enzymes. The International Commission of Enzymes of the International Union of Biochemistry has categorized all enzymes into one of these classes: oxidoreductases, transferases, hydrolases, lyases, isomerases, and ligases. Transferases are enzymes that catalyze the transfer of groups, such as amino and phosphate groups, between compounds. Transferases frequently need coenzymes, such as pyridoxal-5’- phosphate (P-5-P), for the amino transfer reactions. Aspartate and alanine aminotransferases, creatine kinase, and gamma-glutamyltransferase are typical examples.

239
Q

Which of the following enzymes does not belong to the class of enzymes known as the hydrolases?

A. Alkaline phosphatase
B. Aldolase
C. Amylase
D. Lipase

A

B. Aldolase

Hydrolases are enzymes that split molecules with the addition of water—for example, amylase, lipase, alkaline phosphatase, acid phosphatase, 5’-nucleotidase, and trypsin. They do not usually require coenzymes but often need
activators.

Aldolase and carbonic anhydrase are examples of the class of enzymes known as the lyases. Lyases are enzymes that split molecules between carbon-to-carbon bonds without the addition of water. The resulting products usually contain carbon double bonds.

240
Q

To what class of enzymes does lactate dehydrogenase belong?

A. Isomerases
B. Ligases
C. Oxidoreductases
D. Transferases

A

C. Oxidoreductases

241
Q

Catalyze the joining of two substrate molecules, coupled with breaking of the pyrophosphate bond in adenosine triphosphate (ATP) or a similar compound:

A. Oxidoreductases
B. Hydrolases
C. Lyases
D. Ligases

A

D. Ligases

In addition to naming enzymes, the IUB system identifi es each enzyme by an EC numerical code containing four digits separated by decimal points. The first digit places the enzyme in one of the following six classes:

  1. Oxidoreductases. Catalyze an oxidation–reduction reaction between two substrates
  2. Transferases. Catalyze the transfer of a group other than hydrogen from one substrate to another
  3. Hydrolases. Catalyze hydrolysis of various bonds
  4. Lyases. Catalyze removal of groups from substrates without hydrolysis; the product ontains double bonds
  5. Isomerases. Catalyze the interconversion of geometric, optical, or positional isomers
  6. Ligases. Catalyze the joining of two substrate molecules, coupled with breaking of the pyrophosphate bond in adenosine triphosphate (ATP) or a similar compound
242
Q

Enzymes catalyze physiologic reactions by ____ the activation energy
level that the reactants (substrates) must reach for the reaction to occur.

A. Decreasing the activation energy
B. Increasing the activation energy

A

A. Decreasing the activation energy

Enzymes catalyze physiologic reactions by lowering the activation energy level that the reactants (substrates) must reach for the reaction to occur.

243
Q

The highest levels of total LD are seen in:

A. AMI and pulmonary infarction
B. Pernicious anemia and hemolytic disorders
C. Skeletal muscle disorders
D. Viral hepatitis and cirrhosis

A

B. Pernicious anemia and hemolytic disorders

B. Pernicious anemia and hemolytic disorders

244
Q

Most labile LD isoenzyme:

A. LD-1
B. LD-2
C. LD-3
D. LD-4
E. LD-5

A

E. LD-5

LD-5 is the most labile isoenzyme. Loss of activity occurs more quickly at 4°C than at 25°C. Serum samples for LD isoenzyme analysis should be stored at 25°C and analyzed within 24 hours of collection.

245
Q

The highest elevations of ALP activity occur in:

A. Biliary tract obstruction
B. Hepatitis
C. Osteomalacia
D. Paget’s disease

A

D. Paget’s disease

Elevated ALP levels may be observed in various bone disorders. Perhaps the highest elevations of ALP activity occur in Paget’s disease (osteitis deformans). Other bone disorders include osteomalacia, rickets, hyperparathyroidism, and osteogenic sarcoma.

246
Q

Which of the following disorders is NOT associated with an elevation of serum creatine kinase?

A. Duchenne-type progressive muscular dystrophy
B. Myocardial infarction
C. Cerebrovascular accidents (stroke)
D. Bone disease
E. Intramuscular injection

A

D. Bone disease

Increased serum creatine kinase (CK), formerly called creatine phosphokinase (CPK), values are caused primarily by lesions of cardiac muscle, skeletal muscle, or brain tissue. CK increases in the early stages of Duchenne-type progressive muscular dystrophy.
Assays of total CK and CK isoenzymes are commonly used in the diagnosis of myocardial infarction. Hypothyroidism causes a moderate increase in CK values. Elevation of this enzyme also occurs after vigorous muscular activity, in cases of cerebrovascular accidents (stroke), and after repeated intramuscular injections.

247
Q

To aid in the diagnosis of skeletal muscle disease, which of the following serum enzyme measurements would be of most use?

A. Creatine kinase
B. Alkaline phosphatase
C. Aspartate aminotransferase
D. Alanine aminotransferase

A

A. Creatine kinase

To aid in the diagnosis of skeletal muscle disease, measurement of creatine kinase would be most useful. CK yields the most reliable information when skeletal muscle disease is suspected.

Other enzymes that are also useful to measure are aspartate aminotransferase and lactate dehydrogenase. Both of these enzymes will be moderately elevated, whereas CK is significantly increased.

248
Q

When an AMI occurs, in what order (list first to last) will the enzymes aspartate aminotransferase (AST), creatine kinase(CK), and lactate dehydrogenase (LD) become elevated in the serum?

A. AST, LD, CK
B. CK, LD, AST
C. CK, AST, LD
D. LD, CK, AST

A

C. CK, AST, LD

When an AMI occurs, CK is the first enzyme to become elevated in the blood, rising within 4 to 6 hours following chest pain. AST exhibits a rise in the serum level within 6 to 8 hours. LD shows an increase in 8 to 12 hours following
infarction.
Measurement of these three enzymes to assess acute myocardial infarction has been replaced by cardiac troponin, myoglobin, and CK-MB.

249
Q

Pathological levels are DECREASED from the normal values, sometimes as much as 80 to 90%:

A. Acid phosphatase
B. Alkaline phosphatase
C. Cholinesterase
D. Creatine kinase

A

C. Cholinesterase

Normal serum levels of cholinesterase are quite high, refl ecting its continual synthesis and release by the liver. Decreased values are considered abnormal.

250
Q

The smallest enzyme:

A. Amylase
B. Lipase
C. CK
D. GGT

A

A. Amylase

AMYLASE is the smallest enzyme, with a molecular weight of 50,000 to 55,000 Da. Because of its small size, it is readily filtered by the renal glomerulus and also appears in the urine.

251
Q

All of the following are macroenzymes, except:

A. ACP and ALP
B. ALT and AST
C. CK
D. GGT
E. G6PD

A

E. G6PD

Macroenzymes are high-molecular-mass forms of the serum enzymes (ACP, ALP, ALT, AMY, AST, CK, GGT, LD, and LPS) that can be bound to either an immunoglobulin (macroenzyme type 1) or a nonimmunoglobulin substance (macroenzyme type 2).

Macroenzymes are usually found in patients who have an unexplained persistent increase of enzyme concentrations in serum. The presence of macroenzymes can also increase with increasing age.

252
Q

Which substrate is used in the Bowers–McComb method for ALP?

A. p-Nitrophenyl phosphate
B. β-Glycerophosphate
C. Phenylphosphate
D. α-Naphthylphosphate

A

A. p-Nitrophenyl phosphate

The method of Bowers–McComb (Szasz modifi cation) is the IFCC-recommended method for ALP. This method uses 2-amino-2-methyl-1-propanol, pH 10.15, and measures the increase in absorbance at 405 nm as p-nitrophenyl phosphate is hydrolyzed to p-nitrophenol.

253
Q

Which of the following buffers is used in the IFCC recommended method for ALP?

A. Glycine
B. Phosphate
C. 2-Amino-2-methyl-1-propanol
D. Citrate

A

C. 2-Amino-2-methyl-1-propanol

The method of Bowers–McComb (Szasz modifi cation) is the IFCC-recommended method for ALP. This method uses 2-amino-2-methyl-1-propanol, pH 10.15, and measures the increase in absorbance at 405 nm as p-nitrophenyl phosphate is hydrolyzed to p-nitrophenol.

254
Q

Kinetic enzymatic assays are best performed during which phase of an enzymatic reaction?

A. Linear phase
B. Lag phase
C. Plateau phase
D. Any phase as long as temperature and pH are constant

A

A. Linear phase

Enzyme assays are recommended to be performed during the linear phase, so that a consistent change over time can be used to calculate the enzyme concentration.

255
Q

To what metal does ceruloplasmin firmly bind?

A. Chromium
B. Copper
C. Zinc
D. Iron

A

B. Copper

Copper is found in the plasma mainly in two forms: a minor fraction loosely bound to albumin and the majority, representing about 80-95%, firmly bound to the enzyme ceruloplasmin, an alpha2-globulin, which is important in the oxidation of iron from the ferrous to the ferric state.

256
Q

Which trace metal is contained in glucose tolerance factor?

A. Chromium
B. Copper
C. Selenium
D. Zinc

A

A. Chromium

Cr(3+) is an essential dietary element and plays a role in maintaining normal metabolism of glucose, fat, and cholesterol.

Glucose tolerance factor (GTF), the biologically active form of chromium, is an essential dietary agent that potentiates the action of insulin and thereby functions in regulating carbohydrate metabolism.

257
Q

Manganese toxicity resembles the following disease:

A. Parkinson’s disease
B. Wilson’s disease
C. Alzheimer’s disease
D. Menkes disease

A

A. Parkinson’s disease

Chronic manganese toxicity resembles Parkinson’s disease with akinesia, rigidity, tremors, and mask-like faces.

258
Q

The metal ion essential for the activity of xanthine oxidase and xanthine dehydrogenase is:

A. Iron
B. Manganese
C. Molybdenum
D. Zinc

A

C. Molybdenum

Molybdenum is vital to human health through its inclusion in at least three enzymes: xanthine oxidase, aldehyde oxidase, and sulfite oxidase. The active site of these enzymes binds molybdenum in the form of a cofactor “molybdopterin”.

259
Q

Which of the following tumor markers is used to monitor persons with breast cancer for recurrence of disease?

A. Cathepsin-D
B. CA-15-3
C. Retinoblastoma gene
D. Estrogen receptor (ER)

A

B. CA-15-3

260
Q

Which tumor marker is used to determine trastuzumab (Herceptin) therapy for breast cancer?

A. PR
B. CEA
C. HER-2/neu
D. Myc

A

C. HER-2/neu

261
Q

Which of the following is the best analyte to monitor for recurrence of ovarian cancer?

A. CA 15-3
B. CA 19-9
C. CA-125
D. CEA

A

C. CA-125

262
Q

Which tumor marker is associated with cancer of the urinary bladder?

A. CA-19-9
B. CA-72-4
C. Nuclear matrix protein
D. Cathepsin-D

A

C. Nuclear matrix protein

263
Q

Which type of cancer is associated with the highest level of AFP?

A. Hepatoma
B. Ovarian cancer
C. Testicular cancer
D. Breast cancer

A

A. Hepatoma

264
Q

Major cation, or positively charged particle, and is found in the highest concentration in extracellular fluid:

A. Bicarbonate
B. Chloride
C. Potassium
D. Sodium

A

D. Sodium

265
Q

Major intracellular cation:

A. Bicarbonate
B. Chloride
C. Potassium
D. Sodium

A

C. Potassium

266
Q

Integral part of the transmission of nerve impulses:

A. Bicarbonate
B. Chloride
C. Potassium
D. Sodium

A

C. Potassium

As the primary intracellular cation, potassium is an integral part of the transmission of nerve impulses. Movement of potassium across the nerve tissue membrane permits the neural signal to move down the nerve fiber. Potassium also seems to be involved in synaptic processes, where the impulse “jumps” from one nerve fiber to another.

267
Q

It is the major anion that counterbalances the major cation, sodium.

A. Bicarbonate
B. Calcium
C. Chloride
D. Potassium

A

C. Chloride

268
Q

Two main functions in the body: (1) determining the osmotic pressure,
which controls the distribution of water among cells, plasma, and
interstitial fluid, and (2) maintaining electrical neutrality.

A. Bicarbonate
B. Chloride
C. Potassium
D. Sodium

A

B. Chloride

269
Q

Second most abundant anion in the extracellular fluid; major component
of the blood buffering system, accounts for 90% of total blood carbon
dioxide, and maintains charge neutrality in the cell:

A. Bicarbonate
B. Chloride
C. Magnesium
D. Potassium

A

A. Bicarbonate

Bicarbonate is the second most abundant anion in the extracellular fluid. It is a major component of the blood buffering system, accounts for 90% of total blood carbon dioxide, and maintains charge neutrality in the cell.

270
Q

Fourth most abundant cation in the body and second most abundant
intracellular ion:

A. Calcium
B. Magnesium
C. Potassium
D. Sodium

A

B. Magnesium

271
Q

Electrolyte(s) essential for blood coagulation:

A. Calcium
B. Calcium and magnesium
C. Calcium, magnesium and potassium
D. Bicarbonate, potassium and chloride

A

B. Calcium and magnesium

Electrolytes are an essential component in numerous processes, including:
1. Volume and osmotic regulation
(sodium [Na+], chloride [Cl−], potassium [K+])
2. Myocardial rhythm and contractility
(K+, magnesium [Mg2+], calcium [Ca2+])
3. Cofactors in enzyme activation
(e.g., Mg2+, Ca2+, zinc [Zn2+])
4. Regulation of adenosine triphosphatase (ATPase) ion pumps (Mg2+)
5. Acid–base balance
(bicarbonate HCO3−, K+, Cl−)
6. Blood coagulation
(Ca2+, Mg2+)
7. Neuromuscular excitability
(K+, Ca2+, Mg2+)
8. Production and use of ATP from glucose
(e.g., Mg2+, phosphate PO4−)

272
Q

Electrolyte(s) essential for acid-base balance:

A. Bicarbonate and calcium
B. Bicarbonate and chloride
C. Bicarbonate, potassium and chloride
D. Calcium and magnesium

A

C. Bicarbonate, potassium and chloride

273
Q

The presence of only slightly visible hemolysis will significantly increase
the serum level of which of the following electrolytes?

A. Sodium
B. Potassium
C. Chloride
D. Bicarbonate

A

B. Potassium

Hemolysis of blood specimens because of physiological factors is often difficult to differentiate from hemolysis produced by the blood collection itself. In either case, the concentration of potassium will be increased in the serum because of the release of the very high level of intracellular potassium from the erythrocytes into the plasma.
When hemolysis is present, the serum concentrations of sodium, bicarbonate, chloride, and calcium will be decreased because their concentrations are lower in erythrocytes than in plasma.

274
Q

Most abundant cation in the ECF, representing 90% of all extracellular
cations, and largely determines the osmolality of the plasma:

A. Bicarbonate
B. Chloride
C. Potassium
D. Sodium

A

D. Sodium

Na+ is the most abundant cation in the ECF, representing 90% of all extracellular cations, and largely determines the osmolality of the plasma.

275
Q

Hyponatremia is defined as a serum/plasma level:

A. Less than 165 mmol/L
B. Less than 145 mmol/L
C. Less than 140 mmol/L
D. Less than 135 mmol/L

A

D. Less than 135 mmol/L

Hyponatremia is defi ned as a serum/plasma level less than 135 mmol/L.
Hyponatremia is one of the most common electrolyte disorders in hospitalized and nonhospitalized patients.
Levels below 130 mmol/L are clinically signifi cant. Hyponatremia can be assessed by the cause for the decrease or with the osmolality level.

276
Q

Hyponatremia due to increased water retention, except:

A. Congestive heart failue
B. Hepatic cirrhosis
C. Diuretic use
D. Renal failure

A

C. Diuretic use

Causes Of Hyponatremia
1. Increased Sodium Loss
Hypoadrenalism
Potassium defi ciency
Diuretic use
Ketonuria
Salt-losing nephropathy
Prolonged vomiting or diarrhea
Severe burns

  1. Increased Water Retention
    Renal failure
    Nephrotic syndrome
    Hepatic cirrhosis
    Congestive heart failure
  2. Water Imbalance
    Excess water intake
    SIADH
    Pseudohyponatremia
277
Q

Hyponatremia can also be classified according to:

A. Chloride
B. Glucose
C. Plasma/serum osmolality
D. Urine osmolality

A

C. Plasma/serum osmolality

Hyponatremia can also be classifi ed according to plasma/serum osmolality.
Because Na+ is a major contributor to osmolality, both levels can assist in identifying the cause of hyponatremia.
There are three categories of hyponatremia—low osmolality, normal osmolality, or high osmolality. Most instances of hyponatremia occur with decreased osmolality.

278
Q

Can occur when sodium is measured using indirect ion-selective electrodes (ISEs) in a patient who is HYPERPROTEINEMIC or HYPERLIPIDEMIC.

A. Hyponatremia
B. Hypernatremia
C. Pseudohyponatremia
D. Pseudohypernatremia

A

C. Pseudohyponatremia

Pseudohyponatremia can occur when Na+ is measured using indirect ion-selective electrodes (ISEs) in a patient who is hyperproteinemic or hyperlipidemic. An indirect ISE dilutes the sample prior to analysis and as a result of plasma/serum water displacement; the ion levels are falsely decreased.

279
Q

The measurement of ________ is necessary to evaluate the cause of hypernatremia.

A. Chloride
B. Glucose
C. Plasma/serum osmolality
D. Urine osmolality

A

D. Urine osmolality

The measurement of urine osmolality is necessary to evaluate the cause of hypernatremia. With renal loss of water, the urine osmolality is low or normal. With extrarenal fluid losses, the urine osmolality is increased.

280
Q

With increased water loss, burn patients are most likely to also experience:

A. Hypernatremia
B. Hyponatremia
C. Hypomagnesemia
D. Hypoosmolality

A

A. Hypernatremia

Any condition that increases water loss, such as fever, burns, diarrhea, or exposure to heat, will increase the likelihood of developing hypernatremia.

281
Q

Major intracellular cation in the body:

A. Bicarbonate
B. Chloride
C. Potassium
D. Sodium

A

C. Potassium

Potassium (K+) is the major intracellular cation in the body, with a concentration 20 times greater inside the cells than outside.

Many cellular functions require that the body maintain a low ECF concentration of K+ ions.
As a result, only 2% of the body’s total K+ circulates in the plasma. Functions of K+ in the body include regulation of neuromuscular excitability, contraction of the heart, ICF volume, and H+ concentration.

282
Q

Hypokalemia due to gastrointestinal loss:

A. Acute leukemia
B. Alkalosis
C. Hypomagnesemia
D. Vomiting

A

D. Vomiting

Causes Of Hypokalemia
1. Gastrointestinal Loss
Vomiting
Diarrhea
Gastric suction
Intestinal tumor
Malabsorption
Cancer therapy—chemotherapy, radiation therapy
Large doses of laxatives
2. Renal Loss
Diuretics—thiazides, mineralocorticoids
Nephritis
Renal tubular acidosis
Hyperaldosteronism
Cushing’s syndrome
Hypomagnesemia
Acute leukemia
3. Cellular Shift
Alkalosis
Insulin overdose
4. Decreased Intake

283
Q

All are associated with hyperkalemia, except:

A. Acidosis
B. Alkalosis
C. Oral or intravenous potassium therapy
D. Diuretics

A

B. Alkalosis

K+ concentration also affects the H+ concentration in the blood. For example, in hypokalemia (low serum K+), as K+ is lost from the body, Na+ and H+ move into the cell. The H+ concentration is, therefore, decreased in the ECF, resulting in alkalosis.

CAUSES OF HYPERKALEMIA
1. Decreased Renal Excretion
Acute or chronic renal failure (GFR < 20 mL/min)
Hypoaldosteronism
Addison’s disease
Diuretics
2. Cellular Shift
Acidosis
Muscle/cellular injury
Chemotherapy
Leukemia
Hemolysis
3. Increased Intake
Oral or intravenous potassium replacement therapy
4. Artifactual
Sample hemolysis
Thrombocytosis
Prolonged tourniquet use or excessive fist clenching

284
Q

Major extracellular anion:

A. Bicarbonate
B. Chloride
C. Potassium
D. Sodium

A

B. Chloride

Chloride (Cl−) is the major extracellular anion.
It is involved in maintaining osmolality, blood volume, and electric neutrality.
In most processes, Cl− shifts secondarily to a movement of Na+ or HCO3 −.

285
Q

Which of the following disorders is characterized by increased production
of chloride in sweat?

A. Multiple myeloma
B. Hypoparathyroidism
C. Cystic fibrosis
D. Wilson disease

A

C. Cystic fibrosis

Measuring the concentration of chloride in sweat is a commonly used diagnostic procedure for determining the disorder of cystic fibrosis (CF). The majority of patients with CF will present with increased concentrations of sodium and chloride in their sweat.

286
Q

The second most abundant anion in the ECF:

A. Bicarbonate
B. Chloride
C. Potassium
D. Sodium

A

A. Bicarbonate

Bicarbonate is the second most abundant anion in the ECF. Total CO2 comprises the bicarbonate ion (HCO3−), H2CO3, and dissolved CO2, with HCO3− accounting for more than 90% of the total CO2 at physiologic pH.
Because HCO3− composes the largest fraction of total CO2, total CO2 measurement is indicative of HCO3− measurement.

287
Q

The fourth most abundant cation in the body and second most abundant intracellular ion:

A. Bicarbonate
B. Calcium
C. Chloride
D. Magnesium

A

D. Magnesium

Magnesium (Mg2+) is the fourth most abundant cation in the body and second most abundant intracellular ion. The average human body (70 kg) contains 1 mol (24 g) of Mg2+. Approximately 53% of Mg2+ in the body is found in bone, 46% in muscle and other organs and soft tissue, and less than 1% is present in serum and RBCs.

288
Q

Most frequently observed in hospitalized individuals in intensive care units (ICUs) or those receiving diuretic therapy or digitalis therapy:

A. Hypomagnesemia
B. Hypermagnesemia
C. Hypocalcemia
D. Hypercalcemia

A

A. Hypomagnesemia

Hypomagnesemia is most frequently observed in hospitalized individuals in intensive care units (ICUs) or those receiving diuretic therapy or digitalis therapy.
These patients most likely have an overall tissue depletion of Mg2+ as a result of severe illness or loss, which leads to low serum levels. Hypomagnesemia is rare in nonhospitalized individuals.

289
Q

Regulate(s) calcium:

A. Vitamin D
B. Vitamin D and calcitonin
C. Parathyroid hormone and calcitonin
D. Parathyroid hormone, vitamin D and calcitonin

A

D. Parathyroid hormone, vitamin D and calcitonin

Three hormones, PTH, vitamin D, and calcitonin, are known to regulate serum Ca2+ by altering their secretion rate in response to changes in ionized Ca2+.

290
Q

Of the total serum calcium, free ionized calcium normally represents approximately what percent?

A. 10
B. 40
C. 50
D. 90

A

C. 50

Free ionized calcium normally accounts for about 50% of total serum calcium, with the remainder being made up of complexed calcium (about 10%) and calcium bound to proteins (about 40%).

291
Q

Which of the following reagents is used in a colorimetric method to quantify the concentration of serum calcium?

A. Cresolphthalein complexone
B. Lanthanum
C. Malachite green
D. Amino-naphthol-sulfonic acid

A

A. Cresolphthalein complexone

Total serum calcium concentration is often determined by the spectrophotometric quantification of the color complex formed with cresolphthalein complexone.

292
Q

Which of the following reagents is used to determine the concentration of
serum inorganic phosphate?

A. Ehrlich’s reagent
B. Ammonium molybdate
C. 8-Hydroxyquinoline
D. Bathophenanthroline

A

B. Ammonium molybdate

Serum inorganic phosphate concentrations are determined most commonly by reacting with ammonium molybdate reagent. The molybdenum-phosphate complexes can be quantifi ed at 340 nm. Alternately, treatment of the phosphomolybdate compound formed with a reducing agent leads to the formation of molybdenum blue, which can be measured spectrophotometrically.

293
Q

PLEASE CHECK FIVE (5) BOXES: Elevated anion gap.

  • Hypercalcemia
  • Hypernatremia
  • Hypoalbuminemia
  • Ketoacidosis
  • Lactic acidosis
  • Methanol, ethanol, ethylene glycol poisoning
  • Uremia/renal failure
A
  • Hypernatremia
  • Ketoacidosis
  • Lactic acidosis
  • Methanol, ethanol, ethylene glycol poisoning
  • Uremia/renal failure

An elevated AG may be caused by uremia/renal failure, which leads to PO4 and SO4 retention; ketoacidosis, as seen in cases of starvation or diabetes; methanol, ethanol, ethylene glycol, or salicylate poisoning; lactic acidosis; hypernatremia; and instrument error.

Low AG values are rare but may be seen with hypoalbuminemia (decrease in unmeasured anions) or severe hypercalcemia (increase in unmeasured cations).

294
Q

PLEASE CHECK TWO (2) BOXES: Low anion gap.

  • Hypercalcemia
  • Hypernatremia
  • Hypoalbuminemia
  • Ketoacidosis
  • Lactic acidosis
  • Methanol, ethanol, ethylene glycol poisoning
  • Uremia/renal failure
A
  • Hypercalcemia
  • Hypoalbuminemia

An elevated AG may be caused by uremia/renal failure, which leads to PO4 and SO4 retention; ketoacidosis, as seen in cases of starvation or diabetes; methanol, ethanol, ethylene glycol, or salicylate poisoning; lactic acidosis; hypernatremia; and instrument error.
Low AG values are rare but may be seen with hypoalbuminemia (decrease in unmeasured anions) or severe hypercalcemia (increase in unmeasured cations).

295
Q

The sample of choice for measuring blood osmolality is:

A. Serum
B. Plasma
C. Whole blood
D. Serum or plasma may be used

A

A. Serum

Osmolality may be measured in serum or urine. Major electrolyte concentrations, mainly sodium, chloride, and bicarbonate, provide the largest contribution to the osmolality value of serum.
Plasma use is not recommended because osmotically active substances may be introduced into the specimen from the anticoagulant.

296
Q

Of the total serum osmolality, sodium, chloride, and bicarbonate ions normally contribute approximately what percent?

A. 8
B. 45
C. 75
D. 92

A

D. 92

For monovalent cations or anions the contribution to osmolality is approximately 92%.
Other serum electrolytes, serum proteins, glucose, and urea contribute to the remaining 8%.

297
Q

Which electrolyte level best correlates with plasma osmolality?

A. Sodium
B. Chloride
C. Bicarbonate
D. Calcium

A

A. Sodium

298
Q

Which formula is most accurate in predicting plasma osmolality?

A. Na + 2(Cl) + BUN + glucose
B. 2(Na) + 2(Cl) + glucose + urea
C. 2(Na) + (glucose ÷ 18) + (BUN ÷ 2.8)
D. Na + Cl + K + HCO3

A

C. 2(Na) + (glucose ÷ 18) + (BUN ÷ 2.8)

299
Q

What is the primary storage form of iron?

A. Apotransferrin
B. Myoglobin
C. Ferritin
D. Hemosiderin

A

C. Ferritin

In adults the total body iron content averages 3-4 g. The majority of this iron is found in the active pool as an essential constituent of hemoglobin, with a much lesser amount being an integral component of myoglobin and a number of enzymes.
Approximately 25% of the body iron is found in inactive storage forms. The major storage form of iron is ferritin, with a lesser amount being stored as hemosiderin.

300
Q

The anticoagulant of choice for arterial blood gas measurements is
____ in the ____ state.

A. Lithium heparin; dry
B. EDTA; dry
C. Potassium oxalate; liquid
D. Sodium citrate; dry

A

A. Lithium heparin; dry

Evacuated collection tubes are not appropriate for blood gases. While both dry (lyophilized) and liquid heparin are acceptable anticoagulants, the liquid form is not recommended because excessive amounts can dilute the sample and possibly alter the sample due to equilibration with room air.

301
Q

If a blood gas specimen is left exposed to air, which of the following changes will occur?

A. pO2 and pH increase; pCO2 decreases
B. pO2 and pH decrease; pCO2 increases
C. pO2 increases; pH and pCO2 decrease
D. pO2 decreases; pH and pCO2 increase

A

A. pO2 and pH increase; pCO2 decreases

Note:
OPEN TUBE (ENTRY OF OXYGEN)
⬆️ Increased pO2
⬇️ Deceased pCO2 (H2CO3)
⬆️ Increased pH (ALKALINE)
CLOSED TUBE (OXYGEN UTILIZED BY CELLS)
⬇️ Decreased pO2
⬆️ Increased pCO2 (H2CO3)
⬇️ Decreased pH (ACIDIC)

302
Q

How would blood gas parameters change if a sealed specimen is left at room temperature for 2 or more hours?

A. pO2 increases, pCO2 increases, pH increases
B. pO2 decreases, pCO2 decreases, pH decreases
C. pO2 decreases, pCO2 increases, pH decreases
D. pO2 increases, pCO2 increases, pH decreases

A

C. pO2 decreases, pCO2 increases, pH decreases

Note:
OPEN TUBE (ENTRY OF OXYGEN)
⬆️ Increased pO2
⬇️ Deceased pCO2 (H2CO3)
⬆️ Increased pH (ALKALINE)
CLOSED TUBE (OXYGEN UTILIZED BY CELLS)
⬇️ Decreased pO2
⬆️ Increased pCO2 (H2CO3)
⬇️ Decreased pH (ACIDIC)

303
Q

Which is the most predominant buffer system in the body?

A. Bicarbonate/carbonic acid
B. Acetate/acetic acid
C. Phosphate/phosphorous acid
D. Hemoglobin

A

A. Bicarbonate/carbonic acid

Because of its high concentration in blood, the bicarbonate/carbonic acid pair is the most important buffer system in the blood. This buffer system is also effective in the lungs and in the kidneys in helping to regulate body pH.
The other buffers that also function to help maintain body pH are the phosphate, protein, and hemoglobin buffer systems.

304
Q

To maintain a pH of 7.4 in plasma, it is necessary to maintain a:

A. 10:1 ratio of bicarbonate to carbonic acid
B. 20:1 ratio of bicarbonate to carbonic acid
C. 1:20 ratio of bicarbonate to carbonic acid
D. 20:1 ratio of carbonic acid to bicarbonate

A

B. 20:1 ratio of bicarbonate to carbonic acid

When the ratio of the concentrations of bicarbonate to carbonic acid is 20:1, the pH is 7.4.

305
Q

The normal ratio of CARBONIC ACID TO BICARBONATE in arterial blood
is:

A. 1:20
B. 7.4:6.1
C. 0.003:1.39
D. 20:1

A

A. 1:20

306
Q

Driving force of the bicarbonate buffer system:

A. Bicarbonate
B. Carbon dioxide
C. Chloride
D. Hydrogen

A

B. Carbon dioxide

307
Q

Fever:

A. Will decrease pO2 by 3%
B. Will increase pO2 by 3%
C. Will decrease pO2 by 7%
D. Will increase pO2 by 7%

A

C. Will decrease pO2 by 7%

Fever will decrease pO2 by 7%
Fever will increase pCO2 by 3%

308
Q

Fever:

A. Will decrease pCO2 by 3%
B. Will increase pCO2 by 3%
C. Will decrease pCO2 by 7%
D. Will increase pCO2 by 7%

A

B. Will increase pCO2 by 3%

Fever will decrease pO2 by 7%
Fever will increase pCO2 by 3%

309
Q

The role of the lungs and kidneys in maintaining pH is depicted with the
Henderson-Hasselbalch equation. The numerator denotes:

A. Kidney function
B. Lung function

A

A. Kidney function

The role of the lungs and kidneys in maintaining pH is depicted with the Henderson-Hasselbalch equation.
The numerator (HCO3−) denotes kidney functions, and the denominator (pCO2) denotes lung function.

310
Q

The role of the lungs and kidneys in maintaining pH is depicted with the Henderson-Hasselbalch equation. The denominator denotes:

A. Kidney function
B. Lung function

A

B. Lung function

The role of the lungs and kidneys in maintaining pH is depicted with the Henderson-Hasselbalch equation.
The numerator (HCO3−) denotes kidney functions, and the denominator (pCO2) denotes lung function.

311
Q

In the plasma, an excess in the concentration of bicarbonate without a change in pCO2 from normal will result in what physiological state?

A. Respiratory acidosis
B. Respiratory alkalosis
C. Metabolic acidosis
D. Metabolic alkalosis

A

D. Metabolic alkalosis

An excess of bicarbonate without a change in pCO2 will increase the ratio of bicarbonate
to carbonic acid. Therefore, the pH will increase; that is, the plasma becomes more
alkaline.

312
Q

Which set of results is consistent with uncompensated respiratory alkalosis?

A. pH 7.70 HCO3- 30 mmol/L, pCO2 25 mm Hg
B. pH 7.66 HCO3- 22 mmol/L pCO2 20 mm Hg
C. pH 7.46 HCO3- 38 mmol/L pCO2 55 mm Hg
D. pH 7.36 HCO3- 22 mmol/L pCO2 38 mm Hg

A

B. pH 7.66 HCO3- 22 mmol/L pCO2 20 mm Hg

TIPS FOR EVALUATING ACID-BASE DISORDERS
1. Look at the pH: determine if acidosis or alkalosis
2. Compare pCO2 and HCO3-
A. pCO2 going opposite to pH – RESPIRATORY
Abnormal pCO2 respiratory [↓pH ↑pCO2 respiratory acidosis] [↑pH ↓pCO2 respiratory alkalosis]
B. HCO3- going same direction as pH - METABOLIC
Abnormal HCO3- metabolic [↓pH ↓HCO3- metabolic acidosis] [↑ pH ↑ HCO3- metabolic alkalosis]
3. If pH is normal, full compensation occurred
4. If main compensatory mechanism kicked in, but pH still out of normal range, partial compensation has occurred

313
Q

Which set of results is consistent with uncompensated metabolic acidosis?

A. pH 7.25 HCO3- 15 mmol/L pCO2 37 mm Hg
B. pH 7.30 HCO3- 16 mmol/L pCO2 28 mm Hg
C. pH 7.45 HCO3- 22 mmol/L pCO2 40 mm Hg
D. pH 7.40 HCO3- 25 mmol/L pCO2 40 mm Hg

A

A. pH 7.25 HCO3- 15 mmol/L pCO2 37 mm Hg

314
Q

A patient’s blood gas results are: pH = 7.50; pCO2 = 55 mm Hg; and HCO3– = 40 mmol/L. These results indicate:

A. Respiratory acidosis
B. Respiratory alkalosis
C. Metabolic acidosis
D. Metabolic alkalosis

A

D. Metabolic alkalosis

A pH above 7.45 corresponds with alkalosis. Both bicarbonate and pCO2 are elevated. Bicarbonate is the conjugate base and is under metabolic (renal) control, while pCO2 is an acid and is under respiratory control. Increased bicarbonate (but not increased CO2) results in alkalosis; therefore, the classifi cation is metabolic alkalosis, partially compensated by increased pCO2.

315
Q

The following conditions are all causes of alkalosis. Which condition is
associated with respiratory alkalosis?

A. Anxiety
B. Hypovolemia
C. Hyperaldosteronism
D. Severe diarrhea

A

A. Anxiety

316
Q

Which of the following blood gas parameters are measured directly by the blood gas analyzer electrochemically as opposed to being calculated by the instrument?

A. pH, HCO3- and total CO2
B. pCO2, HCO3- and pO2
C. pH, pCO2 and pO2
D. pO2, HCO3- and total CO2

A

C. pH, pCO2 and pO2

pH, pCO2, and pO2 are measured directly from the specimen by utilizing electrodes. The pH and PCO2 electrodes are potentiometric where the voltage produced across a semipermeable membrane to hydrogen ions or CO2 gas is proportional to the “activity” of those ions in the patient’s sample. Activity is measured in voltage whose value can be
presented in terms of concentration. pO2 is measured similarly, but using an amperometric electrode.

Note:
pH and pCO2 = POTENTIOMETRY
pO2 = AMPEROMETRY

317
Q

A substance that can yield a hydrogen ion (H+) or hydronium ion when dissolved in water:

A. Acid
B. Base
C. Base excess
D. Buffer

A

A. Acid

318
Q

A substance that can yield hydroxyl ions (OH-):

A. Acid
B. Base
C. Base excess
D. Buffer

A

B. Base

319
Q

The combination of a weak acid or weak base and its salt, is a system that resists changes in pH:

A. Acid
B. Base
C. Base excess
D. Buffer

A

D. Buffer

320
Q

Mixed respiratory and nonrespiratory disorders ____ arise from more than one pathologic process.

A. Rarely
B. Occasionally
C. Frequently
D. Mostly

A

B. Occasionally

Mixed respiratory and nonrespiratory disorders occasionally arise from more than one pathologic process and represent the most serious of medical conditions as compensation for the primary disorder is failing.

321
Q

An emphysema patient suffering from fluid accumulation in the alveolar spaces is likely to be in what metabolic state?

A. Respiratory acidosis
B. Respiratory alkalosis
C. Metabolic acidosis
D. Metabolic alkalosis

A

A. Respiratory acidosis

322
Q

Master gland:

A. Adrenal cortex
B. Adrenal medulla
C. Pituitary gland
D. Thyroid gland

A

C. Pituitary gland

323
Q

Adenohypophysis:

A. Anterior pituitary gland
B. Posterior pituitary gland

A

A. Anterior pituitary gland

324
Q

Neurohypophysis:

A. Anterior pituitary gland
B. Posterior pituitary gland

A

B. Posterior pituitary gland

325
Q

Tropic hormones:

  • ACTH
  • FSH
  • GH
  • LH
  • Prolactin
  • TSH
A
  • ACTH
  • FSH
  • LH
  • TSH
326
Q

Direct effectors:

  • ACTH
  • FSH
  • GH
  • LH
  • Prolactin
  • TSH
A
  • GH
  • Prolactin
327
Q

Also called somatotropin:

A. ACTH
B. GH
C. LH
D. Prolactin

A

B. GH

Growth hormone (GH), also called somatotropin, is structurally related to prolactin and human placental lactogen.

328
Q

The definitive suppression test to prove autonomous production of
growth hormone is:

A. Oral glucose loading
B. Somatostatin infusion
C. Estrogen priming
D. Dexamethasone suppression

A

A. Oral glucose loading

329
Q

All of the following inhibit growth hormone secretion, except:

A. Glucose loading
B. Insulin deficiency
C. Thyroxine deficiency
D. Amino acids

A

D. Amino acids

330
Q

Prolactin is produced by the:

A. Anterior pituitary gland
B. Posterior pituitary gland
C. Adrenal cortex
D. Adrenal medulla

A

A. Anterior pituitary gland

331
Q

Diabetes insipidus:

A. Vasopressin deficiency
B. Vasopressin excess

A

A. Vasopressin deficiency

332
Q

Select the most appropriate single screening test for thyroid disease.

A. Free thyroxine index
B. Total T3 assay
C. Total T4
D. TSH assay

A

D. TSH assay

TSH is produced by the anterior pituitary in response to low levels of free T4 or T3. A normal TSH rules out thyroid disease. TSH is low in primary hyperthyroidism and high in primary hypothyroidism.

333
Q

The serum TSH level is almost absent in:

A. Primary hyperthyroidism
B. Primary hypothyroidism
C. Secondary hyperthyroidism
D. Euthyroid sick syndrome

A

A. Primary hyperthyroidism

In primary hyperthyroidism, the TSH will be within a range of 0–0.02 mU/mL, while in nonthyroid illnesses it will be 0.03 mU/mL or higher.

334
Q

A patient has an elevated serum T3 and free T4 and undetectable TSH.
What is the most likely cause of these results?

A. Primary hyperthyroidism
B. Secondary hyperthyroidism
C. Euthyroid with increased thyroxine-binding proteins
D. Euthyroid sick syndrome

A

A. Primary hyperthyroidism

335
Q

Associated with neonatal hypothyroidism:

A. Cretinism
B. Growth retardation
C. Mental retardation
D. All of these

A

D. All of these

336
Q

Critical for sodium retention (volume), potassium, and acid–base
homeostasis.

A. Aldosterone
B. Cortisol

A

A. Aldosterone

Zona glomerulosa (G-zone) cells (outer 10%) synthesize mineralocorticoids (aldosterone) critical for sodium retention (volume), potassium, and acid–base homeostasis. They have low cytoplasmic-to-nuclear ratios and small nuclei with dense chromatin with intermediate lipid inclusions.

337
Q

Critical to blood glucose homeostasis and blood pressure:

A. Aldosterone
B. Cortisol

A

B. Cortisol

Zona fasciculata (F-zone) cells (middle 75%) synthesize glucocorticoids, such as cortisol and cortisone critical to blood glucose homeostasis and blood pressure.

338
Q

First responders to stress by acting within seconds:

A. Aldosterone
B. Catecholamine
C. Cortisol
D. Estrogen

A

B. Catecholamine

Medullary catecholamine products serve as firstresponders to stress by acting within seconds (cortisol takes 20 min) to promote the fight-or-fl ight response, which increases cardiac output and blood pressure, diverts blood toward muscle and brain, and mobilizes fuel from storage.

339
Q

Which of the following is the mechanism causing Cushing’s disease?

A. Excess secretion of pituitary ACTH
B. Adrenal adenoma
C. Treatment with corticosteroids
D. Ectopic ACTH production by tumors

A

A. Excess secretion of pituitary ACTH

Cushing’s disease refers to adrenal hyperplasia resulting from misregulation of the hypothalamic–pituitary axis. It is usually caused by small pituitary adenomas. Cushing’s syndrome may be caused by Cushing’s disease, adrenal adenoma or carcinoma, ectopic ACTH-producing tumors, or excessive corticosteroid administration. The cause of Cushing’s syndrome can be differentiated using the ACTH and dexamethasone suppression tests.

340
Q

Which test is used to distinguish Cushing’s disease (pituitary Cushing’s) from Cushing’s syndrome caused by adrenal tumors?

A. Low-dose overnight dexamethasone suppression
B. Petrosal sinus sampling
C. Serum ACTH
D. Twenty-four–hour urinary free cortisol

A

C. Serum ACTH

Serum ACTH assays are very helpful in distinguishing the cause of Cushing’s syndrome. Patients with adrenal tumors have values approaching zero. Patients with ectopic ACTH tumors have values greater than 200 pg/dL. Fifty percent of patients with Cushing’s disease have high 8 a.m. ACTH levels (between 100–200 pg/dL). The high-dose dexamethasone suppression test is also used.

341
Q

Which is the most widely used screening test for Cushing’s syndrome?

A. Overnight low-dose dexamethasone suppression test
B. Corticotropin-releasing hormone stimulation test
C. Petrosal sinus sampling
D. Metyrapone stimulation test

A

A. Overnight low-dose dexamethasone suppression test

342
Q

The parent substance in the biosynthesis of androgens and estrogens is:

A. Cholesterol
B. Cortisol
Catecholamines
Progesterone

A

A. Cholesterol

343
Q

Select the main estrogen produced by the ovaries and used to evaluate ovarian function.

A. Estriol (E3 )
B. Estradiol (E2)
C. Epiestriol
D. Hydroxyestrone

A

B. Estradiol (E2)

344
Q

The biologically most active, naturally occurring androgen is:

A. DHEA
B. Androstenedione
C. Epiandrosterone
D. Testosterone

A

D. Testosterone

345
Q

Zollinger–Ellison (Z–E) syndrome is characterized by great (e.g., 20-fold) elevation of:

A. Gastrin
B. Cholecystokinin
C. Pepsin
D. Glucagon

A

A. Gastrin

346
Q

Which of the following conditions can be quantified using a measurement technique known as the Ferriman-Gallwey Scale?

A. Acromegaly
B. Cushing’s syndrome
C. Hirsutism
D. PCOS

A

C. Hirsutism

347
Q

It is usually associated with a single, short-term exposure to a substance, the dose of which is sufficient to cause immediate toxic effects:

A. Acute toxicity
B. Chronic toxicity

A

A. Acute toxicity

348
Q

It is usually associated with repeated frequent exposure for extended periods for greater than 3 months and possibly years, at doses that are insufficient to cause an immediate acute response:

A. Acute toxicity
B. Chronic toxicity

A

B. Chronic toxicity

349
Q

Levels of 8-9% carboxyhemoglobin saturation of whole blood are commonly found in which of the following situations?

A. Fatal carbon monoxide poisoning
B. Acute carbon monoxide poisoning
C. Nonsmoking residents of rural areas
D. Cigarette smokers

A

D. Cigarette smokers

Cigarette smokers exhibit levels of 8-9% carboxyhemoglobin, but occasionally saturations of greater than 16% have been reported in heavy smokers.

350
Q

Heroin is synthesized from what drug?

A. Diazepam
B. Morphine
C. Ecgonine
D. Chlorpromazine

A

B. Morphine

Heroin (diacetylmorphine), an abused drug, is a derivative of morphine. The morphine used in its synthesis is generally obtained from opium.

351
Q

THC (tetrahydrocannabinol) is the principal active component of what
drug?

A. Benzodiazepine
B. Marijuana
C. Morphine
D. Codeine

A

B. Marijuana

THC (tetrahydrocannabinol) is the principal active component of marijuana.

352
Q

Which substance has the longest detection time?

A. Amphetamines
B. Cocaine
C. Benzodiazepines
D. Marijuana

A

D. Marijuana

Marijuana is stored in fatty tissue and is metabolized slowly.
In persons who use marijuana several times per week, cannabinoids can be detected several weeks after last use.
For chronic daily users, this extends to months after discontinuation

353
Q

Methylenedioxymethylamphetamine (MDMA) is an illicit amphetamine
derivative that is commonly referred to as:

A. Angel dust
B. Ecstacy
C. Marijuana
D. Shabu

A

B. Ecstacy

354
Q

The half-life of the circulating cocaine:

A. 0.5 to 1 hour
B. 1 to 2 hours
C. 2 to 3 hours
D. 3 to 4 hours

A

A. 0.5 to 1 hour

The half-life of the circulating cocaine is brief: 0.5 to 1 hour. BISHOP

355
Q

Identification of the urinary metabolite benzoylecgonine would be useful in determining exposure to which of the following drugs?

A. Codeine
B. Cocaine
C. Amphetamine
D. Propoxyphene

A

B. Cocaine

Cocaine is an abused drug and not available for therapeutic use. After absorption, cocaine in the blood is rapidly converted into ecgonine and benzoylecgonine.

356
Q

All of the following are CNS depressant, EXCEPT:

A. Barbiturates
B. Benzodiazepines
C. Cocaine
D. Methaqualone

A

C. Cocaine

COCAINE IS A CNS STIMULANT.

CNS STIMULANTS
Cocaine and its metabolite, benzoylecgonine, and amphetamines and methamphetamines

CNS DEPRESSANTS
Barbiturates; methaqualone; benzodiazepines including Valium; and oxycodone and other
opiates, including morphine, heroin (which metabolizes to morphine), codeine
(methylmorphine), and methadone

HALLUCINOGENS OR PSYCHOACTIVES
Cannabinoids and phencyclidine (PCP)

ANTIDEPRESSANTS
Lithium, tricyclic antidepressants

357
Q

Of the following specimens, which would be appropriate for determining exposure to lead?

A. EDTA plasma
B. Serum
C. Whole blood
D. Cerebrospinal fluid

A

C. Whole blood

After absorption, lead is distributed into an active pool in the blood and soft tissue and a storage pool in bone, teeth, and hair. In blood, the majority is found in erythrocytes, with only minor quantities in plasma or serum. Lead is mainly excreted by the kidney; hence urine or whole blood would be appropriate specimens for determining lead exposure.

358
Q

This toxin has high affinity to keratin, can be identified from hair and nails:

A. Arsenic
B. Cyanide
C. Lead
D. Mercury

A

A. Arsenic

ARSENIC
Toxins may BIND SULFHYDRYL GROUPS IN KERATIN FOUND IN HAIR AND FINGERNAILS

359
Q

Clues include the ODOR OF BITTER ALMONDS, the occurrence of an altered mental status and tachypnea in the absence of cyanosis, and an unexplained metabolic acidosis:

A. Arsenic toxicity
B. Carbon monoxide intoxication
C. Cyanide overdose
D. Iron poisoning

A

C. Cyanide overdose

The principal symptoms of cyanide overdose are tachypnea (initially), followed by respiratory depression and cyanosis, hypotension, convulsions, and coma. Death may occur in a matter of minutes because cyanide is a fast-acting toxin. Diagnosis may be difficult, and a high index of suspicion is needed to make the correct diagnosis. Clues include the odor of bitter almonds, the occurrence of an altered mental status and tachypnea in the absence of cyanosis, and an unexplained metabolic acidosis (with an increased anion gap).

360
Q

The ODOR OF GARLIC may be on the breath, and a METALLIC TASTE in the patient’s mouth:

A. Arsenic toxicity
B. Carbon monoxide intoxication
C. Cyanide overdose
D. Iron poisoning

A

A. Arsenic toxicity

ODOR OF BITTER ALMONDS: CYANIDE POISONING
ODOR OF GARLIC, METALLIC TASTE: ARSENIC POISONING

361
Q

Most common drug of abuse:

A. Cocaine
B. Ethanol
C. Methanol
D. Marijuana

A

B. Ethanol

Ethanol is probably the most common drug of abuse and is frequently responsible for the presentation of patients with altered mental status to hospitals and emergency rooms.

362
Q

Select the five pharmacological parameters that determine serum drug concentration.

A. Absorption, anabolism, perfusion, bioactivation, excretion
B. Liberation, equilibration, biotransformation, reabsorption, elimination
C. Liberation, absorption, distribution, metabolism, excretion
D. Ingestion, conjugation, integration, metabolism, elimination

A

C. Liberation, absorption, distribution, metabolism, excretion

LADME: Liberation, absorption, distribution, metabolism and excretion
1. Liberation is the release of the drug
2. Absorption is the transport of drug from the site of administration to the blood
3. Distribution refers to the delivery of the drug to the tissues
4. Metabolism is the process of chemical modifi cation of the drug by cells
5. Excretion is the process by which the drug and its metabolites are removed from the body

363
Q

Blood sample collection time for peak drug levels:

A. Varies with the drug, depending on its rate of absorption
B. Is independent of drug formulation
C. Is independent of the route of administration
D. Is 30 minutes after a bolus intravenous injection is completed

A

A. Varies with the drug, depending on its rate of absorption

The peak concentration of a drug is the highest concentration obtained in the dosing
interval.
For oral drugs, the time of peak concentration is dependent upon their rates of absorption and elimination and is determined by serial blood measurements. Peak levels for oral drugs are usually drawn 1–2 hours after administration of the dose. For drugs given intravenously, peak levels are measured immediately after the infusion is completed.

364
Q

When is a blood sample for determination of the trough level of a drug appropriately drawn?

A. During the absorption phase of the drug
B. During the distribution phase of the drug
C. Shortly before drug administration
D. Two hours after drug administration

A

C. Shortly before drug administration

When peak levels of the drug are required, the blood sample must be drawn at a specified time after drug administration.
Trough levels are most reliably determined by collecting the blood sample before the next drug administration.

365
Q

Which route of administration is associated with 100% bioavailability?

A. Sublingual
B. Intramuscular
C. Oral
D. Intravenous

A

D. Intravenous

When a drug is administered intravenously, all the drug enters the bloodstream.

366
Q

For what colorimetric determination is the Trinder reaction widely used?

A. Acetaminophen
B. Propoxyphene
C. Salicylate
D. Barbiturate

A

C. Salicylate

The Trinder reaction or modifi cation is used almost routinely in the determination of salicylate and is based on the colorimetric reaction with ferric ions.

367
Q

Acetaminophen is particularly toxic to what organ?

A. Heart
B. Kidney
C. Spleen
D. Liver

A

D. Liver

Paracetamol, also known as acetaminophen.
Hepatotoxicity is common in acetaminophen overdose. It is particularly important to be able to determine the acetaminophen serum level rapidly so that the elimination half-life of the drug can be estimated. Hepatic necrosis is more common when the half-life exceeds 4 hours and is very likely when it exceeds 12 hours.

368
Q

Increased trough levels of aminoglycosides in the serum are often associated with toxic effects to which organ?

A. Heart
B. Kidney
C. Pancreas
D. Liver

A

B. Kidney

Tobramycin and gentamicin are examples of aminoglycoside antibiotics. Their use has been associated with both nephrotoxicity and ototoxicity.

369
Q

Which of the following drugs is used as an immunosuppressant in organ transplantation, especially in liver transplants?

A. Methotrexate
B. Amiodarone
C. Tacrolimus
D. Paroxetine

A

C. Tacrolimus

Tacrolimus (Prograf) is an antibiotic that functions as an immunosuppressant in organ transplantation, especially in liver transplants.

370
Q

Which of the following drugs is used as a bronchodilator?

A. Theophylline
B. Phenytoin
C. Amikacin
D. Clozapine

A

A. Theophylline

Theophylline is a bronchodilator that is used to treat asthma. The therapeutic range is 10- 20 ug/mL, and use must be monitored to avoid toxicity. Use of theophylline has been replaced where possible with beta-adrenergic agonists, which are available in the inhaled form.

371
Q

Bronchodilators:
1.Digoxin
2.Phencyclidine
3.Theophylline
4.Theobromine

A. 1 and 2
B. 1 and 3
C. 2 and 4
D. 3 and 4

A

D. 3 and 4

Anti-asthmatic drugs, such as THEOPHYLLINE and THEOBROMINE, are used for treatment of neonatal breathing disorders or of respiratory conditions that affect adults or children, such as asthma.

372
Q

Which of the following is a commonly encountered xanthine that could potentially interfere with the determination of theophylline?

A. Nicotine
B. Caffeine
C. Amphetamine
D. Procainamide

A

B. Caffeine

Theophylline, a xanthine with bronchodilator activity, is widely used in the treatment of asthma. Because of its availability and potential toxicity, it can also be subject to accidental overdose. Chromatographic methods are effective in separating theophylline from caffeine and theobromine, which are two commonly occurring and potentially interfering xanthines.
However, most clinical thin-layer chromatographic methods are relatively insensitive to the xanthines, and suspected theophylline overdose should be confi rmed by HPLC or immunoassay methods.

373
Q

Which of the following is used in the treatment of manic depression?

A. Potassium
B. Lithium
C. Calcium
D. Chloride

A

B. Lithium

Lithium is used in the treatment of manic depression. Because of the small difference between therapeutic and toxic levels in the serum, accurate measurements of lithium concentrations are essential.

374
Q

The major toxicities of ____ are red man syndrome, nephrotoxicity, and ototoxicity.

A. Aminoglycosides
B. Cephalosporin
C. Penicillin
D. Vancomycin

A

D. Vancomycin

The major toxicities of vancomycin are red man syndrome, nephrotoxicity, and ototoxicity.
Red man syndrome is characterized by an erythemic flushing of the extremities.
The renal and hearing effects are similar to those of the aminoglycosides.

375
Q

The drug of choice for absence (petit mal) seizures unaccompanied by other types of seizures:

A. Carbamazepine (Tegretol)
B. Ethosuximide (Zarontin)
C. Primidone (Mysoline)
D. Valproic Acid (Depakene)

A

B. Ethosuximide (Zarontin)

HENRY:
Ethosuximide is the drug of choice for absence (petit mal) seizures unaccompanied by other types of seizures. It is preferred over valproic acid, at least initially, because hepatotoxicity is a rare but serious side effect of valproic acid.

376
Q

Which of the following do not require TDM?
1.Salicylates
2.Acetaminophen
3.Ibuprofen

A. 1 and 2
B. 1 and 3
C. 2 and 3
D. 1, 2 and 3

A

D. 1, 2 and 3

Most analgesics, such as salicylates, acetaminophen, and ibuprofen, do not require TDM because physicians and pharmacists are able to achieve and maintain therapeutic levels with standardized dosing intervals.
Because of the presumed safety of these medications, they are available without a prescription and sold as OVER-THE-COUNTER medications.

377
Q

Factor to convert immunoglobulin value from mg/dL to g/L:

A. 0.01
B. 0.02586
C. 0.05551
D. 10

A

A. 0.01

Immunoglobulin conversion factors:
mg/dL to g/L: 0.01
mg/dL to mg/L: 10

378
Q

Heparin is added ____ per mL of blood in each test tube.

A. 0.2 mcg per mL
B. 2.0 mcg per mL
C. 0.2 mg per mL
D. 2.0 mg per mL

A

C. 0.2 mg per mL

379
Q

Detection and quantification of the separated protein after electrophoresis is
accomplished by:

A. Amperometry
B. Chromatography
C. Coulometry
D. Densitometry

A

D. Densitometry

380
Q

It transports protein for thyroxine and triiodothyronine (thyroid hormones); it
also binds with retinol-binding protein to form a complex that transports
retinol (vitamin A):

A. Albumin
B. Haptoglobin
C. Orosomucoid
D. Prealbumin

A

D. Prealbumin

381
Q

A low ____ level is a sensitive marker of poor nutritional status.

A. Ceruloplasmin
B. Hemopexin
C. Prealbumin
D. Transferrin

A

C. Prealbumin

382
Q

Major roles of ____ is to maintain the equilibrium of cholesterol in peripheral
cells by the REVERSE CHOLESTEROL TRANSPORT pathway:

A. Chylomicrons
B. LDL
C. VLDL
D. HDL

A

D. HDL

383
Q

Which of the following blood samples would serve best to assay lipoproteins
because this anticoagulant acts to preserve lipoproteins?

A. EDTA plasma sample
B. Heparin plasma sample
C. Citrate plasma sample
D. Fluoride plasma sample

A

A. EDTA plasma sample

Ethylenediaminetetraacetic acid (EDTA) (1 mg/1 mL blood) plasma preferred for analysis
of lipoproteins (EDTA preserves lipoproteins); separate plasma within 2 hours.

384
Q

Abetalipoproteinemia is also known as:

A. Anderson’s disease
B. Bassen-Kornzweig syndrome
C. Sitosterolemia
D. Tangier disease

A

B. Bassen-Kornzweig syndrome

385
Q

If left unprotected from light, bilirubin values may reduce by ____ per hour.

A. 10 to 30%
B. 30 to 50%
C. 50 to 70%
D. 70 to 90%

A

B. 30 to 50%

386
Q

Use of wetting agents or incorrect pH buffers ____ the amount of
unconjugated bilirubin measured as direct bilirubin.

A. Decreases unconjugated bilirubin measured as direct bilirubin
B. Increases unconjugated bilirubin measured as direct bilirubin
C. Indeterminate
D. No effect

A

B. Increases unconjugated bilirubin measured as direct bilirubin

387
Q

ALP isoenzyme that will resist heat denaturation at 65°C for 30 minutes:

A. Bone ALP
B. Intestinal ALP
C. Liver ALP
D. Placental ALP

A

D. Placental ALP

388
Q

Given the following results: ALP: marked increased; AST: slight increased; ALT:
slight increased; and GGT: marked increased. This is most consistent with:

A. Acute hepatitis
B. Osteitis fibrosa
C. Chronic hepatitis
D. Obstructive jaundice

A

D. Obstructive jaundice

389
Q

A physician orders several laboratory tests on a 55-year-old male patient who is complaining of pain, stiffness, fatigue and headaches. Based on the
following serum test results, what is the most likely diagnosis? ALP: significantly increased; GGT: normal

A. Biliary obstruction
B. Cirrhosis
C. Hepatitis
D. Osteitis deformans (Paget disease)

A

D. Osteitis deformans (Paget disease)

390
Q

A 42-year-old male presents with anorexia, nausea, fever, and icterus of the
skin and mucous membranes. He noticed that his urine had appeared dark for
the past several days. The physician orders a series of biochemical tests.
Based on the following test results, what is the most likely diagnosis? ALP:
slightly elevated; ALT: markedly elevated; AST: markedly elevated; GGT: slightly
elevated; Serum total bilirubin: moderately elevated; Urine bilirubin: positive;
and Fecal urobilinogen: decreased

A. Acute hepatitis
B. Alcoholic cirrhosis
C. Metastatic carcinoma of the pancreas
D. Obstructive jaundice

A

A. Acute hepatitis

391
Q

Assay methods for AST are generally based on the principle of the Karmen
method, which incorporates a coupled enzymatic reaction using:

A. Glutamic oxaloacetic transaminase
B. Glutamic pyruvic transaminase
C. Lactate dehydrogenase
D. Malate dehydrogenase

A

D. Malate dehydrogenase

392
Q

The typical assay procedure for ALT consists of a coupled enzymatic reaction
using ____ as the indicator enzyme.

A. Glutamic oxaloacetic transaminase
B. Glutamic pyruvic transaminase
C. Lactate dehydrogenase
D. Malate dehydrogenase

A

C. Lactate dehydrogenase

393
Q

Calcium is essential for functional integrity of this enzyme:

A. Alkaline phosphatase
B. Amylase
C. Cholinesterase
D. Lipase

A

B. Amylase

394
Q

LOW LEVELS of maternal AFP indicate an increased risk for:

A. Anencephaly
B. Down syndrome
C. Presence of twins
D. Spina bifida

A

B. Down syndrome

Conditions associated with an elevated AFP level include spina bifi da, neural tube defects,
abdominal wall defects, anencephaly (absence of the major portion of the brain), and
general fetal distress.
Low levels of maternal AFP indicate an increased risk for Down syndrome and trisomy 18,
while it is increased in the presence of twins and neural tube defects.

395
Q

Which set of results is consistent with uncompensated metabolic acidosis?

A. pH 7.25 HCO3- 15 mmol/L pCO2 37 mm Hg
B. pH 7.30 HCO3- 16 mmol/L pCO2 28 mm Hg
C. pH 7.45 HCO3- 22 mmol/L pCO2 40 mm Hg
D. pH 7.40 HCO3- 25 mmol/L pCO2 40 mm Hg

A

A. pH 7.25 HCO3- 15 mmol/L pCO2 37 mm Hg

TIPS FOR EVALUATING ACID-BASE DISORDERS
1. Look at the pH: determine if acidosis or alkalosis
2. Compare pCO2 and HCO3-
pCO2 going opposite to pH – RESPIRATORY
Abnormal pCO2 respiratory
↓pH ↑pCO2 respiratory acidosis
↑pH ↓pCO2 respiratory alkalosis
HCO3- going same direction as pH - METABOLIC
Abnormal HCO3- metabolic
↓pH ↓HCO3- metabolic acidosis
↑ PH ↑ HCO3- METABOLIC ALKALOSIS
3. If pH is normal, full compensation occurred
4. If main compensatory mechanism kicked in, but pH still out of normal range, partial
compensation has occurred

396
Q

Hirsutism, which can be quantified using a measurement technique known as
the:

A. Ferriman-Gallwey scale
B. Liley graph
C. T-score
D. Z-score

A

A. Ferriman-Gallwey scale

397
Q

Decreased T3 and T4, increased TSH:

A. Primary hypothyroidism
B. Secondary hypothyroidism
C. Primary hyperthyroidism
D. Secondary hyperthyroidism

A

A. Primary hypothyroidism

398
Q

Fluorophore-labeled thyroxine competes with patient thyroxine for antibody in
homogeneous system. Antibody-bound labeled thyroxine rotates slowly,
emitting lower energy light.

A. Fluorescent polarization immunoassay (FPIA)
B. Fluorescent substrate-labeled inhibition immunoassay
C. Chemiluminescence
D. Microparticle enzyme immunoassay (MEIA)

A

A. Fluorescent polarization immunoassay (FPIA)

399
Q

Fluorogenic substrate–labeled thyroxine competing with patient T4 for
antibody in a homogeneous assay. Only unbound, leftover labeled T4 reacts
with enzyme to form fluorescent product.

A. Fluorescent polarization immunoassay (FPIA)
B. Fluorescent substrate-labeled inhibition immunoassay
C. Chemiluminescence
D. Microparticle enzyme immunoassay (MEIA)

A

B. Fluorescent substrate-labeled inhibition immunoassay

400
Q

The half-life of the circulating cocaine is: (Bishop)

0.5 to 1 hour
1 to 2 hours
2 to 3 hours
3 to 4 hours

A

0.5 to 1 hour

401
Q

Water that is suitable for human consumption (i.e., water that can be used for
drinking or cooking):

A. Boiled water
B. Distilled water
C. Filtered water
D. Potable water

A

D. Potable water

402
Q

Non-alcoholic:

A. Fruit brandy
B. Root beer
C. Soju
D. Wine

A

B. Root beer

403
Q

In the laboratory, a program that monitors the TOTAL TESTING PROCESS with
the aim of providing the highest quality patient care:

A. Quality assessment/assurance (QA)
B. Quality control (QC)
C. Quality systems (QS)
D. None of these

A

A. Quality assessment/assurance (QA)

404
Q

In an institution, a COMPREHENSIVE PROGRAM in which all areas of operation
are monitored to ensure quality with the aim of providing the highest quality
patient care:

A. Quality assessment
B. Quality assurance
C. Quality control
D. Quality systems

A

D. Quality systems

405
Q

The methodology for a Lean Six Sigma quality improvement team will include
consideration of all of the following factors EXCEPT:

A. Define
B. Measure
C. Analyze
D. Improve
E. Communicate

A

E. Communicate

406
Q

Project team members:

A. Black belts
B. Green belts
C. Blue belts
D. White belts

A

B. Green belts

407
Q

Green belts contribute ____ of their time to improvement projects while
delivering their normal job functions.

A. 20%
B. 40%
C. 60%
D. 100%

A

A. 20%

408
Q

Most effective at reducing hazards:

A. PPE
B. Administrative controls
C. Engineering controls
D. Substitution
E. Elimination

A

E. Elimination

409
Q

In the hierarchy of controls, arrange the following from the least effective to
the most effective:

A. PPE, engineering controls, administrative controls, substitution, elimination
B. PPE, administrative controls, engineering controls, substitution, elimination
C. Administrative controls, engineering controls, PPE, elimination, substitution
D. Administrative controls, engineering controls, elimination, substitution, PPE

A

B. PPE, administrative controls, engineering controls, substitution, elimination

410
Q

Levels may become elevated as one changes position from supine to upright:

ACTH and cortisol
Aldosterone and insulin
Growth hormone and ACP
Albumin and calcium

A

Albumin and calcium

An upright position increases hydrostatic pressure, causing a reduction of plasma volume
and increased concentration of proteins. Albumin and calcium levels may become
elevated as one changes position from supine to upright.
Elements that are affected by postural changes are albumin, total protein, enzymes,
calcium, bilirubin, cholesterol, triglycerides, and drugs bound to proteins.

411
Q

Peaks early to late morning; decreases up to 30% during the day:

Growth hormone
Acid phosphatase
Calcium
Iron

A

Iron

Cortisol: Peaks 4-6 AM; lowest 8 PM–12 AM; 50% lower at 8 PM than at 8 AM; increased with stress

Adrenocorticotropic hormone: Lower at night; increased with stress

Plasma renin activity: Lower at night; higher standing than supine

Aldosterone: Lower at night
Insulin: Lower at night

Growth hormone: Higher in afternoon and evening
Acid phosphatase: Higher in afternoon and evening

Thyroxine: Increases with exercise

Prolactin: Higher with stress; higher levels at 4 and 8 AM and at 8 and 10 PM

Iron: Peaks early to late morning; decreases up to 30% during the day

Calcium: 4% decrease supine

412
Q

Methods used to measure the concentrations of large particles such as antigen–antibody complexes, prealbumin, and other serum proteins:

Nephelometry
Turbidimetry
Nepholometry and turbidimetry
Nephelometry, turbidimetry and absorption spectroscopy

A

Nepholometry and turbidimetry

Nephelometry and turbidimetry are used to measure the concentrations of large particles
(such as antigen–antibody complexes, prealbumin, and other serum proteins) that because of their size cannot be measured by absorption spectroscopy.

413
Q

The measurement of voltage between two electrodes in a solution forms
the basis for a variety of procedures for measuring analyte concentration:

Potentiometry
Coulometry
Amperometry
Voltammetry

A

Potentiometry

Potentiometry
The measurement of potential (voltage) between two electrodes in a solution forms the
basis for a variety of procedures for measuring analyte concentration.

414
Q

It is based on fragmentation and ionization of molecules using a suitable
source of energy:

Conductance
Impedance
Chromatography
Mass spectrometry

A

Mass spectrometry

415
Q

Generally it is used to detect gamma radiation:

Crystal scintillation counter
Liquid scintillation counter
Crystal and liquid scintillation counter
None of these

A

Crystal scintillation counter

Crystal scintillation generally is used to detect gamma radiation. When a gamma ray
penetrates the sodium iodide (NaI) crystal, which contains 1% thallium, it excites the
electrons of iodide atoms and raises them to higher energy states.
Liquid scintillation is primarily used to count radionuclides that emit beta particles.

416
Q

A technique for determining the structure of organic compounds; it is
nondestructive, although it does require a larger sample volume:

Mass spectroscopy
Nuclear magnetic resonanance
Capillary electrophoresis
Mass spectroscopy and nuclear magnetic resonance

A

Nuclear magnetic resonanance

Nuclear magnetic resonance spectroscopy (NMR) is a technique for determining the
structure of organic compounds. Unlike mass spectroscopy (MS), NMR is nondestructive,
although it does require a larger sample volume than MS. Although NMR is widely used as
a diagnostic imaging technique, it has been adapted for only a limited number of clinical
laboratory analyses, the most popular being lipoprotein particle measurements. It also has
the unique capability of performing chemical analysis in vivo.

417
Q

Common causes of hypernatremia, EXCEPT:

Dehydration
Diabetes insipidus
Cushing’s disease or syndrome
Syndrome of inappropriate ADH (SIADH) secretion

A

Syndrome of inappropriate ADH (SIADH) secretion

HYPONATREMIA: Syndrome of Inappropriate ADH (SIADH) Secretion
In this condition, secondary to head trauma, seizures, other CNS diseases, and neoplastic
conditions, especially lung, breast, and ovarian cancers that secrete ADH-like hormones,
the serum sodium is depressed due to the excess retention of water in the collecting
ducts.

418
Q

In all forms of hyponatremia, the chloride ion concentration is also
generally ____ because chloride is the chief counterion for sodium.

High
Low
Variable
Cannot be determined

A

Low

In all forms of hyponatremia, the chloride ion concentration is also generally low because
chloride is the chief counterion for sodium.

419
Q

Low anion gaps:

Uremia/renal failure
Uremia/renal failure, ketoacidosis and salicylate poisoning
Hypoalbuminemia and severe hypercalcemia
Hypoalbuminemia, hypercalcemia and multiple myeloma

A

Hypoalbuminemia, hypercalcemia and multiple myeloma

Low AG values are rare but may be seen with hypoalbuminemia (decrease in unmeasured
anions) or severe hypercalcemia (increase
in unmeasured cations).

HENRY: Persistently low anion gaps are a serious sign of possible malignancy—for
example, multiple myeloma.

420
Q

In panhepatic cirrhosis there is destruction of _____ of liver tissue.

Less than 50%
More than 50%
Less than 80%
More than 80%

A

More than 80%

Because in panhepatic cirrhosis there is destruction of more than 80% of liver tissue, with
no regeneration of damaged liver tissue, the AST/ALT aminotransferases and LD levels (all
from the regenerating nodules) tend to be normal or low, or occasionally mildly elevated. However, the total protein and albumin are both abnormally low.

In hepatitis, much less than 80% of the liver is destroyed, total regeneration will occur, and
enough tissue is present to enable adequate levels of protein synthesis and ammonia
fixation as urea. Therefore, the total protein and albumin and ammonia levels remain
normal.

421
Q

A definitive test for congestive heart failure and appears to be an
excellent marker for early heart failure:

CK and AST
CK, AST and LD
CK-MB and troponin
B-type natriuretic peptide

A

B-type natriuretic peptide

Diagnosis of Congestive Heart Failure
Until recently, this condition was diagnosed strictly on the basis of symptomatology
and/or as a result of procedures such as echocardiography, but more recently a biomarker
for this condition is the brain form or B-type natriuretic peptide (BNP), which has been
approved as a definitive test for this condition and appears to be an excellent marker for
early heart failure.

422
Q

The main waste product of nitrogen-containing chemicals in the body:

Ammonia
Creatinine
Urea
Uric acid

A

Urea

Urea is the main waste product of nitrogen-containing chemicals in the body.

423
Q

This protein appears in the urine when reabsorption is incomplete
because of the proximal tubular damage as in acute kidney injury:

Urea
Creatinine
Alpha2-macroglobulin
Beta2-microglobulin

A

Beta2-microglobulin

Beta-2 microglobulin, a polypeptide with a molecular weight of 11.6 kDa with a length of
99 amino acids, is a component of the MHC (major histocompatibility complex) class I
molecule.
The protein appears in the urine when reabsorption is incomplete because of the proximal
tubular damage as in acute kidney injury.

424
Q

It is characterized by a sudden onset of hematuria and proteinuria and a
decrease in glomerular filtration rate characterized by a rise in plasma
creatinine and a fall in creatinine clearance compared with reference
ranges:

Acute glomerular nephritis
Chronic glomerular nephritis
Diabetic nephropathy
Systemic luus erythematosus

A

Acute glomerular nephritis

425
Q

Chronic glomerular nephritis

-Slower developing disease and may be idiopathic
-Characterized by gradual uremia
-Loss of functioning nephrons
-Slower developing disease and may be idiopathic, and is characterized by gradual uremia and loss of functioning nephrons

A

Slower developing disease and may be idiopathic, and is characterized by gradual uremia and loss of functioning nephrons

Chronic glomerular nephritis is a slower developing disease and may be idiopathic, and is
characterized by gradual uremia and loss of functioning nephrons.

426
Q

A substance that increases the concentration of hydrogen ion (H+) when
dissolved in water:

Acid
Base
Neutral
Buffer

A

Acid

427
Q

A substance that increases the concentration of hydroxyl ion (OH–) when
dissolved in water:

Acid
Base
Neutral
Buffer

A

Base

428
Q

The combination of a weak acid or weak base and its salt, is a system that
resists changes in pH:

Acid
Base
Neutral
Buffer

A

Buffer

A buffer, the combination of a weak acid or weak base and its salt, is a system that resists
changes in pH. The effectiveness of a buffer depends on the pKa of the buffering system
and the pH of the environment in which it is placed. In plasma, the bicarbonate–carbonic
acid system, having a pKa of 6.1, is one of the principal buffers.

429
Q

The role of the lungs and kidneys in maintaining pH is depicted with the
Henderson-Hasselbalch equation. The numerator denotes:

Kidney function
Lung function
Either kidney or lung function
None of these

A

Kidney function

The role of the lungs and kidneys in maintaining pH is depicted with the HendersonHasselbalch equation. The numerator (HCO3−) denotes kidney functions, and the
denominator (Pco2) denotes lung function.

430
Q

The role of the lungs and kidneys in maintaining pH is depicted with the
Henderson-Hasselbalch equation. The denominator denotes:

Kidney function
Lung function
Either kidney or lung function
None of these

A

Lung function

The role of the lungs and kidneys in maintaining pH is depicted with the HendersonHasselbalch equation. The numerator (HCO3−) denotes kidney functions, and the
denominator (Pco2) denotes lung function.

431
Q

Lung diseases such as chronic obstructive lung disease, advanced
interstitial lung disease and acute asthma are causes of:

Respiratory acidosis
Respiratory alkalosis
Metabolic acidosis
Metabolic alkalosis

A

Respiratory acidosis

432
Q

The fifth most common element and is the most prevalent cation in the
human body:

Calcium
Magnesium
Potassium
Sodium

A

Calcium

Calcium is the fifth most common element and is the most prevalent cation in the human
body.
A healthy adult contains approximately 1 to 1.3 kg of calcium, and 99% of this is in the
form of hydroxyapatite in the skeleton. The remaining 1% is contained in the extracellular
fluid (ECF) and soft tissues. Additionally, less than 1% of the skeletal content of calcium is
in bone fluid and exchanges freely with the ECF.

433
Q

The fourth most abundant cation in the body and the second most
prevalent intracellular cation:

Calcium
Magnesium
Potassium
Sodium

A

Magnesium

434
Q

There usually is an autoimmune destruction of insulin-producing beta
cells in the islets of the pancreas, causing an absolute deficiency in
insulin production.

Type 1 diabetes mellitus
Type 2 diabetes mellitus
Type 1 and 2 diabetes
Gestational diabetes

A

Type 1 diabetes mellitus

435
Q

Most widely used to assess short-term(3- to 6-week) glycemic control:

Plasma glucose levels
Glycosylated hemoglobin
Fructosamine
Ketone testing

A

Fructosamine

Fructosamine assays are the most widely used to assess short-term (3- to 6-week)
glycemic control because the average half-life of the proteins is 2 to 3 weeks

436
Q

It plays a key role in role in reverse cholesterol transport, the process by
which excess cholesterol is returned from tissues to the liver, where it is
reused or excreted in bile:

Chylomicrons
VLDL
LDL
HDL

A

HDL

HDL plays a key role in reverse cholesterol transport, the process by which excess
cholesterol is returned from tissues to the liver, where it is reused or excreted in bile.

437
Q

An abnormal lipoprotein found in patients with obstructive biliary disease
and in patients with familial lecithin/cholesterol acyltransferase (LCAT)
deficiency:

Intermediate-density lipoproteins
Lipoprotein (a)
LpX lipoprotein
LDL

A

LpX lipoprotein

LpX is an abnormal lipoprotein found in patients with obstructive biliary disease and in
patients with familial lecithin/cholesterol acyltransferase (LCAT) deficiency.

438
Q

Lipoprotein(s) have a density of less than 1.006 kg/L (density
measurement):

HDL
LDL
LDL and HDL
Chylomicrons and VLDL

A

Chylomicrons and VLDL

VLDL and chylomicrons are the most lipid-rich lipoprotein classes in human blood. This
fact leaves them as the most buoyant in plasma. These lipoproteins have a density of less
than 1.006 kg/L (density measurement).
LDL particles are smaller in size and in lipid content, causing their density to range from
1.006 to 1.063 kg/L.
HDL, the densest lipoprotein, ranges from 1.063 to 1.210 kg/L.

439
Q

A rare autosomal recessive disorder characterized by complete absence of HDL:

LCAT deficiency
Hepatic lipase deficiency
Familial hypoalphalipoproteinemia
Tangier disease

A

Tangier disease

Tangier disease is a rare autosomal recessive disorder characterized by complete absence of HDL due to a mutation in the ABCA1 gene on chromosome 9.
In the homozygous state, patients present with low or undetectable HDL in plasma,
hepatosplenomegaly, peripheral neuropathy, orange tonsils, and premature coronary
disease.

440
Q

It plays a significant role in the metabolism of vitamin A by complexing
with the retinol-binding protein (RBP), which, in turn, complexes with
vitamin A to transport it through the body:

Albumin
Alpha1-antitrypsin
Prealbumin
Transferrin

A

Prealbumin

Prealbumin plays a significant role in the metabolism of vitamin A by complexing with the
retinol-binding protein (RBP), which, in turn, complexes with vitamin A to transport it
through the body.

441
Q

This protein, also known as orosomucoid, has a very high carbohydrate
content, which minimizes its visualization by standard protein stains:

Gc-globulin
Hemopexin
C-reactive protein
Alpha1-acid glycoprotein

A

Alpha1-acid glycoprotein

442
Q

A striking elevation of transferrin in the β-region sometimes occurs in
patients suffering from:

Acute inflammation
Chronic inflammation
Nephrotic syndrome
Iron deficiency anemia

A

Iron deficiency anemia

A striking elevation of transferrin in the β-region sometimes occurs in patients suffering
from iron deficiency anemia. The increase in transferrin corresponds to increased IBC, and
the percent saturation is low.

443
Q

Enzymes _____ the activation energies of the chemical reactions that they
catalyze, so as to cause greatly enhanced rates of reaction.

Elevate
Lower
Affect variably
Produces no effect

A

Lower

Enzymes lower the activation energies of the chemical reactions that they catalyze, so as
to cause greatly enhanced rates of reaction. They do not become modified in these
reactions and do not affect the equilibrium between reactants and products in the
reaction.

444
Q

Different forms of enzymes that catalyze the same reaction:

Activators
Coenzymes
Isoenzymes
Substrates

A

Isoenzymes

Many enzymes have isoenzymes, called isozymes, of different forms that catalyze the
same reaction. These different forms occur because of differences in the amino acid
sequences of enzymes.
Despite these differences in sequence, the enzymes fold to the same three-dimensional
structures and frequently exhibit similar affinities for and catalytic rates with substrates.

445
Q

A nonprotein molecule necessary for enzyme activity:

Activators
Apoenzymes
Coenzymes
Cofactors

A

Cofactors

In addition to the basic enzyme structure, a nonprotein molecule, called a cofactor, may be
necessary for enzyme activity.
Inorganic cofactors, such as chloride or magnesium ions, are called activators.
A coenzyme is an organic cofactor, such as NAD.

446
Q

Stated as E + S = ES = E + P

Catalytic mechanism
Henderson-Hasselbalch equation
Lineweaver-Burk plot
Combination reaction

A

Catalytic mechanism

447
Q

In liver or skeletal muscle disease, which LD isozymes become elevated
predominantly in serum?

LD1 and LD2
LD2 and LD3
LD3 and LD4
LD4 and LD5

A

LD4 and LD5

In myocardial damage, the predominant isozymes that become elevated in serum are LD1
and LD2; in liver or skeletal muscle disease, the LD4 and LD5 isozymes become elevated
predominantly in serum.

448
Q

This probably represents alcohol dehydrogenase:

LD1
LD3
LD5
LD6

A

LD6

Another band detected in electrophoresis and termed LD6 can be seen; this probably
represents alcohol dehydrogenase, which can also metabolize lactate.

449
Q

CHECK 3 BOXES: Causes of elevated serum levels of unconjugated
bilirubin:

Dubin-Johnson syndrome
Biliary obstruction
Hemolysis
Gilbert’s syndrome
Crigler-Najjar syndrome

A

Hemolysis
Gilbert’s syndrome
Crigler-Najjar syndrome

450
Q

CHECK 2 BOXES: Causes of elevated serum levels of conjugated bilirubin:

Dubin-Johnson syndrome
Biliary obstruction
Hemolysis
Gilbert’s syndrome
Crigler-Najjar syndrome

A

Dubin-Johnson syndrome
Biliary obstruction

451
Q

Given the common practice of dismissing mother and newborn baby
within 48 hours of the mother’s admission, it is likely that the neonate’s
thyroxine levels may:

Increase above the normal reference ranges due to starvation
Increase above the normal reference ranges due to hyperthyroidism
Fall below normal references ranges due to congenital hypothyroidism
Fall below normal references ranges due to inadequate protein feeding

A

Fall below normal references ranges due to inadequate protein feeding

The common recommendation for neonatal testing is that specimens
are collected when the newborn is at least 3 days old and after 24 hours of protein feeding
in order to adequately supply nutrients to produce thyroid hormones.
Given the common practice of dismissing mother and newborn baby
within 48 hours of the mother’s admission, it is likely that thyroxine levels may
occasionally fall below normal reference ranges due to inadequate protein feeding rather
than congenital hypothyroidism.

452
Q

It usually associated with a single, short-term exposure to a substance,
the dose of which is sufficient to cause immediate toxic effects:

Acute toxicity
Chronic toxicity
Either acute or chronic toxicity
None of these

A

Acute toxicity

Acute toxicity is usually associated with a single, short-term exposure to a substance, the
dose of which is sufficient to cause immediate toxic effects.

453
Q

It is usually associated with repeated frequent exposure for extended
periods for greater than 3 months and possibly years, at doses that are
insufficient to cause an immediate response:

Acute toxicity
Chronic toxicity
Acute and chronic toxicity
None of these

A

Chronic toxicity

Chronic toxicity is usually associated with repeated frequent exposure for extended
periods for greater than 3 months and possibly years, at doses that are insufficient to
cause an immediate acute response.
In many instances, chronic exposure is related to an accumulation of the toxicant or the
toxic effects within the individual. Chronic toxicity may affect different systems than those
associated with acute toxicity.

454
Q

This drug is traded on the streets under the name of angel dust or angel hair:

Cocaine
Amphetamine
Phencyclidine
Benzodiazepine

A

Phencyclidine

PHENCYCLIDINE
Used almost exclusively as a drug of abuse, this drug is traded on the streets under the
name of angel dust or angel hair

455
Q

Panic reactions—a bad trip—are the most common adverse reactions.

Methaqualone
Marijuana
Phencyclidine
Lysergic acid diethylamide

A

Lysergic acid diethylamide

Lysergic acid diethylamide (LSD) is a semisynthetic indolalkylamine and a hallucinogen.
LSD toxicity levels are low, and deaths are generally due to trauma secondary to errors in
the user’s judgment. Panic reactions—a bad trip—are the most common adverse reactions.

456
Q

The drug of choice for absence (petit mal) seizures unaccompanied by
other types of seizures:

Ethosuximide (Zarontin)
Phenytoin (Dilantin)
Primidone (Mysoline)
Valproic Acid (Depakene)

A

Ethosuximide (Zarontin)

Ethosuximide is the drug of choice for absence (petit mal) seizures unaccompanied by
other types of seizures. It is preferred over valproic acid, at least initially, because
hepatotoxicity is a rare but serious side effect of valproic acid.

457
Q

Anti-asthmatic drugs:

Digoxin
Digoxine and procainamide
Theophylline
Theophylline and theobromine

A

Theophylline and theobromine

Anti-asthmatic drugs, such as theophylline and theobromine, are used for treatment of
neonatal breathing disorders or of respiratory conditions that affect adults or children,
such as asthma.

458
Q

The most common drug of abuse and is frequently responsible for the
presentation of patients with altered mental status to hospitals and
emergency rooms:

Cocaine
Marijuana
Ethanol
Methanol

A

Ethanol

Ethanol is probably the most common drug of abuse and is frequently responsible for the
presentation of patients with altered mental status to hospitals and emergency rooms.
Peak plasma concentrations are usually reached within 1 hour after ingestion.

459
Q

Lead is generally measured in:

CSF
Serum
Plasma
Whole blood

A

Whole blood

Unlike many other toxins, lead is generally measured in whole blood rather than in serum
or plasma, because most of the circulating lead is bound within the blood cells.

460
Q

In an institution, a comprehensive program in which all areas of operation
are monitored to ensure quality with the aim of providing the highest
quality patient care:

Quality assessment
Quality assurance
Quality control
Quality systems

A

Quality systems

Quality assessment (QA) in the laboratory, a program that monitors the total testing
process with the aim of providing the highest quality patient care; formerly called quality
assurance.
Quality control (QC) a system that verifies the reliability of analytical test results through
the use of standards, controls, and statistical analysis.
Quality systems (QS) in an institution, a comprehensive program in which all areas of
operation are monitored to ensure quality with the aim of providing the highest quality
patient care.

461
Q

An indication of error in the analysis, detected by a progressive drift of
control values in one direction for at least 5 consecutive runs:

Dispesion
Shift
Trend
Random error

A

Trend

TREND: an indication of error in the analysis, detected by a progressive drift of control
values in one direction for at least 5 consecutive runs
SHIFT: an abrupt change from the established mean indicated by the occurrence of all
control values on one side of the mean
——
Assaying control specimens and standards along with patient specimens serves several
major functions:
1. Provides a guide to the functioning of equipment, reagents, and individual technique
2. Confirms the accuracy of testing when compared with reference values
3. Detects an increase in the frequency of both high and low minimally acceptable values
(dispersion)
4. Detects any progressive drift of values to one side of the average value for at least 3
days (trends)
5. Demonstrates an abrupt shift or change from the established average value for 3 days
in a row (shift)

462
Q

Centrifuge used when rapid centrifugation of solutions containing small
particles is needed:

Horizontal-head centrifuge
Swinging-bucket centrifuge
Fixed-angle centrifuge
Cytocentrifuge

A

Swinging-bucket centrifuge
Fixed-angle centrifuge

Fixed angle–head centrifuges are used when rapid centrifugation of solutions containing
small particles is needed; an example is the microhematocrit centrifuge.
Ultracentrifuges are high-speed centrifuges generally used for research projects, but for
certain clinical uses, a small air-driven ultracentrifuge is available that operates at 90,000
to 100,000 rpm and generates a maximum RCF of 178,000 g. Ultracentrifuges are often
refrigerated.

463
Q

Components of Quality Systems Program:

Personnel qualifications, training, and competency
Quality Assessment components including pre-analytical, analytical and postanalytical factors
Proficiency testing
All of these

A

All of these

COMPONENTS OF A QUALITY SYSTEMS PROGRAM
A comprehensive quality systems program is designed to follow a specimen all the way
through the testing process, from the time a test is ordered, through specimen collection
and testing to reporting, charting, and delivery and use of results. The scope of QS
programs differs among laboratories, but in general QS programs are broad, ongoing, and
encompass evaluation of:
1. Personnel qualifications, training, and competency
2. Quality Assessment components including preanalytical (before test) factors, analytical
factors and QC methods and postanalytical (after test) factors
3. Proficiency testin

464
Q

An example of analytical factor affecting laboratory test result:

Patient identification procedure
Specimen labeling and transport
Reporting and charting of test results
Instrument maintenance and calibration

A

Instrument maintenance and calibration

Analytical Factors Affecting Laboratory Test Results
Quality assessment programs also evaluate analytical factors that can affect the actual
test procedure. Many of these factors fall under the umbrella of QC. Analytical factors
include:
1. Instrument maintenance and calibration
2. Use of standards and procedural controls
3. Techniques and test components associated with performing the test procedure
(reagents, laboratory water, pipetting, timing, etc.)
4. Interfering substances or conditions
5. Statistical analysis of control results
An example of an analytical error is inaccurate sampling caused by problems with an
instrument’s automatic sampler, perhaps because of dirt or protein buildup in the sampling
probe. The error might be detected when control values show an abrupt or steady change.
Such an error can usually be prevented by performing recommended maintenance tasks.

465
Q

A method of monitoring accurate outcome; in which test samples from
an external source are analyzed and results compared to those of
reference laboratories and scored for accuracy:

Quality control
Quality assurance
Proficiency testing
Material safety data sheet

A

Proficiency testing

Proficiency testing (PT) is another component of laboratory quality programs.
Laboratories that perform nonwaived procedures are required to subscribe to an external
PT program. At regular intervals during the year, the PT agency sends blind samples to the
laboratory. These are samples that have been assayed multiple times by the PT agency.
The subscribing laboratory analyzes these blind samples and sends the results to the PT
agency. The subscribing laboratory’s results are then compared to the PT agency’s
assayed values and to the results of peer laboratories participating in the PT program.
Participating laboratories receive a report evaluating their performance

466
Q

Tests that are very simple or pose no reasonable risk of harm to the
patient if the test is performed incorrectlyTests that are very simple or pose no reasonable risk of harm to the
patient if the test is performed incorrectly:

Waived
Nonwaived

A

Waived

467
Q

Complex tests that require skill to perform and interpret and are therefore
regulated:

Waived
Nonwaived

A

Nonwaived

468
Q

A procedure with minimal complexity, instrumentation, and personnel
requirements so that the results can be quickly determined:

Definitive test
Presumptive test

A

Presumptive test

Presumptive test
A procedure with minimal complexity, instrumentation, and personnel requirements so that
the results can be quickly determined.

469
Q

Highly sensitive and specific test in which results can be used as legal
evidence:

Definitive test
Presumptive test

A

Definitive test

Definitive test
Highly sensitive and specific test in which results can be used as legal evidence

470
Q

It measures light blocked as a decrease in the light transmitted through
the solution; dependent on particle size and concentration:

Nephelometry
Turbidimetry
Fluorometry
Chromatography

A

Turbidimetry

Turbidimetry measures light blocked as a decrease in the light transmitted through the
solution; dependent on particle size and concentration.

471
Q

Ultraviolet (UV) light has _______ wavelengths.

Slightly short wavelengths
Very short wavelengths
Slightly long wavelengths
Very long wavelengths

A

Very short wavelengths

Ultraviolet (UV) light has very short wavelengths and infrared (IR) light has very long
wavelengths. When all visible wavelengths of light (400-700 nm) are combined, white light
results

472
Q

Which of the following represents a primary advantage of performing
fluorometric over absorption spectroscopic methods of analysis?

Increased specificity and increased sensitivity
Increased specificity and decreased sensitivity
Purity of reagents used not as critical
Ease of performing assays

A

Increased specificity and increased sensitivity

Fluorometric methods are extremely sensitive and highly specific. Because of this extreme
sensitivity, reagents used must be of a higher degree of purity than is required for
spectroscopy, because even slight traces of impurities may fluoresce.

473
Q

Large particles scattering light predominantly a in the forward direction:

Mie scatter
Raleigh scatter

A

Mie scatter

Mie scatter - large particles scattering light predominantly a in the forward direction
Raleigh scatter - small particles scattering light in all directions
with maximum scatter forward and backward

474
Q

Small particles scattering light in all directions with maximum scatter
forward and backward:

Mie scatter
Raleigh scatter

A

Raleigh scatter

Mie scatter - large particles scattering light predominantly a in the forward direction
Raleigh scatter - small particles scattering light in all directions
with maximum scatter forward and backward

475
Q

Increasing temperature:

Fluorescence intensity increases
Fluorescence intensity decreases
Variable fluorescence
No relationship

A

Fluorescence intensity decreases

In general, fluorescence intensity decreases with increasing temperature by approximately
1 to 5% per degree Celsius.

476
Q

Pipettes have a cylindrical glass bulb near the center of the pipette that
helps to distinguish them from other types of transfer pipettes:

Serological measuring pipettes
Mohr pipettes
Volumetric transfer pipettes
Ostwald-Folin transfer pipettes

A

Volumetric transfer pipettes

477
Q

Pipettes used for delivering small volumes of viscous solutions such as
protein or whole blood standards:

Serological measuring pipettes
Mohr pipettes
Volumetric transfer pipettes
Ostwald-Folin transfer pipettes

A

Ostwald-Folin transfer pipettes

478
Q

High-speed centrifuges generally used for research projects, but for
certain clinical uses, a small air-driven ultracentrifuge is available that
operates at 90,000 to 100,000 rpm and generates a maximum RCF of
178,000 g; often refrigerated:

Horizontal-head centrifuge
Fixed angle-head centrifuge
Ultracentrifuge
Cytocentrifuge

A

Ultracentrifuge

479
Q

A very high-torque and low-inertia motor to spread monolayers of cells
rapidly across a special slide for critical morphologic studies:

Horizontal-head centrifuge
Fixed angle-head centrifuge
Ultracentrifuge
Cytocentrifuge

A

Cytocentrifuge

This type of preparation can be used for blood, urine, body fluid, or any other liquid
specimen that can be spread on a slide. An advantage of this technology is that only a
small amount of sample is used, producing evenly distributed cells that can then be
stained for microscopic study. The slide produced can be saved and examined at a later
time, in contrast to “wet” preparations, which must be examined immediately.

480
Q

Fist pumping during venipuncture:

Decreased potassium and calcium
Decreased potassium, increased calcium
Increased potassium and calcium
Increased potassium, decreased calcium

A

Increased potassium and calcium

Fist pumping during venipuncture
↑ K+, lactic acid, Ca2+, phosphorus; ↓ pH
Pumping of the fist before venipuncture should be avoided because it causes an increase
in plasma potassium, phosphate, and lactate concentrations. Lowering of blood pH by
accumulation of lactate causes the plasma ionized calcium concentration to increase.

481
Q

Hemolysis is graded based on visible presence of hemoglobin, when
greater than _____ mg/dL.

2 mg/dL
10 mg/dL
12 mg/dL
20 mg/dL

A

20 mg/dL

Hemolysis is generally a preanalytical problem that can be avoided. It is graded based on
visible presence of hemoglobin, when greater than 20 mg/dL, and it is often graded as
mild, moderate, or gross hemolysis.

482
Q

Glucose measurements can be _______by reducing methods than by more
accurate enzymatic methods that are highly specific for glucose.

1 to 5 mg/dL erroneously higher
1 to 5 mg/dL erroneously lower
5 to 15 mg/dL erroneously higher
5 to 15 mg/dL erroneously lower

A

5 to 15 mg/dL erroneously higher

Glucose measurements can be 5 to 15 mg/dL erroneously higher by reducing methods
than by more accurate enzymatic methods that are highly specific for glucose.

483
Q

Simple but nonspecific assay for creatinine:

Enzymatic
Colorimetric: endpoint
Colorimetric: kinetic
None of these

A

Colorimetric: endpoint

484
Q

Assay for uric acid that deals with turbidity problems and drug
interferences:

Colorimetric
Enzymatic: UV
Enzymatic: H2O2
None of these

A

Colorimetric

485
Q

Using standard serum protein electrophoresis (SPE) methods, serum
proteins appear in ____ bands.

Four bands
Five bands
Six bands
Twelve bands

A

Five bands

Standard SPE separates the protein into 5 distinct bands but, by modifying the
electrophoretic parameters, proteins can be further separated into as many as 12 bands.

The modification, known as high-resolution protein electrophoresis (HRE), uses a higher
voltage coupled with a cooling system in the electrophoretic apparatus and a more
concentrated buffer.

486
Q

What may be the cause of NEONATAL PHYSIOLOGICAL JAUNDICE of the
hepatic type?

Hemolytic episode caused by an ABO incompatibility
Structure of the common bile duct
Hemolytic episode caused by an Rh incompatibility
Deficiency in the bilirubin conjugation enzyme system

A

Deficiency in the bilirubin conjugation enzyme system

The increased levels of unconjugated bilirubin will cause the infant to appear jaundiced.
Generally, this condition persists for only a short period because the enzyme system
usually becomes functional within several days after birth. Neonatal physiological
jaundice resulting from an enzyme deficiency is hepatic in origin.

487
Q

Which of the following is not a type of support media used for serum
protein electrophoresis?

Agarose gel
Cellulose acetate
Acrylamide
Celite

A

Celite

Support media that may be used for electrophoretic separations
include agarose gel, starch gel, cellulose acetate, and acrylamide.
Celite provides the inert supporting phase in gas-liquid chromatography

488
Q

What dye may be used for staining protein bands following electrophoresis?

Fat red 7B
Sudan black B
Ponceau S
Oil red O

A

Ponceau S

Amido black 10B, Coomassie brilliant blue, and Ponceau S are dyes that are used to stain
serum proteins after electrophoresis.
Oil red O and fat red 7B are dyes that are used to stain lipoproteins following
electrophoresis.

489
Q

Using standard SPE, which protein travels farthest to the anode?

Alpha1-globulins
Alpha2-globulins
Beta-globulins
Gamma-globulins
Albumin

A

Albumin

Using standard SPE methods, serum proteins appear in five bands: albumin travels
farthest to the anode, followed by α1-globulins, α2-globulins, β-globulins, and γ-globulins,
in that order.

490
Q

Which term describes a congenital disorder that is characterized by a
split in the albumin band when serum is subjected to electrophoresis?

Analbuminemia
Anodic albuminemia
Prealbuminemia
Bisalbuminemia

A

Bisalbuminemia

Bisalbuminemia is a congenital disorder that does not exhibit any clinical manifestations.
The only sign of this disorder is the splitting of albumin into two distinct bands when
serum is subjected to electrophoresis.

491
Q

All but one protein are components of the BETA-GLOBULINS:

Ceruloplasmin
Transferrin
Hemopexin
Complement components

A

Ceruloplasmin

CERULOPLASMIN - ALPHA2-GLOBULIN

492
Q

Which of the following dyes is the most specific for measurement of
albumin?

Bromcresol green (BCG)
Bromcresol purple (BCP)
Tetrabromosulfonthalein
Tetrabromphenol blue

A

Bromcresol purple (BCP)

BCP is more specific for albumin than BCG.
However, BCG is the method used most often. One reason for this is that renal dialysis
patients produce an organic acid that competes with BCP for the binding site on albumin,
causing a falsely low result.

493
Q

The physician is concerned that a pregnant patient may be at risk for
delivering prematurely. What would be the best biochemical marker to
measure to assess the situation?

Inhibin A
Alpha-fetoprotein
Fetal fibronectin
Human chorionic gonadotropin

A

Fetal fibronectin

The level of fetal fibronectin increases in the secretions of the cervix and vagina. When
this occurs prematurely, the increase in fetal fibronectin is used to predict risk of
premature birth.
Inhibin A, alpha-fetoprotein, human chorionic gonadotropin, and unconjugated estriol are
used together in the quadruple test to assess risk for such disorders as Down syndrome.

494
Q

In which of the following disorders would the maternal serum level of
alpha-fetoprotein not be elevated?

Neural tube defect
Spina bifida
Fetal distress
Down syndrome

A

Down syndrome

An increased AFP level in maternal serum is associated with such disorders as neural tube
defects, spina bifida, and fetal distress.
A decreased AFP level in maternal serum is characteristic of Down syndrome.

495
Q

Which of the following is not quantified in the triple test for Down
syndrome?

Alpha-fetoprotein
Unconjugated estriol
Progesterone
Human chorionic gonadotropin

A

Progesterone

SCREENING FOR BIRTH DEFECTS
1. Triple test: AFP, HCG and ESTRIOL
2. Quadruple (QUAD) screen: AFP, HCG, ESTRIOL and INHIBIN-A

496
Q

All antigen diffuse from the well and the concentration of the antigen is
related to the square of the diameter of the precipitin ring:

Fahey-McKelvey or the kinetic RID
Mancini or the endpoint RID

A

Mancini or the endpoint RID

The endpoint method requires that all Ag diffuse from the well and the concentration of
the Ag is related to the square of the diameter of the precipitin ring; the standard curve is
plotted and a line of the best fit is computed.

To ensure that all of the Ag has diffused, the incubation time is 48 to 72 hours, depending
on the molecular weight of the Ag; for example, IgG quantitation requires 48 hours, and
IgM requires 72 hours.

For those performing RID, the endpoint method is favored because of its stability and
indifference to temperature variations; however, turnaround time is longer compared with
the kinetic method.

497
Q

It requires that all rings be measured at a fixed time of 18 hours; diameter
of the precipitin ring is plotted against the antigen concentration on a
logarithmic scale:

Fahey-McKelvey or the kinetic RID
Mancini or the endpoint RID

A

Fahey-McKelvey or the kinetic RID

The kinetic method requires that all rings be measured at a fixed time of 18 hours; a
sample with a greater concentration will diffuse at a faster rate and will be larger at a fixed
time.
The diameter of the precipitin ring is plotted against the Ag concentration on a logarithmic
scale.

498
Q

To produce reliable results, when should blood specimens for lipid
studies be drawn?

Immediately after eating
Anytime during the day
In the fasting state, approximately 2 to 4 hours after eating
In the fasting state, approximately 12 hours after eating

A

In the fasting state, approximately 12 hours after eating

499
Q

Direct measurements of T4, except:

Dialysis
Immunoassay
Ultrafiltration
Uptake

A

Uptake

The FTI has been largely replaced by the direct measurement of free T4 by immunoassay,
dialysis, or ultrafiltration.

500
Q

Errors in LDL-C become noticeable at triglycerides > _____ mg/dL and
become unacceptably large at triglyceride levels > _____ mg/dL.

> 50 mg/dL, > 150 mg/dL
100 mg/dL, > 200 mg/dL
200 mg/dL, > 400 mg/dL
300 mg/dL, > 600 mg/dL

A

> 200 mg/dL, > 400 mg/dL

Errors in LDL-C become noticeable at triglyceride levels >2.26 mmol/L (200 mg/dL) and
become unacceptably large at triglyceride levels >4.52 mmol/L (400 mg/dL).

501
Q

The Friedewald formula for computation of LDL-cholesterol is not valid
for triglycerides over ___ mg/dL.

Over 100 mg/dL
Over 200 mg/dL
Over 300 mg/dL
Over 400 mg/dL

A

Over 400 mg/dL

The equation should not be used with triglyceride values exceeding 400 mg/dL because
the VLDL composition is abnormal, making the [triglyceride/5] factor inapplicable

502
Q

Compete with the substrate for the active site of the enzyme and prevent
formation of product, but have a higher Km than the preferred substrate
and can be overcome by addition of more substrate:

Competitive inhibitors
Noncompetitive inhibitors
Unncompetitive inhibitors

A

Competitive inhibitors

Competitive inhibitors: compete with the substrate for the active site of the enzyme and
prevent formation of product, but have a higher Km than the preferred substrate and can
be overcome by addition of more substrate.

Noncompetitive inhibitors: bind on a different active site of the enzyme than the substrate
and so cannot be overcome by addition of more substrate, but prevent formation of
product despite the enzyme-substrate complex.

Uncompetitive inhibitors: bind to the enzyme-substrate complex and prevent the formation
of product. This type of inhibition is also reversible.

503
Q

Bind on a different active site of the enzyme than the substrate and so
cannot be overcome by addition of more substrate, but prevent formation
of product despite the enzyme-substrate complex:

Competitive inhibitors
Noncompetitive inhbitors
Uncompetitive inhibitors

A

Noncompetitive inhbitors

Competitive inhibitors: compete with the substrate for the active site of the enzyme and
prevent formation of product, but have a higher Km than the preferred substrate and can
be overcome by addition of more substrate.
Noncompetitive inhibitors: bind on a different active site of the enzyme than the substrate
and so cannot be overcome by addition of more substrate, but prevent formation of
product despite the enzyme-substrate complex.
Uncompetitive inhibitors: bind to the enzyme-substrate complex and prevent the formation
of product. This type of inhibition is also reversible.

504
Q

Bind to the enzyme-substrate complex and prevent the formation of
product:

Competitive inhibitors
Noncompetitive inhibitors
Uncompetitive inhibitors

A

Uncompetitive inhibitors

Competitive inhibitors: compete with the substrate for the active site of the enzyme and
prevent formation of product, but have a higher Km than the preferred substrate and can
be overcome by addition of more substrate.
Noncompetitive inhibitors: bind on a different active site of the enzyme than the substrate
and so cannot be overcome by addition of more substrate, but prevent formation of
product despite the enzyme-substrate complex.
Uncompetitive inhibitors: bind to the enzyme-substrate complex and prevent the formation
of product. This type of inhibition is also reversible.

505
Q

The reactants are combined, the reaction proceeds for a designated time,
the reaction is stopped (usually by inactivating the enzyme with a weak
acid),and a measurement of the amount of reaction that has occurred is
made:

Kinetic method
Fixed-time method

A

Fixed-time method

In the fixed-time method, the reactants are combined, the reaction proceeds for a
designated time, the reaction is stopped (usually by inactivating the enzyme with a weak
acid), and a measurement of the amount of reaction that has occurred is made.

506
Q

Multiple measurements, usually of absorbance change, are made during
the reaction, either at specific time intervals (usually every 30 or 60
seconds) or continuously by a continuous-recording spectrophotometer:

Kinetic method
Fixed-time method

A

Kinetic method

In continuous-monitoring or kinetic assays, multiple measurements, usually of absorbance
change, are made during the reaction, either at specific time intervals (usually every 30 or
60 seconds) or continuously by a continuous-recording spectrophotometer.

507
Q

Most labile LD isoenzyme; loss of activity occurs more quicklyat 4°C than
at 25°C.

LD-1
LD-2
LD-3
LD-4
LD-5

A

LD-5

LD-5 is the most labile isoenzyme. Loss of activity occurs more quickly at 4°C than at
25°C. Serum samples for LD isoenzyme analysis should be stored at 25°C and analyzed
within 24 hours of collection.

508
Q

The International Federation for Clinical Chemistry (IFCC) recommends
the use of methods such as the Bessey-Lowry-Brock method for
determining alkaline phosphatase activity. The substrate used in this
type of method is:

Monophosphate
Phenylphosphate
Disodium phenylphosphate
Para-nitrophenylphosphate

A

Para-nitrophenylphosphate

ALP catalyzes the hydrolysis of para-nitrophenyl phosphate forming phosphate and free 4-
nitrophenyl (4-NPP) which, under alkaline conditions has very intense yellow color.

509
Q

Of the total serum osmolality, sodium, chloride, and bicarbonate ions
normally contribute approximately what percent?

8%
45%
92%
98%

A

92%

For monovalent cations or anions the contribution to osmolality is approximately 92%.
Other serum electrolytes, serum proteins, glucose, and urea contribute to the remaining 8%

510
Q

Hepatic cirrhosis:

Hypernateremia due to excess water loss
Hypernatremia due to decreased water intake
Hyponatremia due to increased sodium loss
Hyponatremia due to increased water retention

A

Hyponatremia due to increased water retention

511
Q

Diabetes insipidus:

Hypernatremia due to decreased water intake
Hypernatremia due to excess water loss
Hyponatremia due to increased sodium loss
Hyponatremia due to increased water retention

A

Hypernatremia due to excess water loss

512
Q

Artifactual hyperkalemia:

Sample hemolysis
Sample hemolysis, thrombocytosis
Sample hemolysis, excessive fist clenching
Sample hemolysis, thrombocytosis, prolonged tourniquet use or excessive fist
clenching

A

Sample hemolysis, thrombocytosis, prolonged tourniquet use or excessive fist
clenching

513
Q

Hormonal regulation of calcium:

Calcitonin, parathyroid hormone
Calcitonin, vitamin D
Parathyroid hormone, vitamin D
Parathyroid hormone, calcitonin and vitamin D

A

Parathyroid hormone, calcitonin and vitamin D

514
Q

It corrects renal blood flow in the following ways: causing vasodilation of
the afferent arterioles and constriction of the efferent arterioles,
stimulating reabsorption of sodium and water in the proximal convoluted
tubules, and triggering the release of the sodium-retaining hormone
aldosterone by the adrenal cortex and antidiuretic hormone by the
hypothalamus:

Aldosterone
Angiotensin I
Angiotensin II
Renin

A

Angiotensin II

515
Q

Increased antidiuretic hormone:

Fluid loss, low serum sodium
Fluid loss, high serum sodium
Fluid retention, low serum sodium
Fluid retention, high serum sodium

A

Fluid retention, low serum sodium

516
Q

Decreased anion gaps of less than 10 mmol/L

-Decreased measured cations or increased measured anions
-Decreased unmeasured cations or decreased unmeasured anions
-Increased measured cations or decreased measured anions
-Increased unmeasured cations or decreased unmeasured anions
-Increased unmeasured cations and anions
Decreased unmeasured cations and anions

A

Increased unmeasured cations or decreased unmeasured anions

Decreased anion gaps of less than 10 mmol/L: either an increase in unmeasured cations
(Ca2+, Mg2+) or a decrease in the unmeasured anions
Anion gap exceeds 16 mmol/L Indication of increased concentrations of the unmeasured
anions (PO4 3−, SO4 2−, protein ions)

517
Q

Serum osmolality increases by about ____ mOsm/kg for each 60 mg/dL
increase in serum ethanol.

1 mOsm/kg
10 mOsm/kg
15 mOsm/kg
20 mOsm/kg

A

10 mOsm/kg

The degree of increase in osmolality due to ethanol is expressed as the difference
between the measured and the calculated osmolality; the difference is called the osmolar
gap.
Serum osmolality increases by about 10 mOsm/kg for each 60 mg/dL increase in serum
ethanol.

518
Q

Blood received in the laboratory for blood gas analysis must meet which
of the following requirements?

On ice, thin fibrin strands only, no air bubbles
On ice, no clots, fewer than 4 air bubbles
On ice, no clots, no air bubbles
Room temperature, no clots, no air bubbles

A

On ice, no clots, no air bubbles

Specimen requirements for blood gas analysis include the following.
1. Arterial blood is collected in a glass or plastic syringe. Capillary specimens can also be
used (blood must be “arterialized”).
2. Lyophilized or liquid heparin is the preferred anticoagulant.
3. No air bubbles should exist in the sample because they lower the pCO2 value.
4. The specimen must be placed on ice and transported to the laboratory in 15 minutes at
4◦C and tested immediately. Otherwise, pH values decrease, and pCO2 values increase.
5. Blood clots are unacceptable

519
Q

For each degree of fever in the patient, pO2 will ____ and pCO2 will ______.

pO2 an pCO2 will fall 7% and 3% respectively
pO2 and pCO2 will rise 3% and 7% respectively
pO2 will rise 7% and pCO2 will fall 3%
pO2 will fall 7% and pCO2 will rise 3%

A

pO2 will fall 7% and pCO2 will rise 3%

519
Q

What is the predominant form of thyroid hormone in the circulation?

Thyroxine
Triiodothyronine
Diiodotyrosine
Monoiodotyrosine

A

Thyroxine

T4 is the predominant form of the thyroid hormones secreted into the circulation, having a
concentration in the plasma significantly greater than T3.
However, in terms of physiological activity, T3 must be considered because it is four to five
times more potent than T4.

520
Q

The recommended initial thyroid function test for either a healthy,
asymptomatic patient or a patient with symptoms which may be related
to a thyroid disorder is:

Free thyroxine (free T4)
Thyroid-stimulating hormone (TSH)
Total thyroxine (T4)
Triiodothyronine (T3)

A

Thyroid-stimulating hormone (TSH)

TSH is the American Thyroid Association’s recommended screening test.

521
Q

Patient has signs and symptoms suggestive of acromegaly. The
diagnosis would be confirmed if the patient had which of the following?

An elevated serum phosphate concentration
A decreased serum growth hormone releasing factor concentration
No decrease in serum growth hormone concentration 90 minutes after oral
glucose administration
An increased serum somatostatin concentration

A

No decrease in serum growth hormone concentration 90 minutes after oral
glucose administration

Following an overnight fast, a 100-gram oral glucose load will cause a large drop in serum
growth hormone in a normal individual, but will not suppress in patients with acromegaly.

522
Q

The target DNA must be denatured to ____ before it can serve as a
template for the PCR reaction.

Double-stranded DNA
Single-stranded DNA
Either dsDNA or ssDNA
None of these

A

Single-stranded DNA

The target DNA must be denatured to single-stranded DNA before it can serve as a
template for the PCR reaction.

523
Q

An isotonic saline solution contains 0.85% NaCl. How many grams of
NaCl are needed to prepare 5 L of this solution?

4.25 grams
8.5 grams
42.5 grams
425 gramss

A

42.5 grams